PBL Exam 2 Practice Questions

Pataasin ang iyong marka sa homework at exams ngayon gamit ang Quizwiz!

E

During which phase of the cardiac cycle does the mitral valve open? (A) Atrial systole (B) Isovolumetric ventricular contraction (C) Rapid ventricular ejection (D) Reduced ventricular ejection (E) Isovolumetric ventricular relaxation (F) Rapid ventricular filling (G) Reduced ventricular filling (diastasis)

D

During which phase of the cardiac cycle is aortic pressure highest? (A) Atrial systole (B) Isovolumetric ventricular contraction (C) Rapid ventricular ejection (D) Reduced ventricular ejection (E) Isovolumetric ventricular relaxation (F) Rapid ventricular filling (G) Reduced ventricular filling (diastasis)

E

During which phase of the cardiac cycle is ventricular volume lowest? (A) Atrial systole (B) Isovolumetric ventricular contraction (C) Rapid ventricular ejection (D) Reduced ventricular ejection (E) Isovolumetric ventricular relaxation (F) Rapid ventricular filling (G) Reduced ventricular filling (diastasis)

C

During which phase of the ventricular action potential is the conductance to Ca2+ highest? (A) Phase 0 (B) Phase 1 (C) Phase 2 (D) Phase 3 (E) Phase 4

E

During which phase of the ventricular action potential is the membrane potential closest to the K+ equilibrium potential? (A) Phase 0 (B) Phase 1 (C) Phase 2 (D) Phase 3 (E) Phase 4

B

Each lymph node filters ______ & provides a site for ____ activation & differentiation to antibody-secreting ______ cells (A) blood, B-cell, macrophage (B) lymph, B-cell, plasma (C) lymph, T-cell, regulatory T cells (D) blood, T-cell, cytotoxic T cells

E

Edema occurs when volume of interstitial fluid exceeds capacity of lymphatics to return it to circulation and can be caused by (A) Excess filtration (B) Excess absorption (C) Blocked lymphatics (D) B & C (E) A & C

phenylalanine, tyrosine

PKU is deficiency of phenylalanine hydroxylase. This deficiency causes high _______ and low ________, which in turn causes low melanin

D

PS & other phospholipids w/ an alcohol head group synthesized by an alternative pathway. PA is activated by CTP to form → ______ (A) G3P (B) ATP-DAG (C) PC (D) CDP-DAG

B

PTH receptors are found on _______ that respond by secreting RANKL to stimulate osteoclast formation & activity (A) osteoclasts (B) osteoblasts (C) osteocytes (D) osteoprogenitor cells

E

Parasympathetic control of ionotropism (contractility) of atria is via ______ receptors coupled to adenyl cyclase via inhibitory G protein (A) alpha 1 (B) alpha 2 (C) beta 1 (D) beta 2 (E) muscarinic

D

Effective vaccines can be developed if which of the following criteria are met I. Infectious agent should not establish latency II. Agent should not undergo antigenic variation III. Infectious agent should integrate its genome into host chromosome IV. Infectious agent should not interfere with immune response (A) I & III (B) I & II (C) II, III, & IV (D) I, II, & IV

D

Elevated LDH is a sign of _____ (A) vitamin B12 deficiency (B) liver disease (C) spina bifida (D) hemolysis

D

Elevated values indicate: -Multiple pregnancy -Hydatidiform mole -Gestational trophoblastic neoplasia (choriocarcinoma) (A) AFP (B) estriol unconjugated (C) progesterone (D) hCG (E) oxytocin (F) hPL

primary, secondary

Endocrine failure of testis may be _______, due to trauma or inflammation of the testes or ________ due to failure of hypothalamus or pituitary

E

Endothelium restored → clot dissolved by plasmin which is formed by plasminogen activators from endothelium on plasminogen from plasma (A) primary aggregation (B) secondary aggregation (C) Blood coagulation (D) Clot retraction (E) Clot removal

lower, lower

Entire pulmonary vasculature is at much (lower/higher) pressure and (lower/higher) resistance than systemic vasculature

C

Peripheral chemoreceptors in carotid & aortic bodies are primarily sensitive to (A) changes in blood volume (B) decreases in hormone balance (C) decreases in PO2 (D) changes in Pa (mean arterial pressure)

B

Phenylketonuria is an inborn error of metabolism, which results from (A) deficiency in enzyme tyrosine hydroxylase (B) deficiency in enzyme phenylalanine hydroxylase (C) overactive phenylalanine hydroxylase (D) overactive tyrosine hydroxylase

C

Phosphorylation of _________ stimulates Ca2+ ATPase in SR → greater uptake & storage of Ca2+ → faster relaxation and increased amount of stored Ca2+ for release (A) calmodulin (B) protein kinase A (C) phospholamban (D) actin

oppose

Pi is the interstitial fluid hydrostatic pressure. Increases in Pi (favor/oppose) filtration out of capillary

C, D, A, E, F, B

Place the following stages of erythropoiesis in order: A. Polychomatophilic erythroblast B. Erthyrocytes C. Proerythroblasts D. Basophilic Erythroblast E. Orthochromatophilic erythroblasts (normoblasts) F. Reticulocytes

C

Excess ____ → precocious puberty Deficiency in ____ → secondary hypogonadism (A) ACTH (B) TSH (C) FSH/LH (D) Growth Hormone (E) Prolactin

bilirubin, heme

Excess _______, which is the catabolic product of _______, causes jaundice

C

Extreme hyperprolactinemia is highly suggestive of prolactinoma in patients not taking _____ drugs (A) dopaminergic (B) gabanergic (C) antidopaminergic (D) progesterone

C

Fibrinogen, vWF, platelet factor 4, & other proteins promote cascade of plasma proteins → fibrin polymer → blood clot (thrombus) (A) primary aggregation (B) secondary aggregation (C) Blood coagulation (D) Clot retraction (E) Clot removal

A

Preload of left ventricle = left ventricular end diastolic volume (A) Degree of overlap of thick & thin filaments (B) Increasing muscle length which increases Ca2+ sensitivity to troponin C (C) Increasing muscle length → increases Ca2+ release from SR (D) A & B (E) All of the above

C

From PCT, filtrate flow into ______, located in medulla with its squamous thin descending & ascending limbs (A) renal corpuscle (B) connecting tubules (C) Loop of Henle (D) DCT

C

Giant cells with large polyploid nuclei with irregularly lobulated coarse chromatin that are rich in MT, golgi, & RER (A) Myelocyte (B) Reticulocyte (C) Megakaryoblast (D) Monocyte

glucose, urea, carbons

Glucose-alanine cycle allows (1) net conversion of AA carbons to _______, (2) elimination of AA nitrogen as _____, and (3) return of _____ to the peripheral tissues as glucose

C

Granulocyte that destroys parasites and participates in allergic reaction (A) neutrophils (B) basophils (C) eosinophils (D) B lymphocytes (E) T lymphocytes (F) macrophages

A

Granulocyte that destroys small organisms (A) neutrophils (B) basophils (C) eosinophils (D) B lymphocytes (E) T lymphocytes (F) macrophages

B

Granulocyte that secretes histamine, mediates inflammatory responses, & secretes platelet-activating factor (A) neutrophils (B) basophils (C) eosinophils (D) B lymphocytes (E) T lymphocytes (F) macrophages

B

Granulosa cells secrete _____, a heterodimeric glycoprotein composed of an alpha-subunit linked by a disulfide bridge to one of 2 homologous beta subunits (A) estradiol (B) inhibin (C) progesterone (D) cAMP (E) selectin (F) prostaglandin

C

HBV & HPV vaccines are examples of (A) live attenuated vaccines (B) inactivated vaccines (C) subunit vaccines (D) all of the above

Volkmann's canal

HORIZONTAL connects central (haversian) canals

A, E, C, F, I, B, G, D, H, J

Put the following in the correct order of blood flow through the heart: A. Right atrium B. Left atrium C. Right ventricle D. Left ventricle E. Tricuspid (Right AV) valve F. Pulmonary valve G. Mitral (bicuspid/Left AV) valve H. Aortic valve I. Lungs J. Body

B

RBC plasmalemma is (A)10% lipid, 40% carbohydrate, 50% protein (B) 40% lipid, 10% carbohydrate, 50% protein (C) 20% lipid, 30% carbohydrate, 50% protein (D) 50% lipid, 10% carbohydrate, 40% protein

pluripotent stem cell

RBCs, WBCs, & platelets have a common ancestry in the bone marrow, which is a _______ _______ _______

C

Heart rate is the number of ______ or R waves per minute (A) P wave (B) PR interval (C) QRS complex (D) T wave (E) QT interval

B

Heme controls its synthesis rate by feedback inhibition of ______ (A) ferrochelatase (B) 5-ALA synthase (C) PBG (D) 5-ALA

C

Hereditary hemochromatosis is a genetically determined disorder of ____ metabolism (A) copper (B) heme (C) iron (D) fat

B

Hormone secreted by posterior pituitary gland that regulates blood flow osmolarity & participates in regulation of Pa (A) Aldosterone (B) ADH or Vasopressin (C) Cortisol (D) Leptin

low, high

Hyperthyroidism (overactive thyroid) can be caused by grave's disease, (high/low) TSH, or (high/low) fT3 and/or fT4

high, low

IGF-1 would be increased in patients with (low/high) GH IGF-1 would be reduced in patients with (low/high) GH

C

If abdomen is protuberant but there is no fluid wave to indicate fluid in the abdomen, what is the cause of the protuberance? (A) Heart failure (B) Bacterial infection (C) organ enlargement (D) virus

C

If the heart rate is 70 beats/min, then the cardiac output of this ventricle is closest to (A) 3.45 L/min (B) 4.55 L/min (C) 5.25 L/min (D) 8.00 L/min (E) 9.85 L/min

C

Immunogenicity improved by conjugation to a protein carrier → conjugate vaccines are a subset of what type of vaccine? (A) live attenuated vaccines (B) inactivated vaccines (C) subunit vaccines (D) all of the above

heavy

Immunoglobulins share a common Y-shaped structure of 2 heavy & 2 light chains. The nature of the ______ chain determines the class of the Ig

A

Impaired clearance of ________ due to an impaired urea cycle causes brain damage (A) ammonia (NH3) (B) ammonium ions (NH4+) (C) chloride (Cl-) (D) potassium (K+)

phenylalanine, tyrosine

In PKU, mental retardation can be avoided w/ early detection & use of diet w/ limited _______ but supplemented w/ _______

A

In _______, P wave (atrial depolarization) → atrial contraction → increase in L atrial pressure → ventricle fills with blood from atrium (A) Atrial Systole (B) Isovolumetric Ventricular Contraction (C) Rapid Ventricular Ejection (D) Reduced Ventricular Ejection (E) Isovolumetric Ventricular Relaxation (F) Rapid Ventricular Filling (G) Reduced Ventricular Filling or Diastasis

F

In _______, the impulse is blocked in the bundle of His. Every few beats there will be a missing beat and the PR interval will _______ (A) 1st degree, stay the same (B) 1st degree, lengthen (C) 2nd degree type 1, shorten (D) 2nd degree type 2, lengthen (E) 2nd degree type 1, stay the same (F) 2nd degree type 2, stay the same (G) 3rd degree, shorten (H) 3rd degree, lengthen

cortisol, increase, decrease, increase

In congenital adrenal hyperplasia, an enzyme defect in ______ synthesis causes an accumulation of its precursors, some of which have strong aldosterone-like activity. These act to (increase/decrease) serum Na+ levels, (increase/decrease) K+ levels, & (increase/decrease) blood volume, all of which diminish angiotensin levels

forebrain, midbrain, hindbrain

In early development, the neural tube dilates at the cephalic end into what 3 vesicles?

E

In the diaphysis of a typical long bone which of the following structures is in closest proximity to the trabeculae of cancellous bone? (A) Interstitial lamellae (B) Osteons (C) Sharpey fibers (D) Outer circumferential lamellae (E) Inner circumferential lamellae

B

In the sinoatrial (SA) node, phase 4 depolarization (pacemaker potential) is attributable to (A) an increase in K+ conductance (B) an increase in Na+ conductance (C) a decrease in Cl- conductance (D) a decrease in Ca2+ conductance (E) simultaneous increases in K+ and Cl- conductances

C

In which of the following situations would you most likely administer a single dose of parenteral dexamethasone to reduce edema & trismus? (A) oral ulcerations, such as denture-induced ulcers or angular cheilitis (B) TMJ disorders - refractory cases or severe acute pain (C) Postoperative sequelae (D) allergic reactions

parts of a differential (CBC)

Includes: -Percentage of PMNs (polymorphonuclear neutrophils) -Percentage of lymphocytes -Percentage of monocytes -Percentage of eosinophils

A

Integral membrane proteins that have their kinase domain activated after ligand-induced crosslinking (A) Receptor Tyrosine Kinases (RTKs) (B) G-protein coupled receptors (C) Nuclear receptors (D) Nonreceptor tyrosine kinase

C

Intervertebral disc is a synarthroses in vertebral column that cushion adjacent vertebrae. Each disc has a thick outer layer of fibrocartilage forming a tough _______ and a shock-absorbing inner, gel-like core, ________ (A) synostoses, annulus fibrosus (B) nucleus pulposus, annulus fibrosis (C) annulus fibrosis, nucleus pulposis (D) synostoses, nucleus pulposis

D

Is derived from the alar plate (A) GSE column (B) GVE column (C) alar plate (D) pontine nuclei (E) SVE column

C

Is derived from the alar plate (A) corticospinal pyramid (tract) (B) tela choroidea (C) inferior olivary nucleus (D) basal plate (E) alar plate

D

Joints with limited or no movement are called _______, which freely mobile joints are called _______ (A) synarthroses, synostoses (B) synostoses, symphyses (C) symphyses, diarthroses (D) synarthroses, diarthroses

B

Key functions of ____ include: 1. Mechanical support 2. Control of cell proliferation 3. Scaffolding for tissue renewal 4. Establishment of tissue microenvironments (A) cytoskeleton (B) extracellular matrix (C) intracellular matrix (D) plasma membrane

erythropoietin (EPO)

Kidneys produce _________ , which stimulates erythropoiesis in bone marrow and the production of RBCs

B

Laminar flow is streamlined and silent. The velocity of flow is (A) equal throughout stream (B) zero at vessel wall and maximal at center of stream (C) maximal at vessel wall and zero at center of stream (D) none of the above

C

Large amounts of ____ are needed in liver for biosynthesis of lipoproteins & bile (A) G3P (B) DHAP (C) PC (D) PE

ACTH

Large doses of exogenous corticosteroids act on hypothalamus & anterior pituitary to suppress release of _______ → suppress production of corticosteroids

D

Live viral vaccine have numerous mutations that prevent them from being pathogenic. ________ are obtained by growing the virus in embryonated eggs or tissue culture cells at low temps to avoid selective pressures of host immune response. _________ don't replicate efficiently in human cells. (A) attenuated viruses, host-range mutants (B) low temperature mutants, host-less mutants (C) attenuated viruses, host-less mutants (D) low temperature mutants, host-range mutants

D

Liver cancer is associated with particularly high plasma concentrations of ________ (A) AST (B) prothrombin (C) heme (D) alpha fetoprotein (AFP)

C

Liver removes cholesterol either in a free form or as a ______ (A) sterol (B) conjugated salt (C) bile acid (D) steroid hormone

A

Low haptoglobin level is a sign of (A) intravascular hemolysis (B) osteoporosis (C) coagulation disease (D) hematopoiesis

D

Low levels of serum ______ may indicate iron deficiency (A) TBG (thyroxine-binding globulin) (B) albumin (C) transferrin (D) Ferritin (E) ceruloplasmin

C

Lysosomal storage diseases result from defects in _________ degradation (A) sphingomyelin (B) glycerophosphate (C) glycolipid (D) hydrolipid

C

Magnitude of muscle tension is (A) proportional to amount of cross-bridge cycling (B) proportional to intracellular [Na+] (C) proportional to intracellular [Ca2+] (D) inversely proportional to intracellular [K+]

1-B 2-A 3-C

Match the spina bifida type to the correct area 1. cervical region 2. cranial region 3. lumbar region A. anencephaly B. encephalocele C. myelomeningocele

A-2, B-1, C-3

Match the type of synarthrosis to its appropriate description A. Synostoses B. Syndesmoses C. Symphyses 1. join bones by DCT only 2. bones linked to other bones allowing no movement; unit skull bones in older adults instead of sutures 3. thick pad of fibrocartilage b/w thin articular cartilage covering ends of bones; occur in midline of body

1-B, 2- C, 3-F, 4-E, 5-A, 6-D

Match the vaccines to the age at which they are given 1. 1st vaccine 2. 2 months old 3. 12 months old 4. yearly starting at 18 months 5. yearly after 2 years old 6. age 11 to 12 years old A. live attenuated influenza virus B. HBV vaccine C. vaccines for diphtheria, tetanus, acellular pertussis (DTaP); rotavirus, H influenzae B; pneumococcal conjugate vaccine; & inactivated poliovirus D. HPV vaccine & a different formulation of DTap (Tdap) E. inactivated influenza virus F. hepatitis A, MMR, varicella

1-A, 2-B

Match the vasopressin (ADH) receptor to its description 1. V1 2. V2 A. vascular smooth muscle - activation causes vasoconstriction of arterioles & increased TPR B. principal cells of renal collecting ducts - water reabsorption in collecting ducts & maintenance of body fluid osmolarity

A

Mesenchyme between each cerebral hemisphere condenses to form the (A) falx cerebri (B) tentorium cerebelli (C) 4th ventricle (D) pontine nuclei

B

Mesenchyme → (A) forebrain, midbrain, hindbrain (B) pia mater, arachnoid mater, dura mater (C) spinal cord and brain (D) organs

carbohydrate and lipid

Metabolism of AAs interfaces with _______ & _____ metabolism

D

Monoclonal immunoglobulin synthesis is a result of benign or malignant transformation of ______ (A) cytotoxic T cells (B) CD 8+ cells (C) helper T cells (D) B cells (E) Macrophages

B

Most lymphocytes (mainly Th cells) enter at the _______ of the LN via HEVs (high endothelial venules) located there only (A) cortex (B) paracortex (C) medulla (D) hilum

A

Most plasma proteins are synthesized in the ________ (A) liver (B) spleen (C) thymus (D) kidney

B

Most potent estrogen. Most abundant in premenopausal women (A) estrone (B) estradiol (C) estriol

transcription factors (TFs)

Most signal transduction pathways ultimately influence cell function by modulating gene transcription through activation & nuclear localization of ______ ________

D

Movement of endocytosed vesicles between apical & basolateral compartments of cells (A) Caveolae-mediated endocytosis (B) Pinocytosis (C) Receptor-mediated Endocytosis (D) Transcytosis

D

Myelination in the CNS is formed and maintained by which neuroglial cell? (A) Schwann cells (B) Astrocytes (C) Ependymal cells (D) Oligodendrocytes

A

Necrosis of myocardium b/c of local ischemia resulting from vasospasm or obstruction of the blood supply, usually by a thrombus or embolus in the coronary arteries (A) MI (B) Cardiac tamponade (C) Pericarditis (D) Pericardial Effusion (E) Angina Pectoris

high, ultrasound

Neural tube defects can be detected prenatally by screening for (low/high) alpha-fetoprotein (AFP) levels in the amniotic fluid or in maternal serum and sequentially confirmed through _________

B

Neuroblasts also form an external layer known as the _______, which becomes myelinated to form the white matter of the spinal cord (A) central canal (B) marginal zone (C) intermediate zone (D) mesenchyme

D

Neuroblasts in basal plates of ____ form neurons that form the nuclei of CNs III, IV, & possible red nuclei, substantia nigra, & reticular formation (A) cerebellum (B) pons (C) medulla oblongata (D) midbrain (mesencephalon)

C

Neuroblasts in the neural tube migrate peripherally, forming the intermediate zone, which forms the ______ (A) gray matter of brain (B) white matter of brain (C) gray matter of spinal cord (D) white matter of spinal cord

E

OCPs usually use a combination of estrogen & progestin. Combination OCPs prevent ovulation to prevent conception by (A) suppressing release of FSH (B) thickening the endometrium (C) suppressing the surge of LH from the anterior pituitary (D) A & B (E) A & C

A

-AKA gangliosidosis -Deficient enzyme = lysosomal hexosamindase A -Accumulation = ganglioside GM2 -Mental retardation & blindness (A) Tay-Sachs Disease (B) Fabry's Disease (C) Gaucher's Disease (D) Hunter's Disease

C

-Follows ARP -Possible to generate another action potential if greater than normal stimulus (A) Absolute Refractory Period (ARP) (B) Effective Refractory Period (ERP) (C) Relative Refractory Period (RRP) (D) Supranormal Period (SNP)

liver function tests

-I can tell you if a specific organ has a self named disease -I can tell you if bilirubin is being removed correctly -I can give you a sign of hemolytic anemia -I can tell you about your total protein

E

-bound to surface of mast cells and basophils -destroys parasitic worms and participates in allergies (A) IgG (B) IgM (C) IgA (D) IgD (E) IgE

parts of APGAR

-checks appearance -checks respiration -test done more than once -checks pulse

A

-defect in the vertebral arches -least severe type of spina bifida -occurs in 10% of the population (A) spina bifida occulta (B) meningocele (C) meningomyelocele (D) myelocele (E) Syringomyelocele

risk factors for a neural tube defect

-folate deficiency -family history -anti-seizure medications (carbamazepine) -obesity -gestational diabetes

neonatal conjunctivitis

-found in new borns, swelling of eyelids, redness of eyes, pus discharge -half of patients have infection in other part of body as well -silver nitrate used to prevent this `

C

-secretions (saliva, milk, tears, etc.) -protects mucosae (A) IgG (B) IgM (C) IgA (D) IgD (E) IgE

D

Only H substance on RBC membrane (A) A blood (B) B blood (C) AB blood (D) O blood

E

A decrease is plasma haptoglobin is a marker of (A) iron-deficiency anemia (B) macrocytic anemia (C) liver disease (D) renal failure (E) hemolysis

A

A smear of blood from a 70-year-old leukemia patient reveals a larger than normal population of cells that have large, round nuclei with one or two nucleoli. The cytoplasm of these cells shows azurophilic granules. Which of the following forms of leukemia would you suspect? (A) Promyelocytic leukemia (B) Basophilic leukemia (C) Lymphoblastic leukemia (D) Stem cell leukemia (E) Eosinophilic leukemia

4 pregnancies 1 full-term pregnancy 0 premature 2 spontaneous abortions 2 living children (twins)

A woman is not currently pregnant and is G4, 1022, which means?

sensory

Alar plate neuroblasts are (motor/sensory)

A

All formed elements of blood enter circulation by crossing the _________ _______ of sinusoids in red marrow (A) discontinuous endothelium (B) stratified squamous epithelium (C) tunica intima (D) tunica adventitia

glutamate, aspartate

Amino groups transferred to α-ketoglutarate → form _________ Amino groups transferred to oxaloacetate → form _____

B

Aortic valve narrowed → decreased stroke volume → less blood enters aorta on each beat → decrease in systolic, pulse, & mean pressure (A) Arteriosclerosis (B) Aortic stenosis (C) Aortic regurgitation (D) Pulmonary HTN

A

Apex of the heart is formed by the______ (A) Left ventricle (B) Left atrium (C) Right ventricle (D) Right atrium

prostaglandin

Aspirin & other NSAIDS block _______ synthesis → inhibitory effect on platelet function & blood coagulation

chronotropic, dromotropic

Autonomic effects on HR are called (chronotropic/dromotropic) effects. Autonomic effects on conduction velocity in AV node are called (chronotropic/dromotropic) effects.

C

B cells produce antibodies for _______ immunity (A) adaptive (B) innate (C) humoral (D) cell-mediated

D

Binding of an antigen-binding site with antigen causes which of the following? (A) Neutralization (B) Agglutination (C) Precipitation (D) all of the above

B

Blood concentration of ________secreted by anterior pituitary gland regulates production of ALL corticosteroids through _________ (A) Aldosterone, positive feedback mechanism (B) ACTH, negative feedback mechanism (C) cortisol, negative feedback mechanism (D) GnRH, positive feedback mechanism

D

CTP + phosphoethanolamine → CDP-ethanolamine → phosphocholine transferred → _______ formed (A) G3P (B) DHAP (C) PC (D) PE

21-hydroxylase

CYP21A2 gene provides instructions for making the enzyme _________, which is part of the cytochrome P450 family of enzymes. Deficiency of this enzyme → CAH (CYP21A1 is the pseudogene CYP21A2 is the gene for the active enzyme)

C

Ca2+ channel blockers, such as nifedipine, diltiazem, & verapamil, work by inhibiting ________ of ______ (A) inward Ca2+ current of voltage-gated channels (B) outward Ca2+ current of L-type channels (C) inward Ca2+ current of L-type channels (D) outward Ca2+ current of voltage-gated channels

B

Ca2+ deficiency in adults → _________ , which is deficient calcification of recently formed bone & partial decalcification of already calcified matrix (A) rickets (B) osteomalacia (C) osteogenesis imperfecta (D) pituitary dwarfism

A

Ca2+ deficiency in kids → ______, when bone matrix doesn't calcify normally & epiphyseal plate can become distorted by normal strains of body weight & muscular activity → bone grows slowly & becomes deformed b/c ossification impeded (A) rickets (B) osteomalacia (C) osteogenesis imperfecta (D) pituitary dwarfism

D

Cardiac glycosides are positive ionotropic agents that act by _____ so that intracellular [Ca2+] increases and there is an increase in tension (A) inhibiting H+ pump (B) stimulating outflow of K+ (C) inhibiting outflow of Na+ (D) inhibiting Na+-K+ pump

B

Cardiac output is a function of (A) end-systolic volume (B) end-diastolic volume (C) aortic pressure (D) heart rate

D

Cardiac output of the right side of the heart is what percentage of the cardiac output of the left side of the heart? (A) 25% (B) 50% (C) 75% (D) 100% (E) 125%

A

Cardiopulmonary baroreceptors are located in veins, atria, & pulmonary arteries and sense (A) changes in blood volume (B) decreases in hormone balance (C) decreases in PO2 (D) changes in Pa (mean arterial pressure)

B

Cellular drinking (A) Caveolae-mediated endocytosis (B) Pinocytosis (C) Receptor-mediated Endocytosis (D) Transcytosis

T4, T3

DIT+DIT = (T3/T4) MIT+DIT = (T3/T4)

B

Degrade macromolecules & phagocytosed microbes & damaged/unwanted cell organelles (A) Proteasomes (B) Lysosomes (C) Peroxisomes (D) Golgi Body

laminar, turbulent

Reynolds number is used to predict whether blood flow will be laminar or turbulent. 1. NR < 2,000 → 2. NR > 3,000 →

C

SOB that occurs when lying flat (A) tachypnea (B) dyspnea (C) orthopnea (D) apnea

B

Sickle cell, thalassemia, & G6PD deficiency are examples of _______ anemia caused by immune attack, extrinsic factors, disorders of RBC membrane, enzymopathies, & hemoglobinopathies A. Folate deficiency anemia B. Hemolytic anemia C. Aplastic anemia D. Iron-deficiency anemia

B

Slow conduction of ______ leads to ventricles having enough time to fill with blood before contracting (A) Bundle of His, Purkinje system, & ventricles (B) AV node (C) SA node (pacemaker) (D) Atrial internodal tracts & atria

A

Small mass of specialized cardiac muscle fibers in myocardium at upper end of crista terminalis near opening of SVC in R atrium (A) SA node (B) AV node (C) Bundle of His (D) Purkinje fibers

dyads

Smooth muscle differs from skeletal muscle in that it forms _____ with SR, which is the site of storage and release of Ca2+ for excitation-contraction coupling, whereas skeletal muscle forms triads

C

Steroid hormone & major glucocorticoid synthesized & secreted by the human adrenal cortex as required (A) ACTH (B) CRH (C) Cortisol (D) GnRH (E) TRH

false

T/F Humans can metabolize the sterol structure

increase (parasympathetic); decrease (sympathetic)

The PR segment correlates with conduction time through AV node. -decrease in conduction velocity → (decrease/increase) in PR interval -increase in conduction velocity → (decrease/increase) in PR interval

B

The Valsalva Maneuver tests the integrity of the baroreceptor mechanism by having a patient expire against a closed glottis, which causes 1. _____ in intrathoracic P 2. _____ in venous return 3. _____ in cardiac output & Pa (A) ↑↑↑ (B) ↑↓↓ (C) ↓↑↑ (D) ↓↓↓

C

The _____ is located at the junction between the superior vena cava and the right atrium. (A) Atrioventricular (AV) node (B) Purkinje fibers (C) Sinoatrial (SA) node (D) Atrioventricular (AV) bundle (of His)

A

The _____ is responsible for regulation of body temperature core temp ___ set-point temp → heat-generating & heat-retaining mechanisms activated core temp ___ set-point temp → heat-dissipating mechanism activated (A) anterior hypothalamus, <, > (B) posterior hypothalamus, >, < (C) anterior pituitary, >, < (D) sympathetic nervous system, <, >

left ventricle, right ventricle

The ______ ______ pumps blood to the body The ______ ______ pumps blood to the lungs

D

The ______ _______ is an important pericardial sinus to cardiac surgeons because while performing surgery on the aorta or pulmonary artery, a surgeon could stop blood circulation with a ligature (A) Oblique sinus (B) Pericardial cavity (C) Pleural space (D) Transverse Sinus (E) Sagittal Sinus

C

The aortic valve closes at point (A) 1 (B) 2 (C) 3 (D) 4

C

The base of the heart DOES NOT include___________ (A) Proximal parts of the great veins (B) Left atrium (C) Left ventricle (D) A small portion of the right atrium

B

The following are disadvantages of what type of vaccine? -Induced immunity is usually just humoral & NOT long-lasting -Local IgA response NOT elicited -Booster shots required (A) live attenuated vaccines (B) inactivated vaccines (C) subunit vaccines (D) all of the above

D

The following statement concerns the development of the cerebral hemispheres: (A) The corpus striatum is formed from the proliferation of the matrix cells lining the roof of the forebrain vesicle. (B) The interventricular foramen is formed by the cavity of the diencephalon. (C) The choroid plexus of the lateral ventricle is formed by vascular ectoderm covered by ependymal cells. (D) The internal capsule is formed by the developing ascending and descending tracts growing between the developing thalamus and caudate nucleus medially and the lentiform nucleus laterally (E) The cortical neurons develop in situ and do not migrate out laterally from the matrix cells lining the cavity of the cerebral hemisphere.

D

The following statement concerns the fate of the forebrain vesicle: (A) The optic vesicle grows out of the midbrain vesicle. (B) The thalamus is formed from the alar plates in the medial walls of the diencephalon. (C) The lamina terminalis is formed from the rostral end of the diencephalon. (D) The pars nervosa of the hypophysis is formed from the floor of the diencephalon. (E) The hypothalamic nuclei are formed from the basal plates of the diencephalon

C

The great cardiac vein is a tributary of the__________ (A) Azygos vein (B) Internal jugular vein (C) Coronary sinus (D) Superior vena cava (E) Inferior vena cava

B

The inactivated influenza vaccine developed yearly to contain __ influenza A & __ influenza B virus strains expected to be most prevalent in flu season (A) 3, 0 (B) 2, 1 (C) 2, 2 (D) 1, 2

B

The interstitial matrix is one form of ECM, which is synthesized by ________, forming a amorphous 3-D gel (A) pluripotent stem cells (B) mesenchymal cells (fibroblasts) (C) overlying epithelium & underlying mesenchymal cells (D) laminin

A

The largest vein draining the heart is the _______, which opens into the right atrium (A) coronary sinus (B) great cardiac vein (C) middle cardiac vein (D) small cardiac vein (E) Anterior cardiac vein

B

The neuroblasts of the basal plate of the intermediate zone become the ______, which those of the alar plate become _________ (A) sensory cells in posterior horn; motor cells of anterior horn (B) motor cells in anterior horn; sensory cells in posterior horn (C) motor cells in posterior horn; motor cells in anterior horn (D) sensory cells in anterior horn; sensory cells in posterior horn

decreasing, increasing, prolongs

The parasympathetic decreases conduction velocity by: -(increasing/decreasing) inward Ca2+ current -(increasing/decreasing) outward K+ current -(shortens/prolongs) ERP

C

The pericardium is innervated by the vasomotor & sensory fibers from the _______ and ________ nerves & the sympathetic trunks (A) phrenic & glossopharyngeal (B) vagus & glossopharyngeal (C) phrenic & vagus (D) glossopharyngeal & axillary

C, E, F, H, I

The pericardium receives blood from which of the following arteries (5) (A) brachiocephalic (B) left anterior descending (C) internal thoracic (D) external thoracic (E) pericardiacophrenic (F) musculocophrenic (G) superior phrenic (H) inferior phrenic (I) thoracic aorta

C

The physiologic function of the relatively slow conduction through the atrioventricular (AV) node is to allow sufficient time for (A) runoff of blood from the aorta to the arteries (B) venous return to the atria (C) filling of the ventricles (D) contraction of the ventricles (E) repolarization of the ventricles

B

The positive staircase effect (aka Bowditch staircase) refers to the stepwise rise in ______, since with each beat more Ca2+ is accumulated by SR until a max storage level is reached (A) HR (B) tension (C) conduction velocity (D) ACh

C

The thoracic duct begins in the abdomen at cisternal chyli, drains the lower limbs, pelvis, abdomen, L thorax, L upper limb, & L side of head & neck, and empties into the junction of the ___________ & __________ veins after passing through the diaphragm at TIV/TV (A) right internal jugular & brachiocephalic veins (B) right internal jugular & subclavian veins (C) left internal jugular & subclavian veins (D) left internal jugular & brachiocephalic veins (E) SVC & subclavian veins

F

The trachea is a midline structure palpable in the notch as it enters the ________ mediastinum. It divides into the R & L main bronchi b/w ______ and _____ (A) superior, sternal angle & T5/T6 (B) middle, sternal angle & T5/T6 (C) middle, sternal angle & T3/4 (D) superior, sternal angle & T3/T4 (E) middle, sternal angle & T4/T5 (F) superior, sternal angle & T4/T5

B

Thyroid hormones _____ metabolic rate, with ____ O2 consumption and heat _____ (A) decrease, decreased, loss (B) increase, increased, production (C) increase, decreased, production (D) increase, increased, loss

B

Thyroid hormones exert their affects via nuclear receptors. Thyroid receptors bind to thyroid-response elements → alteration of _______ of specific thyroid-response genes & protein synthesis (A) translation (B) transcription (C) replication (D) synthesis

C

Thyroid hormones stimulate ______ → increased O2 consumption → increased metabolic rate → increased heat production (A) anterior hypothalamus (B) posterior hypothalamus (C) Na+-K+ ATPase (D) alpha 1 receptors in vascular smooth muscle of skin blood vessels (E) beta receptors in brown fat

C

Tricuspid valve is located at the orifice between the___________ (A) Left ventricle and left atrium (B) Left atrium and right atrium (C) Right ventricle and right atrium (D) Left ventricle and right ventricle

A

Usually closes functionally at birth. Shunts blood from R atrium to L atrium, partially bypassing fetal lungs (A) foramen ovale (B) Ductus arteriosus (C) ductus venosus

B

What AA substitution at the 6th residue of the β chain of Hb leads to HbS in sickle cell anemia? A. Glu → Leu B. Glu → Val C. Val → Glu D. Glu → Ala

cytoskeleton

What gives cells the ability to do the following? 1. adopt a shape 2. maintain polarity 3. organize intracellular organelles 4. move along intracellular scaffolding of proteins

feedback

What type of mechanism normally controls hormone systems?

D

What type of stem cells are hemopoietic stem cells? (A) totipotent (B) multipotent (C) pluripotent (D) none of the above

innate

Which is older, innate or adaptive immunity?

quicker, less

Woven bone forms (slower/quicker/at same rate) than lamellar bone and has (less/more/equal) strength

B

_____ alters rate at which action potentials are conducted from atria to ventricles (A) SA node (B) AV node (C) Purkinje Fibers (D) Bundle of His

osteoblasts

________ entrapped in lacunae become osteocytes

D

________ structures on glycosphingolipids of red cell membranes are also antigenic determinants responsible for the ABO & other blood types (A) Protein (B) shingolipid (C) polar head group (D) carbohydrate

G

activated neurons secrete neurotransmitters at synapses (A) pathogens & damage to neighboring cells (B) cell-cell contacts (C) cell-ECM contacts (D) secreted molecules (E) paracrine signaling (F) autocrine signaling (G) synaptic signaling (H) endocrine signaling

B

adhesion molecules and/or gap junctions (A) pathogens & damage to neighboring cells (B) cell-cell contacts (C) cell-ECM contacts (D) secreted molecules (E) paracrine signaling (F) autocrine signaling (G) synaptic signaling (H) endocrine signaling

glucogenic

alanine, arginine, asparagine, aspartic acid, cysteine, cystine, glutamine, glutamate, glycine, histidine, methionine, proline, serine, valine are ______ AAs which increase concentrations of glucose when ingested

A

alar plate sensory neuroblasts give rise to the _______ which are made up of the gracile & cuneate nuclei (A) posterior column nuclei (B) inferior olivary nuclei (C) solitary nucleus (D) spinal trigeminal nucleus

B

alar plate sensory neuroblasts give rise to the _______ which form the cerebellar relay nuclei (A) posterior column nuclei (B) inferior olivary nuclei (C) solitary nucleus (D) spinal trigeminal nucleus

motor

basal plate neuroblasts are (motor/sensory)

C

begins w/ formation of bone collar on cartilaginous diaphysis → increasing bone circumference w/ enlargement of central marrow cavity by activity of osteoclasts in endosteum (A) endochondral ossification (B) bone matrix (C) appositional growth (D) intramembranous ossification

B

bind specific solute → series of conformational changes → transfer ligand across membrane (slow) (A) Channel proteins (B) Carrier proteins (C) Passive transport (D) Active transport

sharpey's fibers

bundles of periosteal collagen fiber penetrate bone matrix binding periosteum to bone

E

catalyzes cleavage of arginine to ornithine & urea (A) carbamoyl phosphate synthetase I (CPS I) (B) Ornithine transcarbamoylase (C) Argininosuccinate synthetase (D) Argininosuccinase (E) Arginase

D

catalyzes cleavage of argininosuccinate to arginine & fumarate (A) carbamoyl phosphate synthetase I (CPS I) (B) Ornithine transcarbamoylase (C) Argininosuccinate synthetase (D) Argininosuccinase (E) Arginase

C

catalyzes formation of argininosuccinate from citrulline & aspartate (A) carbamoyl phosphate synthetase I (CPS I) (B) Ornithine transcarbamoylase (C) Argininosuccinate synthetase (D) Argininosuccinase (E) Arginase

A

catalyzes formation of carbamoyl phosphate from ammonia & CO2 (A) carbamoyl phosphate synthetase I (CPS I) (B) Ornithine transcarbamoylase (C) Argininosuccinate synthetase (D) Argininosuccinase (E) Arginase

B

catalyzes formation of citrulline from ornithine & carbamoyl phosphate (A) carbamoyl phosphate synthetase I (CPS I) (B) Ornithine transcarbamoylase (C) Argininosuccinate synthetase (D) Argininosuccinase (E) Arginase

D

caused by obstruction of the biliary tree → pale stools & dark urine (conjugated bilirubin) (A) Prehepatic jaundice (B) Prehepatic hyperbilirubinemia (C) Intrahepatic jaundice (D) Posthepatic jaundice

E

cells in immediate vicinity affected (A) pathogens & damage to neighboring cells (B) cell-cell contacts (C) cell-ECM contacts (D) secreted molecules (E) paracrine signaling (F) autocrine signaling (G) synaptic signaling (H) endocrine signaling

C

clinical term for biliary obstruction (A) Acute hepatitis (B) Chronic hepatitis (C) Cholestatic disease (D) Hemochromatosis

C

concentration and/or electrical gradient drives movement (A) Channel proteins (B) Carrier proteins (C) Passive transport (D) Active transport

B

defect in conversion of coproporphyrinogen III to protoporphyrinogen III (coprooxidase) (A) Acute intermittent porphyria (AIC) (B) Hereditary coroporphyria (C) Variegate porphyria (D) Other porphyrias

A

deficiency of hydrocymethylbilane synthase (converts OBG to linear tetrapyrrole) → increased 5-ALA and PBG in plasma & urine (A) Acute intermittent porphyria (AIC) (B) Hereditary coroporphyria (C) Variegate porphyria (D) Other porphyrias

B

diphtheria & tetanus vaccines are examples of (A) killed bacteria vaccines (B) toxoid containing vaccines (C) vaccines containing protein or polysaccharide subunits of the bacteria (D) live attenuated vaccines

A

end of diastole → ventricle contracts → ventricular P increases but ventricular volume constant b/c all valves are closed (A) isovolumetric contraction (B) ventricular ejection (C) isovolumetric relaxation (D) ventricular filling

A

excitatory neurotransmitter; precursor of GABA (inhibitory NT) (A) Glutamate (B) Histidine (C) Tryptophan (D) Tyrosine

A

failure of anterior neurotube closure causes _______ (A) anencephaly (B) spina bifida (C) craniorachischisis

B

failure of posterior neurotube closure causes __________ (A) anencephaly (B) spina bifida (C) craniorachischisis

B

galactose α1, 3 + H substance = (A) A blood (B) B blood (C) AB blood (D) O blood

D

growth factors, cytokines, & hormones (A) pathogens & damage to neighboring cells (B) cell-cell contacts (C) cell-ECM contacts (D) secreted molecules (E) paracrine signaling (F) autocrine signaling (G) synaptic signaling (H) endocrine signaling

B

hemoglobin is synthesized in erythroblasts * reticulocytes under tight control by concentration of (A) RBCs (B) heme (C) O2 (D) CO2

H

hormone released into blood stream & acts on target cells at a distance (A) pathogens & damage to neighboring cells (B) cell-cell contacts (C) cell-ECM contacts (D) secreted molecules (E) paracrine signaling (F) autocrine signaling (G) synaptic signaling (H) endocrine signaling

hypo, hyper

hormone stimulation tests are used to identify (hypo-/hyper-) secretion of hormones hormone suppression tests are used to identify (hypo-/hyper-) secretion of hormones

gap junctions

how are osteocytes connected?

B

immunoglobulins produced by plasma cells after a progenitor B cell is activated by a specific antigen & rearranges its Ig genes so antibody matches antigen (A) antigens (B) antibodies (C) cytokines (D) interleukins

TORCH

infections that cause teratogenic insult

cement line

layer of bone matrix with few fibers & more Ca2+ phosphate surrounding each osteon

ketogenic

leucine & lysine are ______ AAs which increase concentrations of ketone bodies when ingested

C

lipid-soluble ligands diffuse into cells & interact with proteins → receptor-ligand complex directly binds to nuclear DNA → activation or repression of gene transcription (A) Receptor Tyrosine Kinases (RTKs) (B) G-protein coupled receptors (C) Nuclear receptors (D) Nonreceptor tyrosine kinase

howship's canal

location of osteoclasts

B

maternal urinary level of _______ is a reliable index of fetal placental function (viability) because its production is dependent on a normally functioning fetal adrenal cortex, fetal liver, and placenta (A) AFP (B) estriol unconjugated (C) progesterone (D) hCG (E) oxytocin (F) hPL

hCG (human chorionic gonadotropin)

measurement of ____ in blood is used to confirm pregnancy and early serum levels can help assess viability of pregnancy

hyper, hypo

metabolic rate increases in (hypo-/hyper-)thyroidism metabolic rate decreases in (hypo-/hyper-)thyroidism

surgical, infection

pain before vomiting → (infection/surgical) problem in abdomen vomiting before pain → (infection/surgical) problem in abdomen

amniocentesis

percutaneous transabdominal puncture of uterus to obtain amniotic fluid, performed at 15-17 weeks after pregnant woman's last menstrual period, amniotic fluid withdrawn, fluid containing living cells (aminocytes) shed by fetus

Osteoclasts

phagocytic bone absorptive cell Origin: Formed from fusion of blood monocytes --> multinucleated Function: localized digestion of bone matrix

joints

places where bones meet (articulate) → allow potential for bending or movement in that portion of skeleton

D

plasma estriol, ultrasound, auscultation for the heartbeat are three ways to determine (A) fetal alcohol syndrome (B) neural tube defects (C) down syndrome (D) fetus viability

D

protein energy malnutrition will lead to decreased plasma colloid osmotic pressure which will cause: (A) transudate (B) hypotension (C) hypertension (D) edema (E) MI

D

short-term changes in albumin levels more likely reflect changes in (A) infection status (B) cardiac function (C) nutritional status (D) hydration

B

the oxidative removal of the amino group, resulting in keto acids, a reduced flavin coenzyme, and ammonia (A) Transamination (B) Oxidative deamination (C) Removal of a molecule of water by a dehydratase (D) Reductive deamination

AB, O

type ___ cells are universal recipients and type ___ cells are universal donors

D

uses carrier molecules & ATP hydrolysis or couple ion gradient (A) Channel proteins (B) Carrier proteins (C) Passive transport (D) Active transport

1. esophageal atresia and/or duodenal atresia of the fetus 2. Possibly anencephaly

what causes polyhydramnios (2)?

C

what is present in azurophilic granules? (A) oxidase (B) perforins (C) peroxidase (D) granzymes

heart rate = 1 / cycle length

what is the relationship of heart rate to cycle length? (cycle length = R-R interval)

neonatal (after birth) screening tests

-PKU -Sickle cell -Congenital hypothyroidism

D

-Ventricles eject blood into arteries (slower rate) -Ventricular V reaches minimum -Aortic P starts to fall as blood runs off into arteries (A) Atrial Systole (B) Isovolumetric Ventricular Contraction (C) Rapid Ventricular Ejection (D) Reduced Ventricular Ejection (E) Isovolumetric Ventricular Relaxation (F) Rapid Ventricular Filling (G) Reduced Ventricular Filling or Diastasis

G

-Ventricles relaxed -Final phase of ventricular filling (A) Atrial Systole (B) Isovolumetric Ventricular Contraction (C) Rapid Ventricular Ejection (D) Reduced Ventricular Ejection (E) Isovolumetric Ventricular Relaxation (F) Rapid Ventricular Filling (G) Reduced Ventricular Filling or Diastasis

F

-Ventricles relaxed -Ventricles fill passively w/ blood from atria -Ventricular V increases -Ventricular P is low and constant -Mitral valve opens (A) Atrial Systole (B) Isovolumetric Ventricular Contraction (C) Rapid Ventricular Ejection (D) Reduced Ventricular Ejection (E) Isovolumetric Ventricular Relaxation (F) Rapid Ventricular Filling (G) Reduced Ventricular Filling or Diastasis

E

A 12-year-old boy was admitted to a local hospital with a known history of heart problems. His left ventricular hypertrophy could result from which of the following conditions? (A) A constricted pulmonary trunk (B) An abnormally small left AV opening (C) Improper closing of the pulmonary valves (D) An abnormally large right AV opening (E) Stenosis of the aorta

E

A 12-year-old girl of African descent presents with anemia and a large percentage of her peripheral erythrocytes appear sickle-shaped. Genetic testing reveals homozygosity for sickle cell disease. In which of the following sites will the abnormal RBCs be removed from the circulation? (A) Thymic cortex (B) Periarteriolar lymphoid sheathes of splenic white pulp (C) Medullary sinuses of lymph nodes (D) Thymic medulla (E) Splenic cords (of Billroth)

D

A 15-year-old male presents with hematuria, hearing loss, lens dislocation, and the onset of cataracts. Genetic analysis reveals a mutation in the COL4A5 gene. Transmission EM examination of a renal biopsy confirms that the disorder has affected a component of the renal corpuscles in which damage disrupts normal glomerular filtration. Which one of the following structures would most likely be abnormal in the TEM of this patient's biopsy? (A) Pedicels (B) Filtration slits (C) Slit diaphragms (D) Glomerular basement membranes (E) Fenestrated endothelium of glomerular capillaries

B

A 19-year-old man came to the emergency department, and his angiogram exhibited that he was bleeding from the vein that is accompanied by the posterior interventricular artery. Which of the following veins is most likely to be ruptured? (A) Great cardiac vein (B) Middle cardiac vein (C) Anterior cardiac vein (D) Small cardiac vein (E) Oblique veins of the left atrium

B

A 62-year-old woman who is a heavy smoker has an advanced lung cancer that spread into her right third posterior intercostal space posterior to the midaxillary line. If cancer cells are carried in the venous drainage, they would travel first to which of the following veins? (A) SVC (B) Right superior intercostal vein (C) Right brachiocephalic vein (D) Azygos vein (E) Hemiazygos vein

A

A _________ is a method routinely used in molecular biology for detection of a specific DNA sequence in DNA samples (A) southern blot (B) western blot (C) eastern blot (D) northern blot

D

A better index of hepatocyte synthetic function is the production of ________ (A) albumin (B) ALT & AST (C) globulin proteins (D) coagulation factors 2, 7, 9, and 10

B

A better index of hepatocyte synthetic function is the production of coagulation factors 2, 7, 9, and 10, the functional concentrations of which are measured using (A) activated partial thromboplastin time (aPTT) (B) prothrombin time (PT) (C) liver function test (D) CBC

E

A differential cell count of a blood smear from a patient with a parasitic infection is likely to reveal an increase in the circulating numbers of which cell type? (A) Neutrophils (B) Lymphocytes (C) Monocytes (D) Basophils (E) Eosinophils

seizure

A disordered, synchronous, & rhythmic firing of population of neurons leading to transient alteration in behavior

C

A radiologist examines posterior-anterior chest radiographs of a 27-year-old victim of a car accident. Which of the following structures forms the right border of the cardiovascular silhouette? (A) Arch of the aorta (B) Pulmonary trunk (C) SVC (D) Ascending aorta (E) Left ventricle

C

AAs destined for energy metabolism must be ______ to yield the carbon skeleton (A) esterified (B) condensed (C) deaminated (D) oxidized

appearance, pulse, grimace, activity, respiration

APGAR measures:

increase, increase

Aldosterone and increased Na+-H+ exchange both work to (decrease/increase) Na+reabsorption by renal distal tubule → (decrease/increase) ECF V, blood V, & Pa

C

Ammonia is detoxified by incorporation into ______, then eventually into urea (A) alanine (B) aspartate (C) glutamine (D) glycine

aneuploidy

Amniocentesis can provide an indication of fetal __________ in 1-2 days

D

An 8-year-old boy with ASD presents to a pediatrician. This congenital heart defect shunts blood from the left atrium to the right atrium and causes hypertrophy of the right atrium, right ventricle, and pulmonary trunk. Which of the following veins opens into the hypertrophied atrium? (A) Middle cardiac vein (B) Small cardiac vein (C) Oblique cardiac vein (D) Anterior cardiac vein (E) Right pulmonary vein

A

An immunohistochemical technique using antibodies against aquaporins to stain a section of kidney would be expected to stain cells in which structures most intensely? (A) Collecting ducts (B) Lining of the major and minor calyces (C) Proximal convoluted tubules (D) Distal convoluted tubules (E) Glomeruli

decreased, decreased, decreased

An increase in HR leads to (decreased/increased) time available for ventricular refilling, (decreased/increased) end-diastolic volume, & (decreased/increased) stroke volume

C

An increase in arteriolar resistance, without a change in any other component of the cardiovascular system, will produce (A) a decrease in total peripheral resistance (TPR) (B) an increase in capillary filtration (C) an increase in arterial pressure (D) a decrease in afterload

stressed, unstressed

An increase in venous compliance means blood is shifted from (stressed/unstressed) of the arteries to (stressed/unstressed) of the veins

convulsants, anticonvulsants

Antagonists of GABA-A → (anticonvulsants/convulsants) Facilitators of GABAergic mechanisms → (anticonvulsants/convulsants)

A

Anterior pituitary synthesizes ______ → stimulates liver to produce IGF-1 → growth-promoting effect, esp. on epiphyseal cartilage (A) GH (somatrotropin) (B) PTH (C) Calcitonin (D) Ca2+

C

Aortic valve incompetent → blood ejected into aorta flows backward into ventricle (A) Arteriosclerosis (B) Aortic stenosis (C) Aortic regurgitation (D) Pulmonary HTN

increases, increase, increases

As venous return increases, R atrial pressure (decreases/increases), and end-diastolic V & end-diastolic fiber length (decrease/increase) → cardiac output (decreases/increases)

C

Aspirin works to reduce fever by (A) inhibiting production of IL-1 (B) increasing production of pyrogens (C) inhibiting COX enzyme needed for synthesis of prostaglandins (D) inhibiting ability of anterior hypothalamus to activate heat generating mechanisms

E

Associate with heparan sulfate in ECM → contribute to wound healing responses, hematopoiesis, & development (A) EGF & TGF-α (B) HGF (C) PDGF (D) VEGFs (E) FGF (F) TGF-β

C

At birth, the distal end of the spinal cord, the conus medullaris is at the level of the______ vertebral body (A) T12 (B) T11 (C) L3 (D) T10 (E) L1

E

At which site is systolic blood pressure the highest? (A) Aorta (B) Central vein (C) Pulmonary artery (D) Right atrium (E) Renal artery (F) Renal vein

D

Azurophilic granules stain with _______ dyes and are _____ among all 3 types of granulocytes (A) acidic; different (B) acidic; similar (C) basic; different (D) basic; similar

C

B-cell receptors (BCRs) are ____ or _____ antibodies on cell surface that bind specific antigens whenever they contact them (A) IgM or IgE (B) IgD or IgA (C) IgM or IgD (D) IgA or IgE

E

Baby boy (girl) Colman was diagnosed with CAH. Which of the following could be complications resulting from excessive testosterone? (A) Premature closure of epiphysis (B) Early menarche (C) Infertility (D) Short stature (E) all of the above

C

Baby boy Murphy did not have an anal wink reflex elicited, corresponding to a motor deficit in ________ (A) L2-L4 (B) S1-S2 (C) S2-S4 (D) L1-L3 (E) S1-S3

C

Bile acids are key elements in ____ metabolism (A) protein (B) carbohydrate (C) fat (D) nucleic acid

C

Binds bile acids → increase in 7alpha-hydroxylase activity → increased bile acid synthesis → increased bile acid excretion → cellular cholesterol synthesis & expression of LDL receptors increase → lowers plasma cholesterol (A) bile salts (B) statins (C) cholestyramine (D) cytochrome P450 monoxygenases

hypothalamus

Both anterior & posterior pituitary are under influence of the ________

C

Broad-spectrum anticonvulsant with the follow MoA: -Frequency-dependent block of voltage-gated Na+ channels -Activate hyperpolarizing K+ current -Potentiation of postsynaptic GABA-A receptors -Inhibition of AMPA/kainite subtype of glutamate receptors -Weak inhibition of carbonic anyhdrase enzyme (A) Gabapentin (B) Lamotrigine (C) Topiramate (D) Zonisamide

4 pregnancies 1 full term pregnancy 0 premature 2 spontaneous abortions 1 living children

Caroline Weisen was G4, 1021, which means?

junctions

Cells interact & communicate with each other by forming _______ → provide mechanical links & enable surface receptors to recognize ligands on other cells

lymphoid nodules

Cells produced in ______ ______ are dispersed as plasma cells, various T cells, & B & T memory cells that respond & proliferate quickly if specific antigen reappears

B

Central chemoreceptors are mainly sensitive to (A) decreases in PO2 (B) changes in PCO2 & pH (C) changes in Pa (D) decreases in Pa

1. dense, inactive 2. dispersed, active

Chromatin organization occurs in 2 basic forms: 1. heterochromatin - histochemically (dispersed/dense) & transcriptionally (active/inactive) 2. euchromatin - histochemically (dispersed/dense) & transcriptionally (active/inactive)

C

Chronic hepatitis is inflammation of the liver lasting > __ _______ (A) 48 hours (B) 2 weeks (C) 6 months (D) 2 years

B

Closure of the neural tube to form the 3 primary vesicles (forebrain, midbrain, & hindbrain) occurs during the _____ week of development (A) third (B) fourth (C) sixth (D) tenth

D

Clot initially bulges into blood vessel lumen then contracts slightly due to activity of platelet-derived actin & myosin (A) primary aggregation (B) secondary aggregation (C) Blood coagulation (D) Clot retraction (E) Clot removal

D

Combination OCP where estrogen & progestin levels are changed 4 times in the cycle of use. Also used for menorrhagia (A) monophasic preparations (B) biphasic preparations (C) triphasic preparations (D) quadraphasic preparations

>

Compliance of veins (<,>,=) compliance of arteries

B

Connecting tubules from several nephrons join to form the cortical collecting ducts of simple cuboidal epithelium, which enter medulla in parallel with _______ & become larger w/ more columnar cells (A) PCT & DCT (B) loops of Henle & vasa recta (C) afferent & efferent arterioles (D) PCT & interlobular arteries

phrenic nerve vagus CN X thoracic duct left recurrent laryngeal nerve brachiocephalic aortic arch thymus trachea lymph nodes esophagus SVC

Contents of the Superior Mediastinum (PVT Left BATTLES)

E

Coronary angiographs of a 44-year-old male patient reveal an occlusion of the circumflex branch of the left coronary artery. This patient has been suffering from myocardial infarction in which of the following areas? (A) Right and left ventricles (B) Right and left atria (C) Interventricular septum (D) Apex of the heart (E) Left atrium and ventricle

diastole, systole

Coronary arteries have maximal blood flow during (systole/diastole) and minimal blood flow during (systole/diastole) because of compression of arterial branches in myocardium during systole

D

Corticosteroids are used for suppression of inflammation & immune reactions. Corticosteroids treat the primary disease in these cases. (A) first true, second true (B) first false, second true (C) first false, second false (D) first true, second false

A

Cortisol & insulin are example of ______ hormones which are transported in blood to a target distant from site of secretion (A) Endocrine (B) Paracrine (C) Autocrine

C

Cortisol has a weak mineralcorticoid action, and the mineralcorticoid receptor binds aldosterone & cortisol with (A) higher affinity for aldosterone (B) higher affinity for cortisol (C) equal affinity

A

Create hydrophilic pores that allow rapid movement of solutes when open (A) Channel proteins (B) Carrier proteins (C) Passive transport (D) Active transport

D

Dopamine stimulates D2 receptors to inhibit adenylyl cyclase & thereby inhibits _____ synthesis and secretion (A) oxytocin (B) estradiol (C) testosterone (D) prolactin (E) hCG

metabolite

Drugs that exert their toxic effects on the liver may do so through the hepatic production of a toxic ________

A

During Rapid ventricular ejection, the ventricle continues to contract → ventricular P > aortic pressure → (A) aortic valve opens (B) aortic valve closes (C) mitral valve opens (D) mitral valve closes

B

During development, ________ can give rise to ALL types of differentiated tissues (A) multipotent (B) totipotent (C) pluripotent

↑↑ ↑ ↑↑ ↑ ↑ ↓↓ ↑↑

During exercise, sympathetic outflow to the heart & blood vessels is increased, resulting in: -HR (↑↑/↓↓) -Stroke volume (↑/↓) -cardiac output (↑↑/↓↓) -arterial pressure (↑/↓) -pulse pressure (↑/↓) -TPR (↑↑/↓↓) -AV O2 difference (↑↑/↓↓)

C

During exercise, total peripheral resistance (TPR) decreases because of the effect of (A) the sympathetic nervous system on splanchnic arterioles (B) the parasympathetic nervous system on skeletal muscle arterioles (C) local metabolites on skeletal muscle arterioles (D) local metabolites on cerebral arterioles (E) histamine on skeletal muscle arterioles

E

During phase 0 (upstroke), there is rapid depolarization due to _______ (A) inward Na+ current (B) outward Na+ current (C) inward K+ current (D) outward Ca2+ current (E) inward Ca2+ current

B

During the _____ stage of granulopoiesis, azurophilic granules are secreted in the golgi (A) myeloblast (B) promyelocyte (C) myelocyte (D) band cell (E) metamyelocyte

C

During the _____ stage of granulopoiesis, the golgi packages proteins for specific granules - 1st visible sign of differentiation into specific granulocyte type (A) myeloblast (B) promyelocyte (C) myelocyte (D) band cell (E) metamyelocyte

E

During the follicular phase, increasing _____ leads to estradiol synthesis & granulosa cell proliferation (A) estrogen (B) LH (C) prolactin (D) oxytocin (E) FSH

D

During the luteal phase, the ruptured follicle becomes the corpus luteum, which secretes _____ & _____ to sustain the oocyte & stimulate preparation of endometrium for implantation of fertilized ovum (A) prolactin & progesterone (B) progesterone & FSH (C) FSH & LH (D) progesterone & estradiol

D

Efferent arterioles from cortical glomeruli branch diffusely as ________. Those from juxtamedullary glomeruli branch as long microvascular loops called _______ (A) afferent arterioles, efferent arterioles (B) interlobular arteries, peritubular capillaries (C) efferent arterioles, interlobular arteries (D) peritubular capillaries, vasa recta

B, E

Estradiol → continued dominant follicle maturity → uterine endometrial growth stimulated → increased ___ secretion from the pituitary and suppression of ___ secretion (A) estrogen (B) LH (C) prolactin (D) oxytocin (E) FSH

C

G6P → (A) release of free glucose to hepatocytes (B) taking up of free glucose by hepatocytes (C) release of free glucose to the blood (D) glucose synthesis

A

Ga1NAc α1, 3 + H substance = (A) A blood (B) B blood (C) AB blood (D) O blood

C

Ga1NAc α1, 3 + galactose α1, 3 + H substance = (A) A blood (B) B blood (C) AB blood (D) O blood

B ("oh my Gauch, he's such a Bro" (in crying voice))

Gaucher Disease has a deficiency in the enzyme _________ and therefore has a build up of __________ (A) alpha-galactosidase A; Ceramide Trihexosidase (B) glucocerebrosidase; glucocerebroside (C) hexosaminidase A; GM2 ganglioside (D) sphingomyelinase; sphingomyelin (E) beta galactocerebrosidase; galactocerebroside

B

Gilbert's syndrome, Crigler-Najjar syndrome, & Dubin-Johnson & Rotor's Syndromes are examples of genetic disorders that (A) impair communication between hepatocytes & plasma (B) impair bilirubin conjugation or secretion (C) impair protein metabolism (D) cause deficiency in alpha1-antitrypsin

B

Heart block rhythms occur when the cardiac electric impulse is delayed or blocked. ______ is a delay rather than an actual block and is caused by a conduction delay at the AV node of bundle of His → PR interval will be ______ (A) 1st degree, shorter (B) 1st degree, longer (C) 2nd degree type 1, shorter (D) 2nd degree type 2, short (E) 2nd degree type 1, longer (F) 2nd degree type 2, longer (G) 3rd degree, shorter (H) 3rd degree, longer

C

Hormone therapy may result in clinical problems attributable to I. Excess hormone administration II. Loss of nocturnal rhythm III. Loss of diurnal rhythm IV. Loss of physiological pusatility (A) I only (B) I and III (C) I, III, and IV (D) I, II, III, and IV (E) I, II, and IV

A

In ALL sphingolipids, long-chain fatty acid attached to amino group of sphingosine in an ________ → sphingolipids are non-saponifiable → facilitates separation from alkali-labile glycerolipids (A) amide linkage (B) ester linkage (C) hydrogen linkage (D) glycerol linkage

C

In healthy bone canaliculi are likely to contain which one of the following? (A) Capillaries (B) Nerve axons (C) Osteocytic processes (D) Osteoid (E) Osteoclasts in resorption lacunae

longer, low

In liver disease, hepatocytes will be affected → ________ PT & ______ serum albumin

C

In mature organisms, Adult Stem Cells (tissue stem cells) have capacity to replace damaged cells & maintain cell populations w/i (A) all cells (B) tissue with common progenitors (C) tissues they reside (D) no capacity to replace or maintain

D

In negative chronotropic effects, the parasympathetic system releases ___ → activates SA node _____ receptors → (A) norepinephrine, beta 1 (B) norepinephrine, beta 2 (C) ACh, alpha 1 (D) ACh, Muscarinic (M2)

left, right

In systemic hypertension, the (right/left) ventricle hypertrophies In pulmonary hypertension, the (right/left) ventricle hypertrophies

C

In the Bainbridge reflex, Increased pressure detected at ________ → increase in HR → increase in cardiac output → increased renal perfusion & increased Na+ & water excretion (A) carotid sinus baroreceptors (B) medullary chemoreceptors (C) venous high-pressure receptors (D) Atrial high-pressure receptors

D

In the SA node (A) phase 0 through 5 are present (B) phase 1 is absent (C) phase 2 is absent (D) B & C

B

In the _____, glucose, organic nutrients, small proteins & peptides (AAs), & a lot of water & electrolytes are REABSORBED from the filtrate & transferred to peritubular capillaries (A) renal corpuscle (B) PCT (C) Loop of Henle (D) DCT

B

In the adrenal glands, the zona fasciculata & zona reticularis are places of synthesis of ___________; the outer layer (zona glomerulosa) synthesizes ________ (A) estrogens; androgens (B) cortisol & adrenal androgens; aldosterone (C) aldosterone; cortisol & androgens (D) cortisol; adrenal androgens & aldosterone

D

Increased secretion of ANP binds to ANP receptors on vascular smooth muscle causing (A) vasoconstriction → increased Na+ & water reabsorption by kidneys (B) vasodilation → increased Na+ & water reabsorption by kidneys (C) vasoconstriction → increased Na+ & water excretion by kidneys (D) vasodilation → increased Na+ & water excretion by kidneys

vasodilation, vasoconstriction

Increases in PCO2 → (vasoconstriction/vasodilation) of cerebral arterioles Decreases in PCO2 → (vasoconstriction/vasodilation) of cerebral arterioles

vasoconstriction, vasodilation

Increases in sympathetic tone → (vasoconstriction/vasodilation) Decreases in sympathetic tone → (vasoconstriction/vasodilation)

A

Innervates the lateral rectus muscle (A) GSE column (B) GVE column (C) alar plate (D) pontine nuclei (E) SVE column

A

Inspiration "splits" the second heart sound because (A) the aortic valve closes before the pulmonic valve (B) the pulmonic valve closes before the aortic valve (C) the mitral valve closes before the tricuspid valve (D) the tricuspid valve closes before the mitral valve (E) filling of the ventricles has fast and slow components

A

Ionotropic agents change (A) cardiac output curve (B) venous return curve (C) A & B

D

Isovolumetric Ventricular contraction corresponds to the depolarization of the ventricles seen as the QRS complex. L ventricle contracts → when L ventricular P > L atrial P → _________ → S1 sound (A) aortic valve opens (B) aortic valve closes (C) mitral valve opens (D) mitral valve closes

C

Major estrogen detected in women (A) estrone (B) estradiol (C) estriol

D

Major metabolic products of corticosteroid metabolism found in urine - ____________ - can be measured to assess adrenal-pituitary function, however direct measurement of blood cortisol concentrations by radioimmunoassay more commonly used (A) 17-hydroxycorticosteroids (B) 17-ketosteroids (C) 17-hydroxyestradiol (D) A & B (E) A & C

C

Many immune-related cellular activities are often impaired in aged patients. Which lymphoid organ(s) normally develop less functionality and increasing amounts of adipose tissue with age? (A) Axillary lymph nodes (B) Lingual tonsils (C) Thymus (D) Splenic white pulp (E) Splenic red pulp

monocytes, lymphocytes

Monoblasts produce ________ in red marrow but lymphoblasts produce ________ in lymphoid tissues in processes involving acquired immunity

F

Monocyte that destroys invading organisms (A) neutrophils (B) basophils (C) eosinophils (D) B lymphocytes (E) T lymphocytes (F) macrophages

C

Most common reasons for _____Rx are (1) prevention of conception and (2) reduce sequlae associated w/ declining hormone levels during & after menopause (A) progesterone (B) androgens (C) estrogen (D) progestin

C

Myocardial contractility is best correlated with the intracellular concentration of (A) Na+ (B) K+ (C) Ca2+ (D) Cl- (E) Mg2+

B

Neurotransmitters are examples of ________ hormones which exert action locally at site of secretion (A) Endocrine (B) Paracrine (C) Autocrine

B

Nitrogen atoms are incorporated into urea from 2 sources: (A) tryptophan & alanine (B) glutamate & asparate (C) glycine & asparagine (D) tyrosine & histidine

cortisol

Normal patient - stressful situation → increased release of ACTH → production of ________

1. SA node 2. 60 to 100 3. myocardium

Normal sinus rhythm is the normal pattern & timing of electrical activation of heart, with the following 3 criteria: 1. Action potential originates at ____ _____ 2. SA nodal impulses at regular rate of ___ to ____ impulses per minute 3. Activation of _______ is in correct sequence with correct timing & delays

B

On the graph showing left ventricular volume and pressure, isovolumetric contraction occurs between points (A) 4→1 (B) 1→2 (C) 2→3 (D) 3→4

D

Osteogenesis imperfecta or "Brittle Bone Disease" is a group of related congenital disorders in which osteoblasts produce deficient amounts of or defective _______ → fragility of bones (A) hydroxyapatite (B) calcium (C) fibroblasts (D) type I collagen

C

Osteogenesis occurs by 1 of 2 processes: 1. Osteoblasts differentiate from mesenchyme & begin secreting osteoid 2. A preexisting matrix of hyaline cartilage is eroded & invaded by osteoblasts, which then begin osteoid production (A) 1. endochondral ossification 2. intramembranous ossification (B) 1. appositional growth 2. endochondral ossification (C) 1. intramembranous ossification 2. endochondral growth (D) 1. intramembranous ossification 2. appositional growth

A

Osteosarcoma arises in (A) osteoprogenitor cells (B) osteocytes (C) osteoclasts (D) osteoblasts

A

Ovarian follicles become depleted of oocytes after ___ to ____ years of ovulatory cycles & normal pregnancy is no longer possible (A) 30 to 40 (B) 50 to 60 (C) 35 to 40 (D) 45 to 50

D

PA group transferred to free _____ (A) serine, guanine, or inositol (B) serine, adenosine, leucine (C) glycine, glutamate, tyrosine (D) serine, glycerol, or inositol

B

Phospholipids undergo maturation in remodeling pathways. _____ groups at sn-2 position replaced with new ones → yield diversity & asymmetry of hydrophobic moiety of phospholipids (A) hydroxyl (B) acyl (C) alcohol (D) ester

C, B, A, D

Place the conducting system of the heart in the correct order: A. AV bundle (Bundle of His) B. AV node C. SA node D. Purkinje fibers

B, D, A, C

Place the following basic mechanism of signal transduction pathways in order: A. biochemical events B. Ligand binds receptor C. signal transduction D. conformational change in intracellular domain of receptor

B, D, A, C

Place the following characteristic sequealae of severe liver disease in the correct order: A. Abdominal distension due to accumulation of fluid (ascites) B. Jaundice C. Altered consciousness level (hepatic encephalopathy) D. Ready bruising & profuse bleeding

B, D, A, E, C

Place the following epiphyseal plate regions of activity in order from FARTHEST to CLOSEST to ossification center: A. Zone of hypertrophy B. Zone of reserve cartilage C. Zone of ossification D. Proliferative zone E. Zone of calcified cartilage

C, D, B, A

Place the following in the correct order in the spread of action potential throughout the myocardium: A. Bundle of His, Purkinje system, & ventricles B. AV node C. SA node (pacemaker) D. Atrial internodal tracts & atria

C, F, B, D, E, A, G

Place the following steps in the current hypothesis for the MoA of steroid hormones in order: A. Gene activation & transcription of mRNA B. Transforms receptor to active configuration C. Secretion of hormones into blood stream D. Activated complex binds w/ high affinity to specific nuclear sites E. Recruits a coactivator to promotor region F. Unbound hormones enter cell by diffusion & bind to receptors in nucleus G. After nuclear interaction, receptor-hormone complex dissociates

A, D, C, F, B, E

Place the following steps in the mechanism of G-protein couple receptor action in order: A. Ligand binding B. G protein activated C. Interaction w/ receptor-ligand complex D. G protein bound to GDP E. cAMP & IP3 released F. GDP exchanged for GTP

D, A, B, F, E, C, G

Place the following steps in the mechanism of the biosynthesis of thyroid hormones in order: A. Oxidation of TPO in peroxisome → iodine B. Conversion of tyrosyl residues on surface of thyroid glycoprotein Tg to either MIT or DIT C. Tg is hydrolyzed into lysosomes to release free T4 & T3 D. Iodide concentrated in follicular epithelial cells after entry via sodium-iodide symporter E. Tg is exocytosed F. Coupling of iodinated tyrosine residues to form T4 & T3 G. free T4 & T3 are transported to plasma membrane and released into bloodstream

C

Protein assembly (A) RER (B) Mitochondria (C) Golgi apparatus (D) SER (E) Cytoskeleton

C

Risk of down syndrome increases with maternal age. A 40 year old woman has a ______ risk of having a child with Down syndrome. (A) 1 in 1000 (B) 1 in 10 (C) 1 in 100 (D) 1 in 10000

C

S1 ("lub") heart sound is caused by (A) closure of tricuspid & mitral valves at onset of ventricular diastole (B) opening of tricuspid & mitral valves at onset of ventricular diastole (C) closure of tricuspid & mitral valves at onset of ventricular systole (D) opening of tricuspid & mitral valves at onset of ventricular systole

A

S1 (Lub) is made by __________ and occurs at the _________ (A) vibration and closure of AV valves (tricuspid & mitral); beginning of systole (B) vibration and closure of AV valves (tricuspid & mitral; end of systole (C) closure of semilunar (aortic & pulmonic) valves; beginning of systole (D) closure of semilunar (aortic & pulmonic) valves; end of systole

C

Sphingolipids containing covalently bound sugars are known as _______ (A) sphingomyelin (B) glycerophosphate (C) glycolipids (D) hydrolipids

D

The ______ gives rise to the metencephalon (pons & cerebellum) and the myelencephalon (medulla oblongota) (A) prosencephalon (B) mesencephalon (C) midbrain (D) rhombencephalon

B

The aortic valve is most audible (A) Over the medial end of the second left intercostal space (B) Over the medial end of the second right intercostal space (C) In the left fourth intercostal space at the midclavicular line (D) In the left fifth intercostal space at the mid- clavicular line (E) Over the right half of the lower end of the body of the sternum

C

The attending faculty in the coronary intensive care unit demonstrates to his students a normal heart examination. The first heart sound is produced by near-simultaneous closure of which of the following valves? (A) Aortic and tricuspid (B) Aortic and pulmonary (C) Tricuspid and mitral (D) Mitral and pulmonary (E) Tricuspid and pulmonary

C

The commonly used drug _______ (paracetamol) is hepatotoxic in excess. (A) NSAIDs (B) Heparin (C) Acetaminophen (D) penicillin

D

The condensation of the mesenchyme leads to formation of the ________ (A) cerebral aqueduct (B) 4th ventricle (C) subarachnoid space (D) ligamentum denticulatum

D

The coronary sinus opens onto the ________________ (A) Left atrium (B) Right ventricle (C) Left ventricle (D) Right atrium

D

To repair baby girl Weisen (Carley), a dacron patch is placed over the holes between the atria and ventricles, and the single AV valve is then divided to make 2 effective valves which will (A) increase flow to right side of heart (B) increase flow to left side of heart (C) block flow to right side of heart (D) return normal blood flow between heart and lungs

C

Transports iron ions and lipids in bloodstream (A) albumin (B) alpha-globulins (C) beta-globulins (D) gamma-globulins (E) fibrinogen

B

Why can the de novo pathway not explain diversity & asymmetry of phospholipids? (A) it can explain asymmetry but not diversity (B) phosphatidic acid & DAG are common precursors of both triglycerides & phospholipids (C) PLA2 and LPLAT are common precursors of both triglycerides & phospholipids (D) It does explain diversity & asymmetry

more force needed to pump blood through the body than the lungs

Why does the left ventricle have a thicker muscular wall than the right ventricle?

D

_______ is synthesized in the paraventricular & ventromedial nuclei of the hypothalamus → suppresses the release of GI hormones gastrin, cholecytokinin, vasoactive intestinal peptide (VIP), & gastric inhibitory polypeptide, insulin, & glucagon (A) GnRH (B) CRH (C) Ghrelin (D) Somatostatin (GHIH) (E) GHRH

B

_______ is the work the heart performs on each beat (A) stroke volume (B) stroke work (C) cardiac output (D) cardiac minute work

D

_______ is used to treat osteoporosis & vasomotor symptoms associated with menopause & urogenital atrophy (A) progestin replacement therapy (B) combination hormone replacement therapy (C) androgen replacement therapy (D) estrogen replacement therapy

D

________ is the primary regulator of blood flow to skeletal muscle at rest, whereas ______ regulates blood flow to skeletal muscle (A) local metabolic factors, sympathetic innervations (B) parasympathetic innervations, sympathetic innervations (C) sympathetic innervations, parasympathetic innervations (D) sympathetic innervations, local metabolic factors

A

________ marks intracellular proteins for proteasomal degradation (A) ubiquitin (B) opsonin (C) nitrogen (D) ammonia

A

________ of TFs allow specific binding to short DNA sequences (A) DNA-binding domains (B) RNA-binding domains (C) protein-protein interaction domains (D) kinase domains

D

a(n) ______ is a substance or a characteristic that is measured as an indicator of normal or pathologic processes (A) CRP (B) interleukin (C) cytokine (D) biomarker

innate immunity

involves immediate, nonspecific actions, including physical barriers such as skin & mucus membranes of GI, respiratory, and urogenital tracts that prevent infections or penetration of host body

C

-Increase in blood flow to an organ that occurs after a period of occlusion of flow -The longer the period of occlusion → the greater the increase in blood flow above preocclusion levels (A) Autoregulation (B) Active hyperemia (C) Reactive hyperemia (D) Hormonal (extrinsic) control of blood flow

C

-Inhibit specific immune responses -Produce peripheral tolerance - acts to supplement the central tolerance that develops in thymus (A) CD4+ T helper cells (B) CD8+ cytotoxic T cells (C) CD4+CD25+ regulatory T cells (D) γδ T cells

D

-Inhibition of T-type Ca2+ channels & inhibition of sustained repetitive neuronal firing by prolonging inactivated state of voltage-gated Na+ channel -Adjunct for refractory partial seizures (A) Gabapentin (B) Lamotrigine (C) Topiramate (D) Zonisamide

C

-Inhibits GAT-1 (GABA transporter) → inhibits GABA uptake → increases concentration of GABA in synaptic cleft → prolongs duration of inhibitory synaptic currents -Adjunct therapy in patients ≥ 12 w/ refractory partial seizures (A) Benzodiazepine (B) Vigabatrin (C) Tiagabine (D) Gabapentin

B

-Initial depolarization of atria → initial depolarization of ventricles (A) P wave (B) PR interval (C) QRS complex (D) T wave (E) QT interval

B

-Irreversible inhibitor of GABA transaminase (major degradation enzyme for GABA) → increase concentration of GABA in brain → increase GABA mediated inhibitory activity -Adjunct therapy for refractory complex partial seizures in adults -Infantile spasms (A) Benzodiazepine (B) Vigabatrin (C) Tiagabine (D) Gabapentin

A

-Kill & phagocytose bacteria -50-70% of leukocytes -2 to 5 lobed nucleus (A) neutrophils (B) lymphocytes (C) eosinophils (D) basophils (E) monocytes

C

-Kill helminthic & other parasites; modulate local inflammation -1-4% of leukocytes -bilobed nucleus (A) neutrophils (B) lymphocytes (C) eosinophils (D) basophils (E) monocytes

A

-Lack of GH during growing years → _________ -Excess GH → ________ -Increase in GH in adults where long bones become very thick → _________ (A) pituitary dwarfism, gigantism, acromegaly (B) acromegaly, giganitism, pituitary dwarfism (C) pituitary dwarfism, rickets, gigantism (D) rickets, gigantism, acromegaly

statins

(HMGR inhibitors) lower cholesterol by binding to HMG-CoA binding site on enzyme & competitively inhibiting its activity → decrease in intracellular cholesterol concentration → stimulates expression of LDL receptors → increases LDL clearance → decreases plasma LDL-cholesterol What class of drug is described above?

D

(aka Type A cells) of synovial membrane remove wear-and-tear debris from synovial fluid (A) osteocytes (B) Fibroblast-like synovial cells (C) Collagen-like synovial cells (D) Macrophage-like synovial cells

B

(aka type B cells) of synovial membrane synthesize hyaluronan that moves into synovial fluid w/ water from local capillaries to lubricate & nourish articular cartilage (A) osteocytes (B) Fibroblast-like synovial cells (C) Collagen-like synovial cells (D) Macrophage-like synovial cells

aortic, pulmonic

(aortic/pulmonic) valve closes followed by (aortic/pulmonic) valve, creating the S2 sounds during isovolumetric ventricular relaxation

C

(increases/decreases) in blood volume or (increases/decreases) in venous compliance → decrease in mean systemic P → venous return curve shifts L in parallel fashion → new equilibrium where (A) increases, increases, both cardiac output & R atrial pressure are increased (B) increases, decreases, both cardiac output & R atrial pressure are increased (C) decreases, increases, both cardiac output & R atrial pressure are decreased (D) decreases, decreases, both cardiac output & R atrial pressure are decreased

B

(increases/decreases) in blood volume or (increases/decreases) in venous compliance → increased mean systemic pressure → venous curve shifts to the R in parallel fashion → new equilibrium at which (A) increases, increases, both cardiac output & R atrial pressure are increased (B) increases, decreases, both cardiac output & R atrial pressure are increased (C) decreases, increases, both cardiac output & R atrial pressure are decreased (D) decreases, decreases, both cardiac output & R atrial pressure are decreased

A

(increasing/decreasing) TPR → decrease in cardiac output & venous return → new equilibrium point where (A) increasing, both cardiac output & venous return are decreased but R atrial pressure is the same (B) increasing, both cardiac output & venous return are increased but R atrial pressure is the same (C) decreasing, both cardiac output & venous return are decreased but R atrial pressure is the same (D) decreasing, both cardiac output & venous return are increased but R atrial pressure is the same

D

(increasing/decreasing) TPR → increase in both cardiac output & venous return → new equilibrium point where (A) increasing, both cardiac output & venous return are decreased but R atrial pressure is the same (B) increasing, both cardiac output & venous return are increased but R atrial pressure is the same (C) decreasing, both cardiac output & venous return are decreased but R atrial pressure is the same (D) decreasing, both cardiac output & venous return are increased but R atrial pressure is the same

high, low

(low/high) AFP indicates neural tube defects or abdominal wall defects (low/high) AFP indicates down syndrome or fetal wastage

essential, nonessential

(nonessential/essential) AAs have C skeletons that can't be derived from normal human metabolism & MUST be supplied in diet (nonessential/essential) AAs are NOT required in diet - can be synthesized through biosynthesis of metabolites

generalized, partial

(partial/generalized) are usually genetic in etiology whereas (partial/generalized) have a broad etiology - tumors, developmental malformation, cortical lesions, or due to trauma/stroke

volume, pressure, pressure

(pressure/volume) work predominates during strenuous exercise (pressure/volume) work predominates during aortic stenosis (pressure/volume) work is more costly

right, left

(right/left) ventricular hypertrophy is usually caused by a problem in your lungs which causes pulmonary arterial HTN or a congenital defect (right/left) ventricular hypertrophy is usually caused by HTN, but can also be caused by aortic stenosis, hypertrophic cardiomyopathy, or athletic training

D

-Liver & spleen enlargement, mental retardation -Major storage product = galactocerebroside -Deficient enzyme = beta-galactosidase (A) Tay-sachs (B) Gaucher's (C) Fabry's (D) Krabbe's

B

-Liver & spleen enlargement, mental retardation in infantile form -Major storage product = glucocerebroside -Deficient enzyme = beta-glucosidase (A) Tay-sachs (B) Gaucher's (C) Fabry's (D) Krabbe's

C

-Macromolecules like LDL & transferrin -Defects → familial hypercholesterolemia (A) Caveolae-mediated endocytosis (B) Pinocytosis (C) Receptor-mediated Endocytosis (D) Transcytosis

B

-Mainly cortical -Simple cuboidal cells w/ long microvilli in lumen -Abundant mitochondria -Large interdigitating basolateral folds (A) renal corpuscle (B) PCT (C) Loop of Henle (D) DCT

B

-Mechanically attach cells & their cytoskeletons to other cells or the ECM -Cadherin & integrin proteins (A) tight junctions (B) anchoring junctions (desmosomes) (C) communicating (gap) junctions (D) all of the above

B

-Mitogen for all erythroid progenitor & precursor cells, also promoting their differentiation -source - peritubular endothelial cells of kidney/hepatocytes (A) stem cell factor (SCF) (B) Erythropoietin (EPO) (C) Thrombopoietin (TPO) (D) Granulocyte Colony-Stimulating Factor (G-CSF) (E) Monocyte Colony-Stimulating (M-CSF) (F) IL-1

A

-Mitogen for all hemopoietic progenitor cells -source - stromal cells of bone marrow (A) stem cell factor (SCF) (B) Erythropoietin (EPO) (C) Thrombopoietin (TPO) (D) Granulocyte Colony-Stimulating Factor (G-CSF) (E) Monocyte Colony-Stimulating (M-CSF) (F) IL-1

C

-Mitogen for megakaryoblasts & their progenitor cells -source - kidney & liver (A) stem cell factor (SCF) (B) Erythropoietin (EPO) (C) Thrombopoietin (TPO) (D) Granulocyte Colony-Stimulating Factor (G-CSF) (E) Monocyte Colony-Stimulating (M-CSF) (F) IL-1

E

-Mitogen for monocyte precursor cells -source - endothelial cells of marrow & macrophages (A) stem cell factor (SCF) (B) Erythropoietin (EPO) (C) Thrombopoietin (TPO) (D) Granulocyte Colony-Stimulating Factor (G-CSF) (E) Monocyte Colony-Stimulating (M-CSF) (F) IL-1

D

-Mitogen for neutrophil precursor cells -source - endothelial cells of bone marrow & macrophages (A) stem cell factor (SCF) (B) Erythropoietin (EPO) (C) Thrombopoietin (TPO) (D) Granulocyte Colony-Stimulating Factor (G-CSF) (E) Monocyte Colony-Stimulating (M-CSF) (F) IL-1

B

-Mitogenic effects on hepatocytes & epithelial cells -Morphogen during embryonic development -Promotes cell migration -Enhances hepatocytes survival -Frequently overexpressed/mutated in tumors (A) EGF & TGF-α (B) HGF (C) PDGF (D) VEGFs (E) FGF (F) TGF-β

D

-Modulate inflammation and release histamine during allergy -< 1% of leukocytes (A) neutrophils (B) lymphocytes (C) eosinophils (D) basophils (E) monocytes

A

-Most abundant Ig -Protects tissue spaces & freely crosses the placenta -Humoral immunity for fetus before maturation of immune system (A) IgG (B) IgA (C) IgM (D) IgD (E) IgE

A

-Non-coated plasma membrane invaginations -Small molecules -Mutations in the main protein associated w/ muscular dystrophy & electrical heart abnormalities (A) Caveolae-mediated endocytosis (B) Pinocytosis (C) Receptor-mediated Endocytosis (D) Transcytosis

C

-Occurs when meninges & spinal cord project through a vertebral defect & forms a sac -usually present in arnold-chiari malformation (A) spina bifida occulta (B) meningocele (C) meningomyelocele (D) myelocele (E) Syringomyelocele

B

-Occurs when meninges project through a vertebral defect forming a sac filled with CSF -exists w/ spinal cord remaining in its normal position (A) spina bifida occulta (B) meningocele (C) meningomyelocele (D) myelocele (E) Syringomyelocele

B

-Organize intracellular pathways -Molecular connectors linking enzymes → promote assembly of complexes -Can determine downstream signaling events by influencing proteins recruited to signaling complexes (A) Kinases (B) Adaptor proteins (C) Transcription Factors (D) Growth Factors

B

-Originate in temporal or frontal lobe → spread to broader areas in bilateral pattern -Consciousness impaired -Flashbacks, autonomic disregulation & automatisms common (A) Simple partial seizure (B) Complex partial seizure (C) Secondary Generalized seizure (D) Generalized tonic-clonic (grand mal) seizures (E) Petit Mal (Absence seizures)

B

-Polypeptides traversing membrane 7 times -ligand binding → receptor associated with intracellular GTP binding protein (A) Receptor Tyrosine Kinases (RTKs) (B) G-protein coupled receptors (C) Nuclear receptors (D) Nonreceptor tyrosine kinase

A

-Positive allosteric mediators -Bind to GABA-A receptors → increase frequency of GABA-A receptor activation & associated Cl- channel opening -Short-term therapy or emergency treatment (A) Benzodiazepine (B) Vigabatrin (C) Tiagabine (D) Gabapentin

B

-Positive allosteric modulator of neuronal K+ channels (KNXQ) -Increases KNXQ channels open at rest → limits overall excitability of neurons → facilitates their recovery from membrane depolarization → limits neuronal action potential burst firing → decreases probability of initiation & propagation of seizures (A) Gabapentin (B) Retigabine (C) Topiramate (D) Zonisamide

E

-Precursors of macrophages and other mononuclear phagocytic cells -large indented or C-shaped nuclei -basophilic (A) neutrophils (B) lymphocytes (C) eosinophils (D) basophils (E) monocytes

A

-Principal mechanism -transfer of amino group to a suitable keto acid acceptor (A) Transamination (B) Oxidative deamination (C) Removal of a molecule of water by a dehydratase (D) Reductive deamination

A

-Produced by macrophages & epithelial cells -Mitogenic for hepatocytes, fibroblasts, & epithelial cells -Intrinsic tyrosine kinase activity -mutations/amplifications occur in many cancers (A) EGF & TGF-α (B) HGF (C) PDGF (D) VEGFs (E) FGF (F) TGF-β

C

-Provide tensile strength & allow cells to bear mechanical stress -Major structural proteins of epidermis & hair (A) Actin (B) Myosin (C) Intermediate filaments (D) Microtubules

E

-Rare -Meningomyelocele + central canal of spinal cord at level of bony defect is grossly dilated (A) spina bifida occulta (B) meningocele (C) meningomyelocele (D) myelocele (E) Syringomyelocele

A

-Reduction in activity & expression of HMG-CoA reductase - limits cholesterol synthesis -Downregulation of LDL receptors - limits cellular uptake of cholesterol -Increase in cholesterol & phospholipid efflux from cell to HDL - decreases intracellular cholesterol -Increase in rate of conversion of cholesterol to bile acids - increases cholesterol elimination The above would be the result of (A) increase in free cholesterol (B) decrease in free cholesterol

F

-Regulates activities & cytokine secretion of many leukocytes & other cells -source - Macrophages & T helper cells (A) stem cell factor (SCF) (B) Erythropoietin (EPO) (C) Thrombopoietin (TPO) (D) Granulocyte Colony-Stimulating Factor (G-CSF) (E) Monocyte Colony-Stimulating (M-CSF) (F) IL-1

A

-Sabin polio vaccine -MMR vaccine -varicella zoster vaccine -BCG vaccine against TB -Salmonella typhi (TY21a) vaccine against typhoid the above are examples of (A) live attenuated vaccines (B) inactivated vaccines (C) subunit vaccines (D) all of the above

B

-Salk polio vaccine -Influenza vaccines -Hep A vaccine -Diphtheria & tetanus toxoid vaccines the above are examples of (A) live attenuated vaccines (B) inactivated vaccines (C) subunit vaccines (D) all of the above

A

-Seal adjacent cells together → continuous barrier restricting movement b/w cells -Occluding & claudin proteins -Form boundary b/w apical & basolateral domains → maintain cell polarity -Dynamic (A) tight junctions (B) anchoring junctions (desmosomes) (C) communicating (gap) junctions (D) all of the above

F

-Serine/threonine kinase receptor activity → Snads →→ activate or inhibit gene transcription -pleiotropic -stimulate MMP activity -inhibit lymphocyte proliferation (A) EGF & TGF-α (B) HGF (C) PDGF (D) VEGFs (E) FGF (F) TGF-β

C

-Skin rash, kidney failure, pain in lower extremities -Major storage product = ceramide trihexoside -Deficient enzyme = alpha-galactosidase (A) Tay-sachs (B) Gaucher's (C) Fabry's (D) Krabbe's

A

-Slow, hormonal mechanism -Used in LT BP regulation by adjustment of blood V (A) Renin-Angiotensin II-Aldosterone System (B) Baroreceptors (C) Peripheral Chemoreceptors (D) Central Chemoreceptors

A

-Small adhesion focus b/w cells → ______ -cadherins associated with intermediate filaments communicate extracellular forces over multiple cells (A) spot desmosome (B) tight junction (C) hemidesmosome (D) belt desmosome

B

-Source of ATP & metabolic intermediates for anabolic metabolism -site of heme synthesis -sensory of cell damage → can initiate/regulate apoptosis (A) RER (B) Mitochondria (C) Golgi apparatus (D) SER (E) Cytoskeleton

A

-Specific or localized motor, sensory -Consciousness NOT impaired -Seizure may stay localized or spread to contiguous brain tissue → progressive symptoms as depolarization spreads along cerebral cortex (Jacksonian March) (A) Simple partial seizure (B) Complex partial seizure (C) Secondary Generalized seizure (D) Generalized tonic-clonic (grand mal) seizures (E) Petit Mal (Absence seizures)

D

-Stabilize inactive state of voltage-gated Na+ channels but binds to a different site on Na+ channels than phenytoin -broad spectrum antiseizure drug + mania in bipolar disorder & for migraine pain (A) Hydrantoins - Phenytoin (B) Barbiturates - Phenobarbital (C) Carbamazepine (D) Valproic Acid (E) Succinimides - Ethosuximide

C

-Stored in platelet granules & released on platelet activation -Bind to 2 cell surface receptors w/ intrinsic tyrosine kinase activity → induces fibroblast, endothelial & smooth muscle cell proliferation & matrix synthesis (A) EGF & TGF-α (B) HGF (C) PDGF (D) VEGFs (E) FGF (F) TGF-β

D

-Support cables for "molecular motor" proteins -Mechanical support for sister chromatid separation during mitosis -Core of primary cilia -Core of motile cilia or flagella (A) Actin (B) Myosin (C) Intermediate filaments (D) Microtubules

C

-Synthesized by biliary tract & bone & in pregnancy by placenta -Plasma activity of another enzyme that originates in biliary tract (i.e. GGT) can be measured to confirm hepatic origin of a raised serum activity (A) AST (B) ALT (C) ALP (D) conjugated bilirubin

C

-Systole ends & ventricle relaxes -Ventricular P < Aortic P → aortic valve closes -Ventricular volume remains constant (A) isovolumetric contraction (B) ventricular ejection (C) isovolumetric relaxation (D) ventricular filling

B

-TSH high; fT4 low -treated w/ thyroid hormone replacement -More common in autoimmune disease (A) iodine deficiency (B) primary hypothyroidism (C) secondary hypothyroidism (D) congenital hypothyroidism

C

-TSH low & disproportionately low fT4 -rare but may occur in patients with pituitary tumors (A) iodine deficiency (B) primary hypothyroidism (C) secondary hypothyroidism (D) congenital hypothyroidism

measures on the Denver development chart

-This thing tracks milestones according to age -Components include gross motor, language, fine motor, and personal-social -Gross motor milestone at age 2 includes standing on tiptoes -Language milestone at age 6 includes ability to define 7 words

B

-Tonic-clonic & partial seizures -Enhances binding of GABA to postsynaptic GABA-A receptors & increases the time that GABA-A-activated Cl- channels are open → inhibitory effect on repetitive neuronal activity in CNS pathways (A) Hydrantoins - Phenytoin (B) Barbiturates - Phenobarbital (C) Carbamazepine (D) Valproic Acid (E) Succinimides - Ethosuximide

things Haemophilus Influenzae type B would say

-Type of bacteria that causes different types of infections (mild ear infections → severe diseases) -I am Gram negative -There's a vaccine for me but you get one for "A" family member of mine every year -Spreads through respiratory droplets

tests to determine if a fetus is viable during pregnancy

-Ultrasound -Amniocentesis -uE3 (estriol unconjugated) -hCG

A

-Unclear MoA - most likely to do with binding w/ vesicle glycoprotein (SV2A) → limit vesicle fusion → reduce synaptic activity -Pregnancy Category C drug -Adjunct therapy (A) Levetiracetam (B) Lamotrigine (C) Topiramate (D) Zonisamide

A

-Used for tonic-clonic & partial seizures -binds reversibly to inactivated Na+ channels → reduces sustained high-frequency neuronal firing & delays neuronal recovery process (A) Hydrantoins - Phenytoin (B) Barbiturates - Phenobarbital (C) Carbamazepine (D) Valproic Acid (E) Succinimides - Ethosuximide

E

-Usually childhood -Abrupt, very short loss of consciousness -Minor muscular twitching -3-Hz spine-and-wave EEG pattern but not loss of postural control -Responsive to pharmacotherapy (A) Simple partial seizure (B) Complex partial seizure (C) Secondary Generalized seizure (D) Generalized tonic-clonic (grand mal) seizures (E) Petit Mal (Absence seizures)

C

-Ventricles contract -Ventricular P increases -Ventricles eject blood into arteries -Ventricular V decreases -Aortic valve opens -Ventricular V reaches maximum (A) Atrial Systole (B) Isovolumetric Ventricular Contraction (C) Rapid Ventricular Ejection (D) Reduced Ventricular Ejection (E) Isovolumetric Ventricular Relaxation (F) Rapid Ventricular Filling (G) Reduced Ventricular Filling or Diastasis

B

-Ventricles contract -Ventricular P increases -All valves closed -QRS complex (A) Atrial Systole (B) Isovolumetric Ventricular Contraction (C) Rapid Ventricular Ejection (D) Reduced Ventricular Ejection (E) Isovolumetric Ventricular Relaxation (F) Rapid Ventricular Filling (G) Reduced Ventricular Filling or Diastasis

E

-Ventricles relaxed -Ventricular pressure decreases -Ventricular V is constant (all valves closed) (A) Atrial Systole (B) Isovolumetric Ventricular Contraction (C) Rapid Ventricular Ejection (D) Reduced Ventricular Ejection (E) Isovolumetric Ventricular Relaxation (F) Rapid Ventricular Filling (G) Reduced Ventricular Filling or Diastasis

D

-Ventricular P < Atrial P → mitral (AV) valve opens → L ventricle fills with blood from L atrium (A) isovolumetric contraction (B) ventricular ejection (C) isovolumetric relaxation (D) ventricular filling

B

-Ventricular P > Aortic P → aortic valve opens -volume decreases as blood is ejected into aorta -width of pressure-volume loop = volume of blood ejected = stroke volume (A) isovolumetric contraction (B) ventricular ejection (C) isovolumetric relaxation (D) ventricular filling

A

-activates phagocytosis and neutralizes antigens -fetal circulation in pregnant women (A) IgG (B) IgM (C) IgA (D) IgD (E) IgE

D

-antigen receptor triggering initial B cell activation -found on surface of B lymphocytes (A) IgG (B) IgA (C) IgM (D) IgD (E) IgE

E

-dose-dependent inhibition of low-threshold Ca2+ currents carried by T-type Ca2+ channels -Used to treat absence seizures in children (A) Hydrantoins - Phenytoin (B) Barbiturates - Phenobarbital (C) Carbamazepine (D) Valproic Acid (E) Succinimides - Ethosuximide

A

-filters blood - removes defective RBCs & recycles Hb iron -Consists of splenic cords w/ macrophages & blood cells of all kinds & splenic sinusoids (A) red pulp (B) cortex (C) white pulp (D) splenic sinusoids

B

-first antibody produced in initial immune response; activates complement -B lymphocyte surface (A) IgG (B) IgM (C) IgA (D) IgD (E) IgE

D

-neural tube fails to close in region of defect -Oval raw area found on surface representing neural groove whose lips are fused -central canal discharges clear CSF onto surface (A) spina bifida occulta (B) meningocele (C) meningomyelocele (D) myelocele (E) Syringomyelocele

D

-surface of B lymphocytes -antigen receptor triggering initial B cell activation (A) IgG (B) IgM (C) IgA (D) IgD (E) IgE

A

-1st commissure to appear -interconnects the olfactory structures and the middle & inferior temporal gyri (A) anterior commisure (B) fornix (C) laminal terminalis (D) corpus callosum

D

-20% of all lamellar bone -interconnected thin spicules or trabecullae covered by endosteum -inner region of bones, adjacent to marrow cavities (A) woven bone (B) lamellar bone (C) compact (cortical) bone (D) cancellous (trabecular) bone

B

-2nd commissure to appear -interconnects the 2 hippocampi (A) anterior commisure (B) fornix (C) laminal terminalis (D) corpus callosum

D

-2nd major class of mitochondrial lipids, enriched in nerve & muscle tissue -May be more resistant to oxidative damage → may provide protection against oxidative stress in tissues with active aerobic metabolism (A) sphingolipids (B) glycerophospholipids (C) triacylglycerols (D) plasmalogens

D

-3rd commissure to appear -interconnects frontal lobes of both sides of parietal lobes (A) anterior commisure (B) fornix (C) laminal terminalis (D) corpus callosum

C

-80% of all lamellar bone -Parallel lamellae or densely packed osteons with interstitial lamellae -thick, outer region of bones (A) woven bone (B) lamellar bone (C) compact (cortical) bone (D) cancellous (trabecular) bone

D

-A dilation of ventricles owing to an excess of CSF -May result from blockage of CSF circulation (aqueductal stenosis) or overproduction of CSF (A) Holoprosencephaly (B) Fetal Alcohol Syndrome (C) Anencephaly (D) Hydrocephalus

B

-AKA sphingolipidosis -Deficient enzyme = lysosomal α-galactosidase -Accumulation = Gb3 -Skin rash, kidney failure, & pain in lower extremities (A) Tay-Sachs Disease (B) Fabry's Disease (C) Gaucher's Disease (D) Hunter's Disease

C

-Adhesion focus attaches cell to ECM → -Integrins associated with intermediate filaments link cytoskeleton to ECM (A) spot desmosome (B) tight junction (C) hemidesmosome (D) belt desmosome

D

-Alar plate derivative located inferior to thalamus & lateral to hypothalamus -Includes subthalamic nucleus, zona incerta, & lenticular & thalamic fasciculi -Contains neuroblasts that migrate into telencephalon to form pallidium (A) epithalamus (B) thalamus (C) hypothalamus (D) subthalamus

B

-Alar plate derivative that gives rise to thalamic nuclei -includes metathalamus w/ lateral & medial geniculate body (A) epithalamus (B) thalamus (C) hypothalamus (D) subthalamus

A

-Atria contract -Final phase of ventricular filling -P wave -S4 heart sound (A) Atrial Systole (B) Isovolumetric Ventricular Contraction (C) Rapid Ventricular Ejection (D) Reduced Ventricular Ejection (E) Isovolumetric Ventricular Relaxation (F) Rapid Ventricular Filling (G) Reduced Ventricular Filling or Diastasis

blood test abnormalities specific to Gaucher Disease

-Beta-glucosidase leukocyte test -Accumulation of glycolipid glucocerebroside -Sphingolipid accumulation in phagocytic cells

E

-Binds antigens & promotes release of vasoactive amines from mast cells -Major role in allergy/atopy & mediates anti-parasitic immunity (A) IgG (B) IgA (C) IgM (D) IgD (E) IgE

A

-Blindness, mental retardation, death b/w 2nd & 3rd year -Major storage product = GM2 ganglioside -Deficient enzyme = hexosaminidase A (A) Tay-sachs (B) Gaucher's (C) Fabry's (D) Krabbe's

B

-Blood flow to an organ is proportional to its metabolic activity -Example: metabolic activity in skeletal muscle increases b/c of strenuous exercise → blood flow to muscle will increase proportionately to meet metabolic demands (A) Autoregulation (B) Active hyperemia (C) Reactive hyperemia (D) Hormonal (extrinsic) control of blood flow

A

-Blood flow to an organ remains constant over a wide range of perfusion pressures -Heart, Brain, & Kidney -Example: perfusion P to heart suddenly decreased → compensatory vasodilation of arterioles to maintain constant flow (A) Autoregulation (B) Active hyperemia (C) Reactive hyperemia (D) Hormonal (extrinsic) control of blood flow

D

-Broad band-like adhesion focus b/w cells → -cadherins associated with actin filaments → cell shape & motility (A) spot desmosome (B) tight junction (C) hemidesmosome (D) belt desmosome

excitability

-Capacity of myocardial cells to generate action potentials in response to inward, depolarizing current -amount of inward current needed to bring a myocardial cell to threshold potential

C

-Causes arteriolar constriction -Released in response to blood vessel damage to help prevent blood loss -Implicated in vascular spasms of migraine headaches (A) Histamine (B) Bradykinin (C) Serotonin (5-hydroxytryptamine) (D) Prostaglandins

A

-Causes arteriolar dilation & venous constriction → increased Pc & increased filtration OUT of capillaries → local edema -released in response to tissue trauma (A) Histamine (B) Bradykinin (C) Serotonin (5-hydroxytryptamine) (D) Prostaglandins

A

1. Hepatitis A & E (viral infections) 2. Alcohol & Acetaminophen (toxicological causes) 3. Metabolic Syndrome The above are causes of (A) Acute hepatitis (B) Chronic hepatitis (C) Cholestatic disease (D) Hemochromatosis

B

1. Hepatitis B & C viruses 2. Alcohol 3. Immunologic diseases The above are causes of (A) Acute hepatitis (B) Chronic hepatitis (C) Cholestatic disease (D) Hemochromatosis

C

-Confined to the intravascular space -Helps eliminate circulating antigens & microorganisms (A) IgG (B) IgA (C) IgM (D) IgD (E) IgE

A

-Control cell shape & movement -Contraction in muscle cells when myosin binds (A) Actin (B) Vimentin (C) Cytokeratins (D) Microtubules

A

-Controlled by extrinsic sympathetic innervation of blood vessels in skeletal muscle AND by local metabolic factors -Important local vasodilators are lactate, adenosine, & K+ (A) skeletal muscle (B) pulmonary circulation (C) coronary circulation (D) cerebral circulation

congenital adrenal hyperplasia

-Cortisol deficiency → increased ACTH secretion -deficiency of 21-hydroxylase -ambiguous genitalia in females (excess 17-OHP → androgens) -"salt-losing crisis"

C

-Deficient enzyme = β-glucosidase (aka glucocerebrosidase) -Characterized by hepatomegaly & neurodegeneration -Recombinant replacement enzyme w/ N-glycan chains w/ terminal mannose residues (A) Tay-Sachs Disease (B) Fabry's Disease (C) Gaucher's Disease (D) Hunter's Disease

C

-Delays recovery of Na+ channels from inactivated state → inhibits Na+ influx in rapidly firing neurons -Monotherapy in adults w/ partial seizures -generalized seizures of Lennox Gastaut syndrome (A) Gabapentin (B) Pregabalin (C) Lamotrigine (D) Felbamate

A

-Depolarization of atria -Correlates with conduction time through atria (A) P wave (B) PR interval (C) QRS complex (D) T wave (E) QT interval

A

-Develops from embryonic roof plate & posterior aspects of alar plates -gives rise to pineal body -gives rise to tela choroidea & choroid plexus of 3rd ventricle from roof plate & pia mater (A) epithalamus (B) thalamus (C) hypothalamus (D) subthalamus

C

-Develops inferior to hypothalamic sulcus from alar plate & floor plates -Gives rise to hypothalamic nuclei including mamillary bodies & neurohypophysis (A) epithalamus (B) thalamus (C) hypothalamus (D) subthalamus

B

-Effector and regulatory cells for adaptive immunity -spherical nuclei w/ highly condensed chromatin (A) neutrophils (B) lymphocytes (C) eosinophils (D) basophils (E) monocytes

A

-Exerts osmotic force to retain fluid within microvasculature -Contributes to blood viscosity -Binds & transports some fatty acids, electrolytes, hormones, & drugs (A) albumin (B) alpha-globulins (C) beta-globulins (D) gamma-globulins (E) fibrinogen

D

-Exhibits autoregulation -Exhibits active & reactive hyperemia -Most important local metabolic factor is CO2 (A) skeletal muscle (B) pulmonary circulation (C) coronary circulation (D) cerebral circulation

C

-Exhibits autoregulation -Exhibits active & reactive hyperemia -Most important local metabolic factors are hypoxia & adenosine (A) skeletal muscle (B) pulmonary circulation (C) coronary circulation (D) cerebral circulation

D

-Family of homodimeric proteins -Induces angiogenesis by promoting endothelial cell migration & proliferation & formation of vascular lumina -Also induce vascular dilation & increase vascular permeability -Hypoxia induces production (A) EGF & TGF-α (B) HGF (C) PDGF (D) VEGFs (E) FGF (F) TGF-β

B

-Fast, neural mechanisms -Stretch receptors within the walls of the carotid sinus near the bifurcation of the common carotid arteries -Negative feedback system responsible for minute-to-minute regulation of Pa (A) Renin-Angiotensin II-Aldosterone System (B) Baroreceptors (C) Peripheral Chemoreceptors (D) Central Chemoreceptors

D

-Follows RRP -Starts when MP = -70 mV and ends when MP = -85 mV -Cell more excitable than normal because Na+ channels are recovered (A) Absolute Refractory Period (ARP) (B) Effective Refractory Period (ERP) (C) Relative Refractory Period (RRP) (D) Supranormal Period (SNP)

B

-Found in secretions -Presents an antiseptic barrier that protects mucosal surfaces -Major component of colostrum -Promotes phagocytosis, eosinophilic degranulation, & activates alternative complement pathway (A) IgG (B) IgA (C) IgM (D) IgD (E) IgE

A

-GABA analogue designed to cross blood brain barrier -Binding to L-type Ca2+ channel subunit → inhibits depolarizing high voltage-activating Ca2+ channel currents -Adjunct for refractory partial seizures -Treat chronic pain conditions (A) Gabapentin (B) Pregabalin (C) Lamotrigine (D) Felbamate

pediatric vaccines

-HBV -DTap -H Influenzae B -Rotavirus -Pneumococcal conjugate -Inactivated poliovirus

A

-Have agonist activity in tissues where estrogen action is desired & antagonist activity in tissues where estrogen activity may be harmful -Possible because of 2 distinct estrogen receptors (A) SERMs (B) Full Estrogen Receptor Antagonists (C) Estrogen Synthesis Inhibitors

descriptions of the obstetric history

-I can tell you if you've had a complicated pregnancy -I can tell you how your children were delivered -Gravida

C

-Iminostilbene derivative closely related to tricyclic antidepressants -Binds to inactivated Na+ channels → slows neuronal recovery after activation -Also reduced Ca2+ & Na+ flux across neuronal membrane (A) Hydrantoins - Phenytoin (B) Barbiturates - Phenobarbital (C) Carbamazepine (D) Valproic Acid (E) Succinimides - Ethosuximide

B

-Includes ARP -Na+ channels start to become unavailable to carry inward current -Conducted action potential cannot be generated (A) Absolute Refractory Period (ARP) (B) Effective Refractory Period (ERP) (C) Relative Refractory Period (RRP) (D) Supranormal Period (SNP)

C

1st antibody synthesized after an antigenic challenge (A) IgG (B) IgA (C) IgM (D) IgD (E) IgE

C

1st step in conversion of carbons into intermediates in glycolytic pathway, TCA cycle, or lipid metabolism is transfer of alpha-amino group by transamination to alpha-ketoglutarate or oxaloacetate but the EXCEPTION is (A) glycine (B) leucine (C) lysine (D) serine

C

2 principal steps in steroid hormone excretion in urine: 1. Removal of biological potency of steroid by series of ________ reactions 2. Steroid structure rendered ________ by conjugation to a glucuronide or sulfate (A) condensation, active (B) oxidation, hydrophobic (C) reduction, water-soluble (D) hydroxylation, inactive

1. OCS (open canicular system) 2. dense tubular system

2 systems of membrane channels in the hyalomere of platelets facilitates rapid exocytosis of proteins from platelets: (1) connected to invaginations of plasma membrane to facilitate platelet's uptake of factors from plasma (2) derived from ER and stores Ca2+ ions

C

20% of spleen - secondary lymphoid tissue associated w/ small central arterioles that are also enclosed by PALS (periarteriolar lymphoid sheaths) of T cells (A) red pulp (B) cortex (C) white pulp (D) splenic sinusoids

A

2nd PA may be added to phosphatidylglycerol → _____ (cardiolipin) found in inner mitochondrial membrane. It is required for efficient activity of electron transport complexes III & IV & ATP:ADP translocase (A) DPG (B) DHAP (C) PC (D) PE

B

3 of the 18 cytochrome P-450 gene families share the responsibility for _________ (A) drug synthesis (B) drug metabolism (C) fat metabolism (D) glucose synthesis

E

A 13-year-old boy has developed polydipsia, polyuria, polyphagia, and weight loss over the past few weeks. He was brought to the emergency room by his parents because he woke up this morning very lethargic. His blood glucose was found to be 600 mg/dL, and he was immediately placed on an insulin drip. Insulin works primarily by which one of the following mechanisms? (A) Activating adenylate cyclase (B) Binding to an intracellular receptor (C) Activating caspases (D) Producing cGMP (E) Causing phosphorylation of tyrosine residues

B

A 14-year-old patient presents in the nephrology clinic with fatigue, malaise, anorexia, abdominal pain, and fever. She reports a loss of 6 lb in the past 2 months. Serum gamma globulin and the immunoglobulins IgG, IgA, and IgM are all elevated. Her serum creatine is 1.4 mg/dL (normal 0.6-1.2 mg/dL) and urinalysis of glucose and protein are 2+ on a dipstick test, confirmed by laboratory at 8.0 g/dL and 0.95 g/dL, respectively. A renal biopsy is prepared for light microscopy, and an infiltrate containing lymphocytes, plasma cells, and eosinophils is found among tubules having cells with prominent brush borders. Which one of the following statements correctly pertains to these epithelial cells? (A) Impermeable to water despite presence of ADH (B) The primary site for the reduction of the tubular fluid volume (C) The site of the countercurrent multiplier (D) The site of action of aldosterone (E) Indirectly involved in the release of renin

C

A 21-year-old patient with a stab wound reveals a laceration of the right vagus nerve proximal to the origin of the recurrent laryngeal nerve. Which of the following conditions would most likely result from this lesion? (A) Contraction of bronchial muscle (B) Stimulation of bronchial gland secretion (C) Dilation of the bronchial lumen (D) Decrease in cardiac rate (E) Constriction of coronary artery

D

A 23-year-old woman is referred to an endocrinologist for weight gain, especially around the waist. She also has striae over the abdomen and a rounded appearance to her face. She is found to have Cushing disease. Which of the following would most likely be found in this patient, compared with someone who does not have Cushing disease? (A) Increased synthesis of immunoglobulins (B) Increased protein synthesis (C) Inhibition of lipolysis (D) Increased gluconeogenesis (E) Reduced liver glycogen stores

E

A 24-year-old woman presents to the emergency department with severe diarrhea. When she is supine (lying down), her blood pressure is 90/60 mm Hg (decreased) and her heart rate is 100 beats/min (increased). When she is moved to a standing position, her heart rate further increases to 120 beats/ min. Which of the following accounts for the further increase in heart rate upon standing? (A) Decreased total peripheral resistance (B) Increased venoconstriction (C) Increased contractility (D) Increased afterload (E) Decreased venous return

E

A 25-year-old man presents with persistent joint pain and a history of recurrent fractures of each humerus. His hematocrit and complete blood count (CBC) are normal, but blood calcium levels are high. Hormone levels are all within normal ranges except PTH that exceeds normal by threefold. Which if the following could be prescribed to offset the effects of the elevated PTH? (A) Vitamin D (B) Vitamin C (C) Recombinant RANK ligand (D) Somatotrophin (growth hormone) (E) Calcitonin

D

A 27-year-old cardiac patient with an irregular heartbeat visits her doctor's office for examination. Where should the physician place the stethoscope to listen to the sound of the mitral valve? (A) Over the medial end of the second left intercostal space (B) Over the medial end of the second right intercostal space (C) In the left fourth intercostal space at the midclavicular line (D) In the left fifth intercostal space at the midclavicular line (E) Over the right half of the lower end of the body of the sternum

D

A 27-year-old patient with Marfan syndrome has an aneurysm of the aortic arch. This may compress which of the following structures? (A) Right vagus nerve (B) Left phrenic nerve (C) Right sympathetic trunk (D) Left recurrent laryngeal nerve (E) Left greater splanchnic nerve

D

A 30-year-old female patient's electrocardiogram (ECG) shows two P waves preceding each QRS complex. The interpretation of this pattern is (A) decreased firing rate of the pacemaker in the sinoatrial (SA) node (B) decreased firing rate of the pacemaker in the atrioventricular (AV) node (C) increased firing rate of the pacemaker in the SA node (D) decreased conduction through the AV node (E) increased conduction through the His- Purkinje system

C

A 32-year-old patient who weighs 275 lb comes to the doctor's office. On the surface of the chest, the physician is able to locate the apex of the heart: (A) At the level of the sternal angle (B) In the left fourth intercostal space (C) In the left fifth intercostal space (D) In the right fifth intercostal space (E) At the level of the xiphoid process of the sternum

C

A 35-year-old woman's physician orders laboratory blood tests. Her fresh blood is drawn and centrifuged in the presence of heparin as an anticoagulant to obtain a hematocrit. From top to bottom, the fractions resulting from centrifugation are which of the following? (A) Serum, packed erythrocytes, and leukocytes (B) Leukocytes, erythrocytes, and serum proteins (C) Plasma, buffy coat, and packed erythrocytes (D) Fibrinogen, platelets, buffy coat, and erythrocytes (E) Albumin, plasma lipoproteins, and erythrocytes

D

A 37-year-old man is brought to the emergency department complaining of severe chest pain. His angiogram reveals thromboses of both brachiocephalic veins just before entering the superior vena cava. This condition would most likely cause a dilation of which of the following veins? (A) Azygos (B) Hemiazygos (C) Right superior intercostal (D) Left superior intercostal (E) Internal thoracic

D

A 37-year-old patient with palpitation was examined by her physician, and one of the diagnostic records included a posterior-anterior chest radiograph. Which of the following comprises the largest portion of the sternocostal surface of the heart seen on the radiograph? (A) Left atrium (B) Right atrium (C) Left ventricle (D) Right ventricle (E) Base of the heart

B

A 37-year-old patient with severe chest pain, shortness of breath, and congestive heart failure was admitted to a local hospital. His coronary angiograms reveal a thrombosis in the circumflex branch of the left coronary artery. Which of the following conditions could result from the blockage of blood flow in the circumflex branch? (A) Tricuspid valve insufficiency (B) Mitral valve insufficiency (C) Ischemia of AV node (D) Paralysis of pectinate muscle (E) Necrosis of septomarginal trabecula

B

A 42-year-old woman, who has been a type I diabetic for 30 years, falls when she trips over the vacuum cleaner hose. She tried to break her fall by placing her hand out to save herself and in the process her wrist was forced backward, breaking her radius near the wrist. Which of the following is produced in the first step in healing this bone injury? (A) Osteoid (B) Hematoma (C) Bony callus (D) Fibrocartilage (E) Compact bone

E

A 43-year-old anatomy professor is working in her garden, pruning rose bushes without gloves, when a thorn deeply penetrates her forefinger. The next day the area has become infected. She removes the tip of the thorn, but there is still pus remaining at the wound site. Which of the following cells function in the formation of pus? (A) Cells with spherical nuclei and scant cytoplasm (B) Biconcave cells with no nuclei (C) Cells with bilobed nuclei and many acidophilic cytoplasmic granules (D) Very small, cell-like elements with no nuclei but many granules (E) Cells with polymorphic, multiply lobed nuclei

D

A 43-year-old man comes to the emergency room with a headache and blurred vision. He complains that his wedding ring no longer fits him, and that his favorite hat no longer fits on his head. His wife feels that his nose has become wider, and he is diagnosed with acromegaly. Which of the following metabolic effects would you expect in this patient? (A) Decreased protein synthesis (B) Inhibition of gluconeogenesis (C) Inhibition of lipolysis (D) Increased protein synthesis (E) Gigantism

A

A 43-year-old man presents to the neurologist with headache and double vision. He also complains of a milky discharge from his breast. An MRI of his head confirms the diagnosis of a prolactinoma. Which of the following substances could inhibit the release of prolactin from the pituitary? (A) Dopamine (B) Caffeine (C) Endorphins (D) Renin (E) PGF2a

C

A 44-year-old man with a stab wound was brought to the emergency department, and a physician found that the patient was suffering from a laceration of his right phrenic nerve. Which of the following conditions has likely occurred? (A) Injury to only GSE fibers (B) Difficulty in expiration (C) Loss of sensation in the fibrous pericardium and mediastinal pleura (D) Normal function of the diaphragm (E) Loss of sensation in the costal part of the diaphragm

E

A 45-year-old man presents with nephrolithiasis or kidney stones. The process of calcium oxalate stone formation as seen in this patient begins with Randall plaques found in the basement membranes of which one of the following structures found only in the renal medulla? (A) Proximal convoluted tubules (B) Distal convoluted tubules (C) Thin loops of Henle (D) Afferent arterioles (E) Collecting ducts

B

A 45-year-old woman presents with a tumor confined to the posterior mediastinum. This could result in compression of which of the following structures? (A) Trachea (B) Descending aorta (C) Arch of the aorta (D) Arch of the azygos vein (E) Phrenic nerve

B

A 46-year-old patient comes to his doctor's office and complains of chest pain and headache. His computed tomography (CT) scan reveals a tumor located just superior to the root of the right lung. Blood flow in which of the following veins is most likely blocked by this tumor? (A) Hemiazygos vein (B) Arch of the azygos vein (C) Right subclavian vein (D) Right brachiocephalic vein (E) Accessory hemiazygos vein

C

A 46-year-old woman presents with pain in the left leg that worsens on weight bearing. An x-ray shows demineralization, and a bone biopsy decalcified with EDTA shows reduction in bone quantity. The patient had undergone menopause at age 45 without estrogen replacement. She reports long-standing diarrhea. In addition, laboratory tests show low levels of vitamin D, calcium, and phosphorus, and elevated alkaline phosphatase. A second bone biopsy was taken but not decalcified, which showed extensive deposition of uncalcified osteoid on all the bone surfaces. On the basis of these data, the best diagnosis would be which of the following? (A) Osteoporosis (B) Scurvy (C) Osteomalacia (D) Rickets (E) Hypoparathyroidism

C

A 47-year-old man with a known atrial fibrillation returns to see his cardiologist for follow-up of his cardiac health. The right atrium is important in this case because it: (A) Receives blood from the oblique cardiac vein (B) Is associated with the apex of the heart (C) Contains the SA node (D) Receives the right pulmonary vein (E) Is hypertrophied by pulmonary stenosis

D

A 5-year-old girl is brought to the emergency department because of difficulty breathing (dyspnea), palpitations, and shortness of breath. Doppler study of the heart reveals an atrial septal defect (ASD). This malformation usually results from incomplete closure of which of the following embryonic structures? (A) Ductus arteriosus (B) Ductus venosus (C) Sinus venarum (D) Foramen ovale (E) Truncus arteriosus

C

A 50-year-old woman complains of feeling warm all of the time. Her eyes appear as though they are ''bulging out of their sockets'' (proptosis). She sees a family physician to evaluate her condition. Laboratory tests demonstrate a decreased level of TSH. Which of the following would you expect in this patient? (A) Reduced blood pressure (B) Weight gain (C) Increased basal metabolic rate (D) Reduced heart rate (E) Excess sleep

D

A 53-year-old woman is found, by arteriography, to have 50% narrowing of her left renal artery. What is the expected change in blood flow through the stenotic artery? (A) Decrease to 1⁄2 (B) Decrease to 1⁄4 (C) Decrease to 1⁄8 (D) Decrease to 1⁄16 (E) No change

D

A 54-year-old man presents with recurrent breathlessness and chronic fatigue. After routine tests followed by a bone marrow biopsy he is diagnosed with lymphocytic leukemia. Chemotherapy is administered to remove the cancerous cells, which also destroys the precursor cells of erythrocytes. To reestablish the erythrocytic lineage, which of the following cells should be transplanted? (A) Reticulocytes (B) Orthochromatophilic erythroblasts (C) Megakaryoblasts (D) Basophilic erythroblasts (E) Metamyelocytes

C

A 54-year-old patient is implanted with an artificial cardiac pacemaker. Which of the following conductive tissues of the heart had a defective function that required the pacemaker? (A) Atrioventricular (AV) bundle (B) AV node (C) Sinoatrial (SA) node (D) Purkinje fiber (E) Moderator band

C

A 56-year-old patient recently suffered a myocardial infarction in the area of the apex of the heart. The occlusion by atherosclerosis is in which of the following arteries? (A) Marginal artery (B) Right coronary artery at its origin (C) Anterior interventricular artery (D) Posterior interventricular artery (E) Circumflex branch of the left coronary artery

B

A 56-year-old woman with a 60-pack year history of smoking is recently found to have a large neoplastic pulmonary mass. Laboratory tests demonstrate a sodium of 127 mmol/L (normal, 135 to 145 mmol/L) and reduced urine osmolality. She likely has which of the following endocrine abnormalities? (A) Cushing disease (B) SIADH (C) Cushing syndrome (D) Acromegaly (E) Prolactinoma

C

A 56-year-old woman with no known medical conditions presents to the emergency room with pain in the upper arm. She denies any trauma; however, a fracture of the humerus is found on radiograph. She is found to have an elevated PTH level. Which of the following statements best describes PTH? (A) It lowers serum calcium (B) It directly promotes the absorption of calcium from the intestine (C) It stimulates the conversion of vitamin D to the active form. (D) It promotes the reabsorption of phosphate from the kidney. (E) It promotes the excretion of calcium from the kidney.

C

A 57-year-old patient has a heart murmur resulting from the inability to maintain constant tension on the cusps of the AV valve. Which of the following structures is most likely damaged? (A) Crista terminalis (B) Septomarginal trabecula (C) Chordae tendineae (D) Pectinate muscle (E) Anulus fibrosus

E

A 6-month-old girl was seen by the plastic surgeon because of the presence of a swelling at the root of the nose. The mother said that she had noticed the swelling when the child was born and that since then, it had gradually increased in size. The neurosurgeon was consulted, and the following possible additional findings were ascertained except: (A) A lateral radiograph of the skull revealed a defect in the membranous bones involving the nasal process of the frontal bone. (B) The defect in the membranous bones is known as cranioschisis. (C) The condition was associated with a cephalic meningocele. (D) There was a herniation of the meninges through the defect in the skull. (E) Brain tissue is never found within the hernia.

E

A 6-month-old girl was seen by the plastic surgeon because of the presence of a swelling at the root of the nose. The mother said that she had noticed the swelling when the child was born and that since then, it had gradually increased in size. The surgeon examined the child and found the following likely signs except: (A) The swelling was situated at the root of the nose in the midline. (B) The swelling was located between the frontal and nasal bones. (C) The swelling was fluctuant and, on gentle pressure, could be reduced in size. (D) The swelling was pulsatile, and the pulse coincided with the heart rate. (E) The pulse did not coincide with the pulse felt over the anterior fontanelle of the skull

B

A 6-year-old boy is brought to the clinic where his mother reports that he was bitten by a neighbor's dog two days earlier. The child's right hand is lacerated between the thumb and index finger and this area is inflamed but healing. The doctor's examination reveals small but painless swellings beneath the skin inside the right elbow and arm pit and he explains to the mother that these are active lymph nodes enlarged in response to the infection in the hand. What has produced the swelling? (A) Increased flow of lymph through the nodes' afferent lymphatics (B) Formation of germinal centers for B-cell proliferation in each node's cortex (C) Arrival of antigen-presenting cells in each node's medulla (D) Enlargement and increased activity of the nodes' high endothelial venules (E) Increased thickness of each node's paracortex

D

A 60-year-old businessman is evaluated by his physician, who determines that his blood pressure is significantly elevated at 185/130 mm Hg. Laboratory tests reveal an increase in plasma renin activity, plasma aldosterone level, and left renal vein renin level. His right renal vein renin level is decreased. What is the most likely cause of the patient's HTN? (A) Aldosterone-secreting tumor (B) Adrenal adenoma secreting aldosterone and cortisol (C) Pheochromocytoma (D) Left renal artery stenosis (E) Right renal artery stenosis

D

A 62-year-old patient with pericardial effusion comes to a local hospital for aspiration of pericardial fluid by pericardiocentesis. The needle is inserted into the pericardial cavity through which of the following intercostal spaces adjacent to the sternum? (A) Right fourth intercostal space (B) Left fourth intercostal space (C) Right fifth intercostal space (D) Left fifth intercostal space (E) Right sixth intercostal space

D

A 66-year-old man, who has had a sympathectomy, experiences a greater- than-normal fall in arterial pressure upon standing up. The explanation for this occurrence is (A) an exaggerated response of the renin- angiotensin-aldosterone system (B) a suppressed response of the renin- angiotensin-aldosterone system (C) an exaggerated response of the baroreceptor mechanism (D) a suppressed response of the baroreceptor mechanism

B

A 72-year-old woman, who is being treated with propranolol, finds that she cannot maintain her previous exercise routine. Her physician explains that the drug has reduced her cardiac output. Blockade of which receptor is responsible for the decrease in cardiac output? (A) α1 Receptors (B) β1 Receptors (C) β2 Receptors (D) Muscarinic receptors (E) Nicotinic receptors

C

A 73-year-old woman is admitted to the intensive care unit for septic shock from a urinary tract infection. The critical care fellow is concerned she may not have an appropriate stress response and orders a cosyntropin test. Which hormone does this test evaluate? (A) Oxytocin (B) Vasopressin (C) Cortisol (D) Corticotropin-releasing hormone (CRH) (E) Adrenocorticotropic hormone (ACTH)

E

A 75-year-old man complains of increased urinary frequency, especially at night. He has difficulty starting his stream (hesitancy) and often dribbles urine when he finishes. His urologist suspects BPH and places him on a 5-alpha-reductase inhibitor. This drug would decrease which of the following? (A) Conversion of cAMP to AMP (B) Release of calcium from the endoplasmic reticulum (C) Prostaglandin synthesis (D) Conversion of angiotensin I to angiotensin II (E) Conversion of testosterone to DHT

E

A 75-year-old woman with osteoporosis complains of back pain. A computed tomographic scan of her back confirms a compression fracture of the L3 vertebra. The attending physician begins treating the patient with morphine for pain control. Morphine is an analgesic that works similarly to which of the following endogenously produced substances? (A) ACTH (B) POMC (C) Lipotropin (D) MSH (E) Endorphin

D

A 9-month-old girl was admitted to the children's hospital with tachypnea (fast breathing) and shortness of breath. Physical examination further exhibits tachycardia (fast heart rate), a bounding peripheral pulse, and her angiographs reveal a patent ductus arteriosus. Which of the following embryonic arterial structures is most likely responsible for the origin of the patent ductus arteriosus? (A) Right fourth arch (B) Left fifth arch (C) Right fifth arch (D) Left sixth arch (E) Right sixth arch

A

A _____ is an enzyme found predominantly in the liver with lesser quantities in the kidneys, heart, & skeletal muscle used to diagnose liver disease (A) ALT (Alanine Transaminase) (B) AST (Aspartate Transaminase) (C) Alkaline phosphatase (D) ALP (Alkaline Phosphatase)

B

A _______ is used for the detection of specific proteins in complex samples (A) southern blot (B) western blot (C) eastern blot (D) northern blot

E

A baby is born with a cleft palate and a condition called DiGeorge syndrome, which involves failure of third and fourth pharyngeal pouch derivatives to develop properly. The palate defect is corrected surgically, but regarding the pharyngeal pouch defect the parents are advised that the growing child may expect which of the following health problems? (A) Insufficient B-cell production by lymph nodes in the head and neck (B) Inability to secrete IgA (C) Excessive numbers of circulating but defective erythrocytes (D) Increased oral infections due lack of palatine and pharyngeal tonsils (E) Conditions related to autoimmunity

lymphoblastic, myelogenous

A bone marrow aspiration is performed on a patient, in which a bone marrow sample is taken and immunocytochemistry with labeled monoclonal antibodies specific to membrane proteins of precursor blood cells is performed. Malignant clones of leukocyte precursors were found in his lymphoid tissue, and he was diagnosed with ________ leukemia. If malignant clones had been found in his bone marrow, he would have been diagnosed with _______ leukemia.

D

A cardiologist is on clinical rounds with her medical students. She asks them, "During the cardiac cycle, which of the following events occurs?" (A) AV valves close during diastole (B) Aortic valve closes during systole (C) Pulmonary valve opens during diastole (D) Blood flow in coronary arteries is maximal during diastole (E) Aortic valve closes at the same time as AV valve

D

A child is diagnosed with VSD b/c of failure of membranous IV septum to develop, which has lead to left-to-right shunting of the blood through the IV foramen and an increase in blood flow to the lungs which causes ________ ________ (A) left ventricle hypertrophy (B) arterial hypertension (C) pneumothorax (D) pulmonary hypertension (E) slowed heart rate

D

A child presenting with a cleft spinal cord due to failure of the neural folds to close would be diagnosed with what? (A) Spina bifida occulta (B) Meningocele (C) Myelomeningocele (D) Myeloschisis (E) Syringomyelocele

A

A healthy 35-year-old man is running a marathon. During the run, there is a increase in his splanchnic vascular resistance. Which receptor is responsible for the increased resistance? (A) α1 Receptors (B) β1 Receptors (C) β2 Receptors (D) Muscarinic receptors

D

A hematologist diagnoses a 34-year-old woman with idiopathic thrombocytic purpura (ITP). Which of the following symptoms/characteristics would one expect in this patient? (A) Normal blood count (B) Hypercoagulation (C) Decreased clotting time (D) Abnormal bruising (E) Light menstrual periods

B

A large number of immature neutrophils (band cells) is known as "shift to the left" and is a sign of (A) leukemia (B) bacterial infection (C) anemia (D) liver failure

D

A major mechanism for changing blood flow in the cardiovascular system is by changing ______ of blood vessels, mainly _______ (A) pressure gradient; arteries (B) pressure gradient; venules (C) resistance; venules (D) resistance; arterioles

A

A mother who failed to take enough folic acid during pregnancy gave birth to a child with a bony defect only with a small tuft of hair over the affected area of the skin because his vertebral arch didn't fuse during development. What would this child be diagnosed with? (A) Spina bifida occulta (B) Meningocele (C) Myelomeningocele (D) Myeloschisis (E) Syringomyelocele

C

A mother with diabetes gives birth to a baby who is diagnosed as having dextroposition of the aorta and the pulmonary trunk with cyanosis and shortness of breath. Which of the following structures is required to remain patent until surgical correction of the deformity? (A) Umbilical arteries (B) Umbilical vein (C) Ductus arteriosus (D) Ductus venosus (E) Sinus venosus

A

A neonate appears severely cyanotic and breathing rapidly. Cardiac echocardiogram reveals that the aorta lies to the right of the pulmonary trunk. Which of the following is most likely occurred during development? (A) AP septum failed to develop in a spiral fashion (B) Excessive resorption of septum primum (C) Pulmonary valve atresia (D) Persistent truncus arteriosus (E) Coarctation of the aorta

C

A newborn baby is readmitted to the hospital with hypoxia and upon testing is found to have pulmonary stenosis, dextraposition of the aorta, interventricular septal defect, and hypertrophy of the right ventricle. Which of the following is best described by these symptoms? (A) ASD (B) Patent ductus arteriosus (C) Tetralogy of Fallot (D) Aortic stenosis (E) Coarctation of the aorta

C

A newborn has multiple congenital defects owing to dysgenesis of the neural crest. Which of the following cells is most likely to be spared? (A) Geniculate ganglion cells (B) Melanocytes (C) Motor neurons (D) Parafollicular cells (E) Spinal ganglion cells

A

A patient has a small but solid tumor in the mediastinum, which is confined at the level of the sternal angle. Which of the following structures would most likely be found at this level? (A) Bifurcation of the trachea (B) Beginning of the ascending aorta (C) Middle of the aortic arch (D) Articulation of the third rib with the sternum (E) Superior border of the superior mediastinum

B

A person is deemed to have status epilepticus if they have repeated or continued seizures > _____ or lack of return to baseline after a single epileptic seizure (A) 2 minutes (B) 5 minutes (C) 10 minutes (D) 20 minutes

B

A person's electrocardiogram (ECG) has no P wave, but has a normal QRS complex and a normal T wave. Therefore, his pacemaker is located in the (A) sinoatrial (SA) node (B) atrioventricular (AV) node (C) bundle of His (D) Purkinje system (E) ventricular muscle

B

A phenyl ring at R1 or R2 (phenytoin) of the common structure of anticonvulsants makes it ideal for treating _______ epilepsy, but an alkyl substitution at R1 or R2 make it better at treating _____ seizures (A) partial, absence (B) tonic-clonic, absence (C) absence, tonic-clonic (D) tonic-clonic, partial

A

A sudden, brief loss of muscle tone, which is refractory to pharmacotherapy is known as _____ (A) Atonic ("Drop Attacks") (B) Status Epilepticus (C) Myoclonic Seizures (D) Simple partial seizures

C

A young toddler presents to her pediatrician with rather new onset of bowel and bladder dysfunction and loss of the lower limb function. Her mother had not taken enough folic acid (to the point of a deficiency) during her pregnancy. On examination, the child has protrusion of the spinal cord and meninges and is diagnosed with which of the following conditions? (A) Spina bifida occulta (B) Meningocele (C) Myelomeningocele (D) Myeloschisis (E) Syringomyelocele

increase, harder

A(n) (decrease/increase) in total peripheral resistance (TPR) leads to COUNTERclockwise rotation of the vascular function curve and blood flow back to heart is made (harder/easier)

decrease, easier

A(n) (decrease/increase) in total peripheral resistance (TPR) leads to clockwise rotation of the vascular function curve and blood flow back to heart is made (harder/easier)

C

ACE inhibitors (e.g. captoPRIL) (A) block action of angiotensin II at its receptor → decrease BP (B) block conversion of angiotensinogen to angiotensin I → decrease BP (C) block conversion of angiotensin I to angiotensin II → decrease BP (D) block synthesis & secretion of aldosterone by adrenal cortex → decrease BP

D

ACTH insufficiency may be due to primary ______ pathology or secondary to _____ failure to produce ACTH (A) anterior pituitary, adrenal (B) posterior pituitary, adrenal (C) adrenal, posterior pituitary (D) adrenal, anterior pituitary

decreases, increases

ACh (increases/decreases) inward Ca2+ current during plateau ACh (increases/decreases) IK-ACh → shortens duration of action potential

A

Abnormally rapid breathing (A) tachypnea (B) dyspnea (C) orthopnea (D) apnea

B

Abrupt withdrawal or major reduction of glucocorticoid dosage → (A) chronic adrenal insufficiency (B) acute adrenal insufficiency (C) Cushing disease (D) overactive adrenal gland

proportional to, proportional to, C

According to the Poiseuille equation, resistance to flow is 1. (proportional to/inversely proportional to) to viscosity of blood 2. (proportional to/inversely proportional to) length of the blood vessel 3. proportional to ________ of the radius of the blood vessel (A) 1/2 power (B) 1/3rd power (C) 1/4th power (D) 1/8th power

C

Acetyl-CoA is supplied by: 1.Beta-oxidation of ________________ 2.Dehydrogenation of ____________ 3.__________ of ketogenic AAs (A) proteins, glucose, oxidation (B) glucose, proteins, reduction (C) long-chain fatty acids, pyruvate, oxidation (D) cholesterol, glucose, reduction

increases, decreases

Acetylation (associated w/ Lys residues) (decreases/increases) transcription while deacetylation (decreases/increases) transcription

B

Acetylcholinesterase (AChE) is primarily active in the CNS and is normally NOT found in amniotic fluid, unless contributed by the fetus as a result of (A) anemia (B) open NTD (C) closed NTD (D) respiratory distress syndrome (E) down syndrome

decrease, decrease, decrease, decrease, increase, increase

Acute hemorrhage → (increase/decrease) in Pa → (increase/decrease) stretch on carotid sinus baroreceptors → (increase/decrease) firing rate of carotid sinus nerve (Hering's nerve) → (increase/decrease) in parasympathetic (vagal) outflow to heart & (increase/decrease) in sympathetic outflow to heart & blood vessels → (increase/decrease) in Pa

D

Acute pericarditis → inflammation → accumulation of fluid in pericardial space → compresses heart → inhibits cardiac filling (A) MI (B) Cardiac tamponade (C) Pericarditis (D) Pericardial Effusion (E) Pericardiocentesis

A

Add a single dose of additional steroid, either by increasing oral dose or providing IM/IV steroids perioperatively w/ no taper (A) Dexamethasone 6 to 10 mg most commonly used in dentistry (B) Methyprednisolone dose pack most commonly used in dentistry (C) By far most common choice when it comes to steroid supplementation (or lack thereof) in dentistry (D) Cortisol most commonly used in dentistry

androgens, corticosteroids

Adrenal cortex using cholesterol as substrate secretes 2 types of steroid hormones: 1. 19-C _______ 2. 21-C ________

cortisol

Adrenal insufficiency may result from genetic conditions caused by defects in steroid biosynthesis, such as Congenital Adrenal Hyperplasia (CAH), which leads to a deficiency in ______ → increased ACTH secretion by pituitary → adrenal hyperplasia & rise in steroid precursors

B

Adult remnant of the ductus arteriosus is the _____________ (A) Medial umbilical ligament (B) Ligamentum arteriosum (C) Ligamentum venosum (D) Oval fossa (E) Round ligament of the liver

D

After macula densa, tubule continues as ________, where electrolyte levels of filtrate are adjusted further controlled by aldosterone (A) TAL (B) connecting tubules (C) Loop of Henle (D) DCT

progesterone

After successful implantation of a fertilized ovum, maintenance of corpus luteum and _______ production is vital to ensure progression of development

A

Agents blocking effects of progesterone are mainly potent (A) competitive antagonists of progesterone receptor (B) agonists of progesterone receptor (C) partial agonists of progesterone receptor (D) all of the above

E

Agranulocyte that participates in specific immune response (A) neutrophils (B) basophils (C) eosinophils (D) B lymphocytes (E) T lymphocytes (F) macrophages

D

Agranulocyte that synthesizes antibodies (A) neutrophils (B) basophils (C) eosinophils (D) B lymphocytes (E) T lymphocytes (F) macrophages

adaptive immunity

Aimed at specific microbial invaders, mediated by lymphocytes and APCs

B

Alar plate sensory neuroblasts of the ____ give rise to pontine nuclei, which consist of cerebellar relay nuclei (A) cerebellum (B) pons (C) medulla oblongata (D) forebrain

D

Alcohol excess is a major cause of liver disease given that ethanol oxidation alters the ________ of the hepatocyte → excessive fat deposition in liver (alcoholic steatosis) → hepatitis → fibrosis (cirrhosis) → liver failure (A) gene sequence (B) enyzymatic activity (C) chemistry (D) redox potential

D

An 83-year-old man with a typical coronary circulation has been suffering from an embolism of the circumflex branch of the left coronary artery. This condition would result in ischemia of which of the following areas of the heart? (A) Anterior part of the left ventricle (B) Anterior interventricular region (C) Posterior interventricular region (D) Posterior part of the left ventricle (E) Anterior part of the right ventricle

C

An ______ provides information on the rhythm and rate of the heart by testing the electrical conduction system of the heart. The _________ is an ultrasound scan of the heart that shows moving pictures that show the structure and function of the heart (A) electrocardiogram (EKG); ABG (B) ABG; electrocardiogram (EKG) (C) electrocardiogram (EKG); echocardiogram (D) echocardiogram, electrocardiogram (EKG)

A

An acute decrease in arterial blood pressure elicits which of the following compensatory changes? (A) Decreased firing rate of the carotid sinus nerve (B) Increased parasympathetic outflow to the heart (C) Decreased heart rate (D) Decreased contractility (E) Decreased mean systemic pressure

A

An electrocardiogram (ECG) on a person shows ventricular extrasystoles. After an extrasystole, the next "normal" ventricular contraction produces (A) increased pulse pressure because the contractility of the ventricle is increased (B) increased pulse pressure because total peripheral resistance (TPR) is decreased (C) increased pulse pressure because compliance of the veins is decreased (D) decreased pulse pressure because the contractility of the ventricle is increased (E) decreased pulse pressure because TPR is decreased

D

An electrocardiogram (ECG) on a person shows ventricular extrasystoles. The extrasystolic beat would produce (A) increased pulse pressure because contractility is increased (B) increased pulse pressure because heart rate is increased (C) decreased pulse pressure because ventricular filling time is increased (D) decreased pulse pressure because stroke volume is decreased (E) decreased pulse pressure because the PR interval is increased

C

An electrocardiogram measures potential differences on the surface of the body reflecting electrical activity of the heart which is measurable because of ______ and _______ of depolarization & repolarization (A) timing and intensity (B) intensity and sequence (C) timing and sequence (D) intensity and frequency

A

An increase in contractility is demonstrated on a Frank-Starling diagram by (A) increased cardiac output for a given end- diastolic volume (B) increased cardiac output for a given end- systolic volume (C) decreased cardiac output for a given end-diastolic volume (D) decreased cardiac output for a given end-systolic volume

A, B, C, F

Angiotensin II has which of the following effects (select all that apply) (A) stimulate synthesis & secretion of aldosterone by adrenal cortex (B) Increases Na+-H+ exchange in proximal convoluted tubule (C) Increases thirst (D) decreases urination (E) Vasodilation (F) Vasoconstriction

A

Angiotensin receptor (AT1) agonists (e.g. losartan) (A) block action of angiotensin II at its receptor → decrease BP (B) block conversion of angiotensinogen to angiotensin I → decrease BP (C) block conversion of angiotensin I to angiotensin II → decrease BP (D) block synthesis & secretion of aldosterone by adrenal cortex → decrease BP

D

Antibodies with various immune functions secreted by plasma cells (A) albumin (B) alpha-globulins (C) beta-globulins (D) gamma-globulins (E) fibrinogen

A

Antiseizure drugs work through different MoAs to _______ underlying seizure generation (A) restore balance between excitatory & inhibitory drive (B) restore mutated gene back to normal (C) turn off excitatory drive only (D) turn off inhibitory drive only

A

Baby boy Murphy had weak patellar tendon reflexes bilaterally, corresponding to motor deficits in ______ (A) L2-L4 (B) S1-S2 (C) S2-S4 (D) L1-L3 (E) S1-S3

B

Baby boy murphy had bilateral absence of calcaneal reflex corresponding to motor deficits in ________ (A) L2-L4 (B) S1-S2 (C) S2-S4 (D) L1-L3 (E) S1-S3

A

Baby girl Weisen could have dudodenoduodenostomy to fix her duodental atresia first to allow time for heart growth so the surgery would only need to be performed once and because of the nature of the septal defect (AVSD), which allowed for normal O2 levels because (A) AVSD → mixing venous blood (B) AVSD → blocks flow from left to right side of the heart (C) AVSD → hyperventilation (D) AVSD → left sided hypertrophy

adhesins

Bacteria have evolved lectins called _____ that recognize & interact w/ specific carbohydrate structures on glycolipids, glycoproteins, & proteoglycans

A

Base pairing between single-stranded miRNA & its target mRNA directs _____ to either cleave mRNA target or repress its translation (A) RISC (B) Dicer (C) XIST (D) CRISPR

C

Begins as simple and/or complex partial seizure → activity spreads to entire cortex, thalamus, & midbrain → tonic-clonic seizure (A) Simple partial seizure (B) Complex partial seizure (C) Secondary Generalized seizure (D) Generalized tonic-clonic (grand mal) seizures (E) Petit Mal (Absence seizures)

B

Bile acids assist the digestion of dietary _______ and recirculate to the ______ (A) protein, intestine (B) fat, liver (C) carbohydrates, stomach (D) protein, stomach

B

Bilirubin is metabolized by the ________ & excreted in _______ (A) kidney, urine (B) hepatocytes, bile (C) liver, kidney (D) gallbladder, urine

B

Bind MHC class I → (in presence of IL-2 from Th cells) activated to proliferate → attaches to cell sources of antigens & release perforins & granzymes to trigger apoptosis (cell-mediated immunity) (A) CD4+ T helper cells (B) CD8+ cytotoxic T cells (C) CD4+CD25+ regulatory T cells (D) γδ T cells

A

Bind MHC class II → activated to produce cytokines that promote differentiation of B cells into plasma cells; activate macrophages to become phagocytic; activate cytotoxic T cells; & induce many parts of inflammatory reaction (A) CD4+ T helper cells (B) CD8+ cytotoxic T cells (C) CD4+CD25+ regulatory T cells (D) γδ T cells

A

Biosynthesis of _______ (main glucocorticoid) depends on stimulation by pituitary adrenocorticotropic hormone (ACTH) (A) cortisol (B) aldosterone (C) prednisolone (D) estrogen

C

Blood filtration in the open circulation involves interaction w/ splenic cord _______, which remove old, swollen RBCs unable to slip b/w stave cells to reenter the venous blood flow (A) monocytes (B) B lymphocytes (C) macrophages (D) T lymphocytes

closed, open

Blood flow in red pulp is either a(n): 1.(open/closed) circulation - moving from capillaries into venous sinusoids 2. (open/closed) circulation - capillaries open directly into splenic cords

A

Blood flow to which organ is controlled primarily by the sympathetic nervous system rather than by local metabolites? (A) Skin (B) Heart (C) Brain (D) Skeletal muscle during exercise

azygous

Blood from the pericardium drains into the _______ system of veins + the internal thoracic & superior phrenic veins

E

Blood in the renal arcuate arteries flows next into which vessels? (A) Afferent arterioles (B) Efferent arterioles (C) Glomerular capillaries (D) Interlobar arteries (E) Interlobular arteries

C

Bone repair after fracture or other injury involves activation of periosteal fibroblasts to produce an initial soft callus of ________, which is gradually replaced by a hard callus of _________ that is soon remodeled to produce stronger lamellar bone (A) woven bone, compact bone (B) fibrocartilage-like tissue, compact bone (C) fibrocartilage-like tissue, woven bone (D) woven bone, fibrocartilage-like tissue

lamellar, woven

Both compact & cancellous bone show mature ________ bone (has discrete sheets of matrix) & _________ bone (newly formed with randomly arranged components)

C

Brain swelling on x-rays and stiff neck would be signs of (A) Anencephaly (B) Spina bifida (C) Meningitis (D) Leukemia

C

By far most common choice when it comes to steroid supplementation (or lack thereof) in dentistry (A) Add a single dose of additional steroid, either by increasing oral dose or providing IM/IV steroids perioperatively w/ no taper (B) Increase preoperative steroid either by increasing oral dose or providing IM/IV steroids perioperatively - taper back over several days until baseline dose is re-achieved (C) Do not change steroid dose at all, & use sedation & quality pain control as primary approaches to avoid a medical crisis (D) none of the above

D

CAH affects which of the following steroid hormones (A) cortisol, which regulates body's response to illness or stress (B) mineralocorticoids (i.e. aldosterone), which regulate Na+ & K+ levels (C) androgens (i.e. testosterone), which are sex hormones (D) all of the above

E

CDKs form complex with relevant ______ to drive cell-cycle progression (A) calmodulin (B) Cadherins (C) Dicer (D) Integrins (E) Cyclins

reduced, decreased

CSF concentrations of GABA are (increased/reduced) in epileptic patients & brain tissue exhibits (increased/decreased) GABAergic activity

C

Carbon dioxide (CO2) regulates blood flow to which one of the following organs? (A) Heart (B) Skin (C) Brain (D) Skeletal muscle at rest (E) Skeletal muscle during exercise

glucogenic, ketogenic

Carbon skeletons of some AAs used to produce glucose through gluconeogenesis → _____ AAs Carbon skeletons of others used to produce equivalents of acetyl-CoA or acetoacetate → ________ AAs

A

Caudal herniation of the cerebellar tonsils and medulla through the foramen magnum is called (A) Arnold-Chiari syndrome (B) cranium bifidum (C) Dandy-Walker syndrome (D) Down syndrome (E) myeloschisis

B

Causes arteriolar dilation & venous constriction → increased filtration OUT of capillaries → local edema (A) Histamine (B) Bradykinin (C) Serotonin (5-hydroxytryptamine) (D) Prostaglandins

de novo, external

Cells acquire cholesterol from ___ ______ synthesis & through _______ supply

A

Cephalic end of neural tube which forms bridge between 2 cerebral hemispheres and enables nerve fibers to pass from one to the other (A) lamina terminalis (B) falx cerebri (C) tentorium cerebelli (D) 4th ventricle

C

Changes in TPR change (A) cardiac output curve (B) venous return curve (C) A & B

B

Changes in blood volume or venous compliance change (A) cardiac output curve (B) venous return curve (C) A & B

preload, afterload, contractility

Changes in the ventricular pressure-volume loops are a result of 1. Increased ________ (end-diastolic V) → increase in stroke volume 2. increased ________ (aortic pressure) → less blood ejected during systole → stroke volume decreases & end systolic volume increases 3. increased ______ → stroke volume increases & end systolic volume decreases

D

Chemical mediators that attract neutrophils and allow PMNs to accumulate where their defensive actions are needed (A) P-selectin (B) Diapedesis (C) Proinflammatory cytokines (D) chemotaxis

A

Childhood _____ deficiency is a possible cause of short stature (A) Growth hormone (GH) (B) LH (C) cortisol (D) GnRH (E) Estrogen (F) Testosterone

C

Cholesterol increases membrane _____ → increases membrane permeability (A) rigidity (B) hydrophobicity (C) fluidity (D) hydrophilicity

C

Cholesterol is absorbed in the ___________ by specific transporters, including NPC1L1 and ABCG5 & ABCG8 (A) stomach (B) esophagus (C) intestine (D) liver

C

Cholesterol is synthesized from __________ in the ______ (A) acetyl-coenzyme A, spleen (B) steroid hormone, spleen (C) acetyl-coenzyme A, liver (D) androgens, liver

B

Cold → activate centers in _______ → activate motoneurons innervating skeletal muscle → rhythmic contraction to generate heat & raise body temp (shivering) (A) anterior hypothalamus (B) posterior hypothalamus (C) Na+-K+ ATPase (D) alpha 1 receptors in vascular smooth muscle of skin blood vessels (E) beta receptors in brown fat

A

Combination OCP with fixed dose of estrogen & progesterone over a 21-day period (A) monophasic preparations (B) biphasic preparations (C) triphasic preparations (D) quadraphasic preparations

C

Combination OCP with varied amounts of estrogen & progestin every 7 days for 21 days (A) monophasic preparations (B) biphasic preparations (C) triphasic preparations (D) quadraphasic preparations

B

Combinations OCP with fixed dose of estrogen but increase the progestin dose over a 21-day period (A) monophasic preparations (B) biphasic preparations (C) triphasic preparations (D) quadraphasic preparations

E

Communicating (gap) junction permeability is decreased by (A) ↓ intracellular pH (B) ↑ intracellular pH (C) ↑ intracellular Ca2+ (D) ↓ intracellular Na+ (E) A & C (F) B & D

C

Communicating (gap) junctions mediate passage of chemical/electrical signals between cells (cardiac myocytes) by allowing passage of small molecules through pores created by hexamers of hydrophilic (A) cadherins (B) integrins (C) connexins (D) claudins

D

Complete absence of thyroid tissue, disorders, of thyroid synthesis, or result of congenital TSH deficiency (A) iodine deficiency (B) primary hypothyroidism (C) secondary hypothyroidism (D) congenital hypothyroidism

D

Concomitant use of _____ & _______ can increase edema & exacerbate existing cardiac or hepatic disease (A) estrogen & benzodiazepines (B) androgens & antibiotics (C) androgens & estrogen (D) androgens & corticosteroids

D

Conduction velocity depends on I. size of inward current during upstroke of action potential II. rate of rise of upstroke III. action potential duration (A) I only (B) III only (C) I and III (D) I and II (E) I, II, and III

C

Congenital defect in septum between atria due to failure of foramen primum or secondum to close → patent foramen ovale Shunts blood from L atrium to R atrium → hypertrophy of R atrium, ventricle, & pulmonary trunk → mixing of oxygenated & deoxygenated blood → cyanosis (A) Tetralogy of Fallot (B) Transposition of great vessels (C) Atrial Septal Defect (ASD) (D) Ventricular Septal Defect (VSD)

A

Continuous growth of _____ beginning at the 5th week of development gives rise to the frontal, parietal, occipital, and temporal lobes (A) cerebral cortex (B) cerebral hemispheres (C) corpus striatum (D) commissures

B

Contraceptives may also be applied subcutaneously, intramuscularly, transdermally, or vaginally → increase hormone plasma concentrations by (A) stimulating release from anterior pituitary (B) reducing exposure to liver (C) injecting additional hormones (D) reducing exposure to kidney

D

Contractility (aka ionotropism) is the intrinsic ability of myocardial cells to develop force at a given muscle length. It correlates directly with intracellular [Ca2+] which depends on amount released from SR which depends on: (A) size of inward current of [Ca2+] during plateau (B) amount of Ca2+ previously stored in SR (C) amount of troponin C (D) A & B (E) All of the above

C

Conversion of _________ involves removal of the methyl group at C-19 (A) cholesterol into aldosterone (B) corticosteroids into androgens (C) androgens into estrogens (D) cholesterol into cortisol

B

Conversion of __________ requires the C17-C20 split & addition of 17alpha-hydroxyl group (A) cholesterol into aldosterone (B) corticosteroids into androgens (C) androgens into estrogens (D) cholesterol into cortisol

E

Conversion of cholesterol to ______ (steroid precursor) is the initial rate-limiting step in steroidogenesis and occurs in _______ (A) pregnenolone, cytoplasm (B) mineralcorticoids, mitochondria (C) pregnenolone, SER (D) mineralcorticoids, SER (E) pregnenolone, mitochondria

B

Coronary circulation & Cerebral circulation are controlled by (A) distant hormones (B) local metabolic factors (C) pressure differences (D) prostaglandins only

A

Curve A in the figure represents (A) aortic pressure (B) ventricular pressure (C) atrial pressure (D) ventricular volume

D

Curve B in the figure represents (A) left atrial pressure (B) ventricular pressure (C) atrial pressure (D) ventricular volume

A

Cushing syndrome is caused by ______, and is diagnosed using _____ to confirm presence of hypercortisolemia (A) pituitary corticotrope adenoma, 24-h urine free cortisol (UFC) (B) pituitary corticotrope adenoma, 24-h blood free cortisol (C) adrenal adenoma, 24-h blood free cortisol (D) adrenal adenoma, 24-h urine free cortisol (UFC)

polymorphisms

Cytochrome P-450 gene ________ determine the response to many drugs

F

Cytokine(s) whose main function is activation of macrophages (A) GM-CSF, M-CSF (B) TNF-α, TGF-β, IL-1 (C) IL-12 (D) IL-2, IL-4 (E) IL-5 (F) interferon-γ, IL-4 (G) IL-10 (H) interferon-α, interferon-β (I) IL-8

H

Cytokine(s) whose main function is antiviral activity (A) GM-CSF, M-CSF (B) TNF-α, TGF-β, IL-1 (C) IL-12 (D) IL-2, IL-4 (E) IL-5 (F) interferon-γ, IL-4 (G) IL-10 (H) interferon-α, interferon-β (I) IL-8

I

Cytokine(s) whose main function is chemokine for neutrophils and T lymphocytes (A) GM-CSF, M-CSF (B) TNF-α, TGF-β, IL-1 (C) IL-12 (D) IL-2, IL-4 (E) IL-5 (F) interferon-γ, IL-4 (G) IL-10 (H) interferon-α, interferon-β (I) IL-8

E

Cytokine(s) whose main function is eosinophil proliferation, differentiation, and activation (A) GM-CSF, M-CSF (B) TNF-α, TGF-β, IL-1 (C) IL-12 (D) IL-2, IL-4 (E) IL-5 (F) interferon-γ, IL-4 (G) IL-10 (H) interferon-α, interferon-β (I) IL-8

D

Cytokine(s) whose main function is growth factors for T helper cells and B lymphocytes (A) GM-CSF, M-CSF (B) TNF-α, TGF-β, IL-1 (C) IL-12 (D) IL-2, IL-4 (E) IL-5 (F) interferon-γ, IL-4 (G) IL-10 (H) interferon-α, interferon-β (I) IL-8

G

Cytokine(s) whose main function is inhibition of macrophages and specific adaptive immune responses (A) GM-CSF, M-CSF (B) TNF-α, TGF-β, IL-1 (C) IL-12 (D) IL-2, IL-4 (E) IL-5 (F) interferon-γ, IL-4 (G) IL-10 (H) interferon-α, interferon-β (I) IL-8

C

Cytokine(s) whose main function is stimulation of growth in T lymphocytes and NK cells (A) GM-CSF, M-CSF (B) TNF-α, TGF-β, IL-1 (C) IL-12 (D) IL-2, IL-4 (E) IL-5 (F) interferon-γ, IL-4 (G) IL-10 (H) interferon-α, interferon-β (I) IL-8

B

Cytokine(s) whose main function is stimulation of inflammation & fever (A) GM-CSF, M-CSF (B) TNF-α, TGF-β, IL-1 (C) IL-12 (D) IL-2, IL-4 (E) IL-5 (F) interferon-γ, IL-4 (G) IL-10 (H) interferon-α, interferon-β (I) IL-8

ubiquitin

Cytosolic proteins often tagged with _______ for degradation by proteasomes

D

Cytosolic proteins tagged with _____ → unfolded & transferred to polymeric protease complex → proteins digested, degraded, & recycled (A) RISC (B) clathrin (C) connexin (D) ubiquitin

CDP-choline

DAG → phosphatidylcholine (PC aka lecithin) requires activation of choline to _____-______ which is activated by a pyrophosphorylase reaction so phosphocholine will be transferred to DAG → PC formed

condensation, silenced

DNA methylation → chromatin ______ → transcription ________

heart block

Damage to the conducting system can cause a ______ _____ which interferes with the ability of ventricles to receive atrial impulses

A

Decreased arterial pressure → increased autonomic stimulation to JGA → JG cells release _______ →cleaves angiotensinogen into inactive angiotensin I → ACE converts angiotensin I to angiotensin II → vasoconstriction that raises systemic BP → stimulates adrenals to secrete _______ → promotes Na+ & water reabsorption in DCT & Connecting tubules → raises blood V to help increase BP (A) renin, aldosterone (B) aldosterone, ADH (C) erythropoietin, vasopressin (D) ADH, renin

C

Deep Brain stimulation (DBS) involves regulation of the anterior nucleus of the _______ (A) frontal lobe (B) hypothalamus (C) thalamus (D) parietal lobe

D

Defect in adrenal cortex, anterior pituitary, or hypothalamus → insufficient production of __________ (A) epinephrine (B) ACTH (C) dopamine (D) corticosteroids

malignant transformation

Defective CDKI checkpoint → cell w/ damaged DNA divides → mutated daughter cells at risk for _________ ________

D

Deficiencies in ______ can cause skeletal deformities because of the dependency of lysyl oxidase which is the enzyme responsible for crosslinking to impart tensile strength to collagens (A) vitamin A (B) vitamin D (C) vitamin K (D) vitamin C

C

Deficiency of α1-antitrypsin presents in infancy as ______ or in adulthood as _______ (A) lung disease, cardiac disease (B) respiratory disease, cardiac disease (C) liver disease, lung disease (D) cardiac disease, lung disease

A

Degrade denatured/tagged cytosolic proteins → short peptides (A) Proteasomes (B) Lysosomes (C) Peroxisomes (D) Golgi Body

D

Dehydration increases protein concentration in blood → increase in ____ → filtration out of capillary is _______ (A) πc (capillary oncotic pressure), favored (B) Pc (capillary hydrostatic pressure), favored (C) Pc (capillary hydrostatic pressure), opposed (D) πc (capillary oncotic pressure), opposed

A

Dehydration → ADH released from posterior pituitary → makes _______ more permeable to water & increases rate at which water molecules pulled osmotically from filtrate (A) collecting ducts (B) PCT (C) Loop of Henle (D) DCT

B

Depending on _________, mitochondrial metabolism can support either cell maintenance or cell growth (A) age of organism (B) growth state of cell (C) cell signaling (D) amount of mitochondria present in cell

C

Depending on _________, the liver can either take up or produce glucose (A) blood chemistry (B) genotype (C) metabolic conditions (D) circadian rhythm

C

Depolarization of the ventricles (A) P wave (B) PR interval (C) QRS complex (D) T wave (E) QT interval

C

Depolarization → Na+ activation gates _____ and inactivation gates ______ → inactivation gates _______ → no inward depolarizing current → no action potential → cell is refractory (A) open, open slowly, open & unavailable (B) closed, open slowly, open (C) open, close slowly, closed & unavailable (D) closed, close rapidly, closed but available

B

Developing T cells w/ nonfunctional TCRs are detected & removed in the thymic cortex by a process of __________; cells w/ functional TCRs move into the thymic medulla (A) negative selection (B) positive selection (C) self selection (D) maturation

B

Development and closure of neural tube occurs within ________, and risk of neural defects is greatly reduced in women taking 4,000 mg of _______ daily (A) 2 months, iron (B) 28 days, folic acid (C) 3 months, folic acid (D) 33 days, iron

C

Dexamethasone is an example of a __________ glucocorticoid that lasts for (A) short-acting, < 6 hours (B) intermediate-acting, 12 to 36 hours (C) long-acting, > 36 hours (D) short-acting, < 12 hours

B

Diaphysis is mostly ______ with a thin region of ______ on inner surface around central marrow cavity (A) woven bone, compact bone (B) compact bone, cancellous bone (C) cancellous bone, woven bone (D) cancellous bone, compact bone

synovial fluid

Diarthroses have a joint cavity filled w/ lubricant _______ _______ enclosed in a tough, fibrous articular capsule

A

Diastolic pressure is (A) lowest, Parteries during ventricular relaxation (B) lowest, Parteries after blood ejected from L ventricle during systole (C) highest, Parteries during ventricular relaxation (D) highest, Parteries after blood ejected from L ventricle during systole

A

Diazepam, clonazepam, clorazepate, midazolam, & lorazepam are examples of (A) Benzodiazepines (B) Barbiturates (C) Hydantoins (D) Succinimides

ketogenic

Dietary modifications are useful for childhood epilepsy. The _______ diet is high in fats & low in carbs and proteins which forces the body to use fat for the primary energy source, increasing concentration of ketones and improving seizure control

B

Difficult or labored breathing (A) tachypnea (B) dyspnea (C) orthopnea (D) apnea

C

Diverse actions of corticosteroids achieved by regulating ___________ (A) hormone release (B) receptor binding interactions (C) gene expression (D) anterior pituitary & hypothalamus

D

During phase 2 (plateau), there is a stable, depolarized membrane potential due to inward ____ current and a balancing outward ___ current (A) K+, Ca2+ (B) Na+, K+ (C) Ca2+, Na+ (D) Ca2+, K+

C

During rapid ventricular filling, ventricular P < atrial P → _________ (A) aortic valve opens (B) aortic valve closes (C) mitral valve opens (D) mitral valve closes

reduces

During systole, mechanical compression of coronary vessels (increases/reduces) blood flow

D

Epilepsies, which are characterized by periodic & abnormal discharge of neurons within the brain are diagnoses if a person has ≥ _____ unprovoked seizures > _____ apart (A) 5, 12 hours (B) 3, 24 hours (C) 2, 2 days (D) 2, 24 hours

B

Epithelial cell membrane domains containing many stiffened plaques of protein are an important feature in which part of the urinary system? (A) Juxtaglomerular apparatus (B) Bladder mucosa (C) Collecting ducts (D) Renal pyramids (E) Membranous urethra

conjugation

Esterification (_________) of bilirubin carboxylic acid side chains with glucuronic acid is catalyzed by UDP-glucoronyl transferase. This reaction makes makes bilirubin water soluble so it can be secreted by the hepatocytes into biliary canaliculi

A

Estrogen most abundant in men & postmenopausal women (A) estrone (B) estradiol (C) estriol

C

Estrogen synthesis inhibits aromatase in order to block conversion of ______ to ____ → decease plasma estrogen levels (A) estriol to estradiol (B) testosterone to estriol (C) testosterone to estradiol (D) estrone to estriol

D

Estrogens may increase the effect of _________ (A) NSAIDs (B) Benzodiazepines (C) ACE-inhibitors (D) corticosteroids

A

Examination of a normal peripheral blood smear reveals a cell more than twice the diameter of an erythrocyte with a kidney-shaped nucleus. These cells are less than 10% of the total leukocytes. Which of the following cell types is being described? (A) Monocyte (B) Basophil (C) Eosinophil (D) Neutrophil (E) Lymphocyte

C

Excess GH secretion is most commonly due to (A) congenital defect (B) adrenal tumor (C) pituitary tumor (D) autoimmune disorder

E

Excess ___ → galactorrhea/impotence (men); infertility (women) Deficiency in ___ → no effect (A) ACTH (B) TSH (C) FSH/LH (D) Growth Hormone (E) Prolactin

B

Excess ____ → secondary hyperthyroidism Deficiency in ____ → secondary hypothyroidism (A) ACTH (B) TSH (C) FSH/LH (D) Growth Hormone (E) Prolactin

D

Excess _____ → gigantism/acromegaly Deficiency in ____ → short stature in children (A) ACTH (B) TSH (C) FSH/LH (D) Growth Hormone (E) Prolactin

A

Excess ________ → Cushing Disease Deficiency in _____→ secondary hypoadrenalism (A) ACTH (B) TSH (C) FSH/LH (D) Growth Hormone (E) Prolactin

C

Excess production of glucocorticoids (cortisol) → _________ Secretion below normal range → ________ (A) Gaucher disease, PKU (B) Goodpasture syndrome, Adenoma (C) Cushing disease, Addison disease (D) Addison disease, Goodpasture syndrome

C

Excessive cholesterol intake & disorders in cholesterol transport/handling by cells linked to _________ (A) osteoporosis (B) hyperlipidemia (C) atherosclerosis (D) hypocholesteremia

D

Ezetamibe is used for treatment of hypercholesterolemia. It works by (A) stimulating the NPC1L1-mediated cholesterol transport (B) inhibiting action of ACAT esterification of cholesterol in Cells (C) inhibiting action of cholesterol-lecithin acyltransferase in plasma (D) inhibiting the NPC1L1-mediated cholesterol transport

B

Factors that INCREASE filtration out of capillary (A) ↓ Pc, ↓ Pi, ↓ πc, ↓, πi (B) ↑ Pc, ↓ Pi, ↓ πc, ↑, πi (C) ↓ Pc, ↑ Pi, ↓ πc, ↑, πi (D) ↑ Pc, ↓ Pi, ↑ πc, ↓, πi

E (primary adrenal insufficiency)

Failure of the adrenal cortex → (A) Cushing Syndrome (B) Acromegaly (C) Marfan Syndrome (D) Grave's Disease (E) Addison's Disease

A

Final stages of cholesterol biosynthesis occur on a carrier protein because (A) All hydrophobic → in order for pathway to occur in an aqueous medium, the carrier protein is needed (B) it speeds up the reaction (C) regulatory mechanism to allow immediate transport of irregularly made cholesterol (D) Allows immediate delivery of cholesterol to sites where it is needed

E

First ventricular depolarization to last ventricular repolarization (A) P wave (B) PR interval (C) QRS complex (D) T wave (E) QT interval

B

Gives rise to a parasympathetic nucleus (A) GSE column (B) GVE column (C) alar plate (D) pontine nuclei (E) SVE column

D

Gives rise to motor neurons that innervate the tongue (A) corticospinal pyramid (tract) (B) tela choroidea (C) inferior olivary nucleus (D) basal plate (E) alar plate

E

Gives rise to motor neurons that migrate into the lateral pontine tegmentum (A) GSE column (B) GVE column (C) alar plate (D) pontine nuclei (E) SVE column

C

Gives rise to the cerebellum (A) GSE column (B) GVE column (C) alar plate (D) pontine nuclei (E) SVE column

B

Gives rise to the choroid plexus (A) corticospinal pyramid (tract) (B) tela choroidea (C) inferior olivary nucleus (D) basal plate (E) alar plate

E

Gives rise to the solitary nucleus (A) corticospinal pyramid (tract) (B) tela choroidea (C) inferior olivary nucleus (D) basal plate (E) alar plate

B

Glucocorticoids are contraindicated for use in treating _______ because suppression of host response may allow dissemination of herpes virus (A) angular cheilitis (B) herpetic ulcers (C) recurrent ulcerative stomatitis (D) erythema multiforme

C

Glycosphingolipids are degraded in _______ by a sequence of reactions that involve a stepwise removal of sugars from the _______end of the molecule (A) lysosomes, reducing (B) peroxisomes, reducing (C) lysosomes, nonreducing (D) peroxisomes, nonreducing

B

Growth factor receptor signaling leads to changes in cell metabolism that support growth, demonstrated in the Warburg effect which involves increased cell uptake of glucose & glutamine, meaning ________ (A) increased glycolysis & increased oxidative phosphorylation (B) increased glycolysis & decreased oxidative phosphorylation (C) decreased glycolysis & increased oxidative phosphorylation (D) decreased glycolysis & decreased oxidative phosphorylation

B

HMG-CoA reductase regulation by cholesterol involves enzyme degradation which is marked by increased (A) SCAP/SREBP complex binding to Insig-1 (B) HMGR binding to insig-1, increasing ubiquination (C) downregulation of LDL receptor genes (D)LXR complex upregulating SREBP1c

B

HPV proteins 6, 11, 16, & 18 combined w/ an adjuvant for HPV vaccine. Proteins ________ are linked with initiation of cervical cancer, making HPC vaccine an anticancer vaccine (A) 6 & 11 (B) 16 & 18 (C) 11, 16, & 18 (D) 6, 11, 16, & 18

B

High levels of __________, can indicate congenital adrenal hyperplasia due to a deficiency in the enzyme _________ (A) homocysteine, cystathionine beta-synthase (B) 17-OH progesterone, 21-hydroxylase (C) glucocerebroside, GCase (D) amino acids, BCKD

C

High-sensitivity CRP assay is used in assessment of ________ (A) liver function (B) leukemia (C) cardiovascular risk (D) renal failure

C

Higher-than-expected levels of hCG → (A) miscarriage or nonviable uterine gestation (B) ectopic (tubal) pregnancy (C) multiple gestations or molar pregnancy

C

Hydrocephalus + spina bifida → _________, which is displacement of hindbrain downward that obstructs flow of CSF through foramina in roof of 4th ventricle (A) Anencephaly (B) Dandy-Walker syndrome (C) Arnold-Chiari phenomenon (D) Holoprosencephaly

D

Hydrocortisone is an example of a __________ glucocorticoid that lasts for (A) short-acting, < 6 hours (B) intermediate-acting, 12 to 36 hours (C) long-acting, > 36 hours (D) short-acting, < 12 hours

D

Hydroxylation of Phe required in both normal degradation of C skeleton of AA AND synthesis of (A) Glutamate (B) Histidine (C) Tryptophan (D) Tyrosine

B

Hyperandrogenism due to testicular androgen excess may cause (A) infertility (B) precocious puberty (C) increased metabolic rate (D) hot flashes

B

If QRS is normal, the ________ is causing the first degree heart block (A) Bundle of His (B) AV node (C) Purkinje fibers (D) SA node

A

If mother is Rh- she will produce Rh antibodies if fetus is Rh+ Situation will not affect first pregnancy but will affect the second pregnancy with Rh+ fetus In the second pregnancy with an Rh+ fetus, a hemolytic condition of RBCs occurs known as _______ Rh- hemolytic disease of newborn (A) erythroblastisis fetalis (B) congenital syphilis (C) placental abruption (D) phenylketonuria (E) Edward's syndrome

D

If the brain becomes ischemic (decreased cerebral blood flow) → cerebral PCO2 _______ & pH ______ → medullary chemoreceptors detect changes → direct _____ in sympathetic outflow → intense arteriolar vasoconstriction & increase in TPR (A) increases, increases, decrease (B) increases, decreases, decrease (C) decreases, increases, increase (D) increases, decreases, increase

B

If the ejection fraction increases, there will be a decrease in (A) cardiac output (B) end-systolic volume (C) heart rate (D) pulse pressure (E) stroke volume (F) systolic pressure

increase, larger

If there is a positive ionotropic effect, there will be (increase/decrease/no change) in stroke volume & cardiac output for a given end-diastolic volume → (smaller/larger/same) fraction of end-diastolic volume ejected per beat

turbulent

If there is an irregularity in a blood vessel, flow becomes ______ and is no longer linear, wastes more kinetic energy, and is now audible

C

If there is increased parasympathetic activity or damage to the AV node, some action potentials will not be conducted at all from atria to ventricles leading to a (A) stroke (B) MI (C) heart block (D) valve defect

C

In 2⁰ lymphoid tissues, BCRs bind the FDCs (follicular dendritic cells) instead of MHC class II as it normally does. With cytokines from Th cells, a FDC-activated B cell proliferates clonally to produce temporally large ____ _______ which develops a pale ________ (A) germinal center, lymphoid nodule (B) thymocyte, germinal center (C) lymphoid nodule, germinal center (D) lymph nodes, lymphoid nodule

A

In Acute Respiratory Distress Syndrome (ARDS), premature infants have immature lungs without enough type 2 epithelial cells to synthesize enough phospholipid _____ → lungs collapse during expiration phase of breathing (A) DPPC (B) PE (C) PC (D) PS

A

In Gaucher Disease (A) sphingolipid accumulates in phagocytic cells (B) lipid accumulation causes cardiomegaly (C) inheritance is autosomal dominant (D) is treated with splenectomy

C

In ________ prolapse the valve everts into the L atrium → fails to close properly when L ventricle contracts (A) Tricuspid (Right AV) valve (B) Pulmonary valve (C) Mitral (Biscuspid/Left AV) valve (D) Aortic Valve

B

In _________, some impulses are blocked but not all. On an EKG, PR intervals will lengthen progressively with each beat until a QRS complex is missing. (AKA Wenckebach block) (A) 1st degree (B) 2nd degree, type 1 (C) 2nd degree type 2 (D) 3rd degree

D

In a _______, no atrial impulses are transmitted to the ventricles. As a result, the ventricles will generate an escape impulse, independent of its atrial beat. (A) 1st degree (B) 2nd degree, type 1 (C) 2nd degree type 2 (D) 3rd degree

D

In a capillary, Pc is 30mmHg, Pi is−2 mm Hg, πc is 25 mm Hg, and πi is 2 mm Hg. What is the direction of fluid movement and the net driving force? (A) Absorption; 6 mm Hg (B) Absorption; 9 mm Hg (C) Filtration; 6 mm Hg (D) Filtration; 9 mm Hg (E) There is no net fluid movement

E

In adults, the distal end of the spinal cord, the conus medullaris is at the level of the ____ vertebral body (A) T12 (B) T11 (C) L3 (D) T10 (E) L1

normal, NTD, abdominal wall defect

In amniotic fluid, no AChE is (normal/NTD/abdominal wall defect). Strong AChE denotes (normal/NTD/abdominal wall defect) and weak AChE denotes (normal/NTD/abdominal wall defect)

B

In an aneurysm of the aortic arch, a sac is formed by dilation of the aortic arch, which compresses the __________ nerve, causing coughing, hoarseness, & paralysis of ipsilateral vocal cord. It can also put pressure on the esophagus (→ dysphagia) & on the trachea or root of the lung (→ dyspnea) (A) R recurrent laryngeal (B) L recurrent laryngeal (C) Superior laryngeal (D) Phrenic (E) Pharyngeal

D

In an anteroposterior chest radiograph, the right border of the heart is represented by the___________ (A) Left auricle (B) Right ventricle (C) Left ventricle (D) Right atrium (E) Right auricle

higher, turbulent

In anemia, there is a decrease in hematocrit (mass of RBCs) → decrease in viscosity → (lower/higher) NR & (laminar/turbulent) blood flow

A

In cortex, _______ enter capillary clusters called glomeruli, which are drained by ________ instead of venules → arrangement allows higher hydrostatic pressure in the capillaries (A) afferent arterioles, efferent arterioles (B) interlobular arteries, peritubular capillaries (C) efferent arterioles, interlobular arteries (D) peritubular capillaries, vasa recta

RER, golgi, plasma membrane

In exocytosis, proteins are synthesized in the ____ & packed in the _____ → secretory vesicles → fuse with _______ → expel contents

pathogen, recombinant DNA

In order to make subunit vaccines, molecular components are either (1) isolated biochemically from _________ OR (2) synthesized by _______ _______ technology

C

In parallel resistance, as seen with blood flow among various major arteries branching the aorta: 1. total resistance ____ any of the individual resistances 2. there is _______ pressure (A) <, decreasing (B) >, decreasing (C) <, no loss of (D) >, no loss of

A

In positive chronotropic effects, the sympathetic system releases ______→ activates SA node _______ receptors → increase in If → increase rate of phase 4 depolarization (A) norepinephrine, beta 1 (B) norepinephrine, beta 2 (C) ACh, alpha 1 (D) ACh, Muscarinic (M2)

D

In series resistance, as seen in the arrangement of blood vessels within a given organ: 1. total resistance ____ the sum of the individual resistances 2. total flow is constant at each level and pressure _____ at each level A. >, is constant B. <, decreases progressively C. <, increases progressively D. =, decreases progressively E. =, is constant

A

In series resistance, the greatest decrease in pressure occurs in ______ because they have the largest resistance (R) (A) arterioles (B) arteries (C) capillaries (D) venules (E) veins

C

In starvation, secondary hepatic pathway supplements CDP-choline pathway through _______ of PE w/ donor S-adenosylmethionine (SAM) to form the PC needed (A) phosphorylation (B) acetylation (C) methylation (D) deacetylation

A

In the amniocentesis performed on Savanna Murphy's baby, the amniotic fluid AFP was elevated at 48.6 micrograms/ml, which was 4.5 MoM (multiple of Median), why is MOM used? (A) normalize the test result (B) easier value to read (C) calculate an average (D) simplify test result for telling patient

A

In the clinical lab, hormone levels in blood & urine are usually measured using ______ or ______ (A) immunoassay or mass spectrometry (B) immunoassay or electrophoresis (C) gas chromatography or electrophoresis (D) ultracentrifugation or mass spectrometry

A

In the cortex, the ____ (aka the distal straight tubule) contacts the arterioles at the vascular pole of its parent renal corpuscle & thickens focally there as macula densa (A) TAL (B) connecting tubules (C) Loop of Henle (D) DCT

D

In the cushing reaction, intracranial pressure increases and compresses the cerebral arteries leading to an increase in PCO2 and decrease in pH, which is detected by ______ that respond by increasing sympathetic outflow which increases TPR → increase Pa (A) Renin-Angiotensin II-Aldosterone System (B) Baroreceptors in carotid sinus (C) Peripheral Chemoreceptors in carotid & aortic bodies (D) Central (Medullary) Chemoreceptors

A

In the de novo pathway, _____ is acylated by transfer of 2 fatty acids from acetyl-CoA → _______, which is then dephosphorylated to ______ by a specific cytosolic phosphatase (A) G3P, phosphatidic acid (PA), diacylglycerol (DAG) (B) G3P, phosphatidylcholine (PC), DHAP (C) DHAP, phosphatidylethanolamine (PE), cytidine-diphosphate (CDP) (D) DHAP, G3P, cytidine-diphosphate (CDP)

placenta

In the fetus, blood is oxygenated in the ________, not the lungs, which is why there are 3 shunts that partially bypass the lungs and liver

>, <

In the starling equation, Jv is fluid flow If Jv ___ 0 → net fluid movement OUT of capillary (filtration) If Jv ___ 0 → net fluid movement INTO capillary (absorption)

B

In the starling equation, ____ is the _______ , which represents the water permeability of the capillary wall (A) Jv, fluid movement (B) Kf, hydraulic conductance (C) Pc, capillary hydrostatic P (D) Pi, interstitial fluid hydrostatic P (E) πc, capillary oncotic pressure (F) πi, interstitial fluid oncotic pressure

A

In the thymic medulla T cells whose TCRs bind strongly to "self-proteins", including proteins of many non-thymus cell types made by thymic epithelial cells expressing the Aire gene, are induced to undergo apoptosis there in a process of _________ (A) negative selection (B) positive selection (C) self selection (D) maturation

D

In which of the following cells involved in erythropoiesis does hemoglobin synthesis begin? (A) Orthochromatic erythroblast (B) Polychromatophilic erythroblast (C) Reticulocyte (D) Basophilic erythroblast (E) Proerythroblast

C

In which of the following situations is pulmonary blood flow greater than aortic blood flow? (A) Normal adult (B) Fetus (C) Left-to-right ventricular shunt (D) Right-to-left ventricular shunt (E) Right ventricular failure (F) Administration of a positive inotropic agent

D

In which of the following situations would you most likely give a systemic glucocorticosteroid? (A) oral ulcerations, such as denture-induced ulcers or angular cheilitis (B) TMJ disorders - refractory cases or severe acute pain (C) Postoperative sequelae (D) allergic reactions - severe response

D

Inactivated vaccines are made from which of the following? (A) killed bacteria (B) inactivated toxins (toxoids) (C) protein or polysaccharide subunits of the bacteria (D) all of the above

D

Inborn error of metabolism with defect in branched-chain keto acid decarboxylase which catalyzes decarboxylation of alpha-keto acid that results from loss of alpha-amino group of branched-chain AAs (A) PKU (B) Alkaptonuria (Black Urine Disease0 (C) Gaucher Disease (D) Maple Syrup Urine Disease (MSUD)

B

Inborn error of metabolism with deficiency in enzyme catalyzing oxidation of homogentisic acid, an intermediate in catabolism of tyrosine & phenylalanine (A) PKU (B) Alkaptonuria (Black Urine Disease) (C) Gaucher Disease (D) Maple Syrup Urine Disease (MSUD)

A

Incapable of generating a 2nd action potential no matter the size of the stimulus (A) Absolute Refractory Period (ARP) (B) Effective Refractory Period (ERP) (C) Relative Refractory Period (RRP) (D) Supranormal Period (SNP)

more, increase, more, increases

Increase in HR → (less/more) action potentials per unit time & (decrease/increase) in total amount of trigger Ca2+ → SR accumulates (less/more) Ca2+ for subsequent release → contractility (decreases/increases)

increased, decreased, vasodilation, increased

Increase in blood V detected by cardiopulmonary baroreceptors leads to 1. (increased/decreased) secretion of ANP → increased Na+ & water excretion in kidneys 2. (increased/decreased) secretion of ADH 3. renal (vasoconstriction/vasodilation) 4. (increased/decreased) HR

A

Increase in either arterial or venous pressure increases ______, which means ______ will be favored (A) Pc, filtration (B) Pi, filtration (C) Pc, absorption (D) Pi, absorption

B

Increase preoperative steroid either by increasing oral dose or providing IM/IV steroids perioperatively - taper back over several days until baseline dose is re-achieved (A) Dexamethasone 6 to 10 mg most commonly used in dentistry (B) Methyprednisolone dose pack most commonly used in dentistry (C) By far most common choice when it comes to steroid supplementation (or lack thereof) in dentistry (D) Cortisol most commonly used in dentistry

vasodilation

Increased ambient temperature → cutaneous (vasoconstriction/vasodilation) in order to dissipate excess body heat

C

Individual variation is encoded in < 0.5% of DNA via ______ which are usually biallelic and influence disease stability if located in regulatory elements and ______ which are different numbers of large contiguous stretches of DNA, 50% of which involve gene-coding sequences (A) SNPs, nonsense mutations (B) nonsense mutations, Copy number variations (CNVs) (C) SNPs, CNVs (D) nonsense mutations, gene translocations

C

Individuals w/ ________ present a higher risk for HTN, osteoporosis, & peptic ulcer (A) hyperthyroidism (B) hypothyroidism (C) hyperadrenalism (D) hypoadrenalism

C

Ineffective megakaryopoiesis due to deficiencies of folic acid or vitamin B12 can lead to a reduction in the number of platelets, known as (A) hyperemia (B) demarcation membrane (C) Thrombocytopenia (D) Erythrocytopenia

D

Inheritable disorder of CT that affects the skeleton (long limbs), eyes (dislocated lens), lungs (pneumothorax), & heart and blood vessels (aortic root dilation, aortic aneurysm, aortic regurgitation, & mitral valve prolapse). May be treated with beta-blocker medications. (A) Ehler-Danlos Syndrome (B) Polymyositis (C) Loeys-Dietz Syndrome (D) Marfan Syndrome

B

Inhibitors of testosterone synthesis & secretion work in 1 of 2 ways to prevent testosterone synthesis 1. GnRH antagonists inhibit ___ secretion → prevent testosterone synthesis in testes 2. Abiraterone acetate inhibits _______ → reduces testosterone synthesis (A) cytochrome P-450c17; 5α-reductase (B) LH; cytochrome P-450c17 (C) 5α-reductase; LH secretion (D) LH; androgen receptor

A

Is derived from the telencephalon (A) corticospinal pyramid (tract) (B) tela choroidea (C) inferior olivary nucleus (D) basal plate (E) alar plate

1-C, 2-B, 3-A

Jaundice can be pre-, post-, or intrahepatic 1. Prehepatic 2. Intrahepatic 3. Posthepatic A. obstruction to biliary excretion B. impaired hepatic metabolism or secretion of bilirubin C. increased production or impaired hepatic uptake of bilirubin

B

Kinesins - Dyneins - (A) retrograde (+ to -) transport; anterograde (- to +) transport (B) anterograde (- to +) transport; retrograde (+ to -) transport

C

Large, bulbous superficial cells of urothelium called ________have apical membranes consisting of hinged regions w/ dense plaques of uroplakin proteins that protect the cytoplasm (A) mesangial cells (B) podocytes (C) umbrella cells (D) juxtaglomerular cells

periosteum

Location: outer surface of most bones Description: -Fibrous layer of condensed CT covering bone -Bound to underlying bone by collagen fibers -Contains numerous osteoprogenitor cells -Does not cover articular surfaces or attachment sites of tendons and ligaments Essential to bone nutrition, growth & repair

D

Long bones have epiphyses made of ________ covered by a thin layer of ________ (A) woven bone, compact bone (B) compact bone, cancellous bone (C) cancellous bone, woven bone (D) cancellous bone, compact bone

C

Long-term changes in albumin levels reflect a patient's _______ (A) history of infection (B) cardiac function (C) nutritional status (D) hydration level

E

Loss of functioning endocrine tissue may be the result of destruction due to ______ or ______ (A) trauma or neoplasia (B) trauma or autoimmunity (C) chronic infection or neoplasia (D) chronic infection or trauma (E) autoimmunity or neoplasia

D

Many plasma membrane proteins function together as larger complexes - may assemble under control of ________ in RER or by _______ in plasma membrane (A) lateral diffusion, chaperone molecules (B) chaperone molecules, simple diffusion (C) G proteins, lateral diffusion (D) chaperone molecules, lateral diffusion

1-B, 2-C, 3-A

Match the chromatin writer complex to its associated AA residues 1. Methylation 2. Acetylation 3. Phosphorylation A. Ser residues B. His Lys residues C. Lys residues

1-B, 2-C, 3-A

Match the component of the ECM to its function 1. Fibrous structural proteins 2. Water-hydrated gels 3. Adhesive glycoproteins A. connect ECM elements to one another & to cells B. tensile strength & recoil C. permit compressive resistance & lubrication

1-B, 2-D, 3-C, 4-A

Match the component of the Renin-Angiotensin II-Aldosterone system to its description 1. Enzyme 2. Inactive 3. Physiologically active vasoconstrictor 4. Degrades angiotensin II A. Angiotensinase B. Renin C. Angiotensin II D. Angiotensin I

1. B 2. C 3. A

Match the following to its correct definition 1. Metabolomics 2. Genomics 3. Transcriptomics A. study of gene expression B. study of all metabolites generated in the organism C. study of genes

1-B, 2-A, 3-C

Match the following: 1. thick-walled with extensive elastic tissue & smooth muscle; under HIGH pressure 2. thin-walled with single layer of endothelial cells; largest total cross-sectional SA 3. thin-walled and under LOW pressure A. Capillaries B. Arteries C. Veins

1-C, 2-A, 3-B

Match the glycolipid to its description 1. Neutral glycolipids 2. Sulfatides 3. Gangliosides A. Formed by addition of sulfate from donor PAPS B. Contains sialic acids C. Contain only neutral & amino sugars

1-B, 2-A, 3-C

Match the hormone to its function: 1. _____ → testosterone secretion by Leydig cells of the testes 2. _____ → promotes spermatogenesis in seminferous tubules 3. _____ → negative feedback to GnRH & LH secretion A. FSH B. LH C. Testosterone (& estradiol)

A-2, B-3, C-1

Match the hormone to the zone of the adrenal cortex in which it is synthesized A. Zona reticularis B. Zona fasciculata C. Zona glomerulosa 1. Mineralcorticoids 2. Adrenal androgens 3. Glucocorticoids

1-B, 2-A. 3-C

Match the male portion of the urethra to the correct lining epithelium 1. prostatic urethra 2. membranous urethra 3. spongy (penile) urethra A. stratified columnar & pseudostratified columnar B. urothelium C. stratified columnar & pseudostratified columnar w/ stratified squamous distally

1. B 2. C 3. A 4. D

Match the neutrophil compartment to its description: 1. compartment in active marrow 2. compartment as mature cells in marrow until release (reserve) 3. population in blood vessels 4. compartment where neutrophils adhere loosely & accumulate along endothelial surface in venules A. Circulating B. Granulopoietic C. Storage D. Marginating

1-B, 2-C, 3-A, 4-D

Match the phase of the cell cycle to its description: 1. G1 2. S 3. G2 4. M A. premitotic growth B. presynthetic growth C. DNA synthesis D. mitotic phase

1-B, 2-A, 3-D, 4-E, 5-C

Match the phase to its event 1. phase 0 2. phase 1 3. phase 2 4. phase 3 5. phase 4 A. initial repolarization (net outward current) B. upstroke - rapid depolarization C. resting membrane potential or electrical diastole D. plateau - stable, depolarized membrane potential E. repolarization - outward currents>inward currents

1-B, 2-C, 3-A

Match the substance to its method of passage across the capillary wall 1. Lipid-soluble (O2 & CO2) 2. Small water-soluble (water, glucose, & AAs) 3. Large water-soluble A. pinocytosis B. simple diffusion C. water-filled clefts b/w endothelial cells

1-B, 2-C, 3-A

Match the temperature regulation disturbance to its description 1. Heat exhaustion 2. Heat stroke 3. Malignant hyperthermia A. massive increase in metabolic rate, increased O2 consumption, & increased heat production in skeletal muscle B. excessive sweating → decreased ECF V, decreased blood V, decreased Pa, & fainting C. body temp increases to point of tissue damage

A

Matrix cells divide to eventually become ____, which are incapable of further division (A) neuroblasts (B) erythroblasts (C) pluripotent stem cells (D) monocytes

C

Mean arterial pressure is the average pressure in a complete cardiac cycle. It is the driving force for blood flow in _______ and is influenced more by _______ than _______ because a greater proportion of the cardiac cycle is spent in ________ (A) veins, systole than diastole, systole (B) veins, diastole than systole, diastole (C) arteries, diastole than systole, diastole (D) arteries, systole than diastole, systole

C

Mean arterial pressure is the driving force for blood flow, and must be maintained at a high constant level of ~ ______. Pa = ? (A) ~50 mm Hg; = cardiac output x venous return (B) ~ 150 mm Hg; = venous return x TPR (C) ~100 mm Hg; =cardiac output x TPR (D) ~200 mm Hg; = TPR x R atrial P

B, C

Mean systemic pressure = R atrial pressure when there is not flow in cardiovascular system. It is decreased (shifted to the LEFT) by (select all that apply): (A) increase in blood volume (B) decrease in blood volume (C) increase in venous compliance (D) decrease in venous compliance

A, D

Mean systemic pressure = R atrial pressure when there is not flow in cardiovascular system. It is increased (shifted to the RIGHT) by (select all that apply): (A) increase in blood volume (B) decrease in blood volume (C) increase in venous compliance (D) decrease in venous compliance

A

Measurement of IGF-1 has clinical utility as an indicator of integrated ___ activity (A) Growth hormone (GH) (B) LH (C) cortisol (D) GnRH (E) Estrogen (F) Testosterone

D

Measurement of blood levels of target & pituitary hormones can help determine whether endocrine dysfunction originates from a _______ (primary) or from hypo-/hyperfunction of the _____ (secondary) (A) congenital defect, hypothalamus (B) peripheral endocrine gland, hypothalamus (C) central endocrine gland, pituitary (D) peripheral endocrine gland, pituitary

blood & fluids

Measurement of hormones in ______ and _____ forms part of assessment of hormone action & endocrine axes

ACTH

Measurement of plasma ______ in the presence of hypercortisolemmia is used to determine whether cortisol production is ACTH-driven rather than autonomous

B

Micro-RNAs (miRNAs) are a fundamental & evolutionary conserved mechanism of gene regulation in all eukaryotes that work by (A) bind to chromatin to restrict RNA polymerase access to coding genes → repression (B) post-transcriptional silencing (C) gene editing (D) repositioning nucleosomes on DNA to expose or hide gene regulatory elements

C

Mineralocorticoid that is responsible for regulation of extracellular volume & control of K+ homeostasis (A) ACTH (B) Cortisol (C) Aldosterone (D) DHEA

D

Muscle protein & adipose lipids are consumed to support _________ during fasting & starvation (A) citric acid cycle (B) metabolism (C) oxidative phosphorylation (D) gluconeogenesis

C

Mutations that cause a loss of a specific enzyme that helps break down certain parts of food and therefore lead to build up of that substance are known as _________ and are tested for with a battery of metabolic screenings (A) gain of function mutations (B) loss of function mutations (C) inborn errors of metabolism (D) inheritable diseases

D

Myocardial oxygen consumption correlates directly with (A) stroke volume (B) stroke work (C) cardiac output (D) cardiac minute work

C

N-acetylglutamate (& indirectly, arginine) is an essential allosteric regulator of _______ (A) glucose metabolism (B) protein synthesis (C) the urea cycle (D) gluconeogenesis

B

N-terminal domains of both H & L chains of antibodies contain variable (V) regions, which determine (A) class of immunoglobulin (B) antigen specificity (C) complement activation (D) phagocytic activation

B

Normally, > 90% of circulating glucocorticoids are bound to plasma proteins, mainly _______ which has a high affinity but low capacity for them & _______, which has opposite characteristics (A) beta-globulin, gamma globulin (B) alpha-globulin, albumin (C) albumin, gamma globulin (D) gamma globulin, albumin

B

Normally, filtration out of capillaries is slightly greater than absorption of fluid into capillaries. Excess filtered fluid is returned to circulation via ______ (A) plasma proteins (B) lymph (C) transmembrane proteins (D) RBCs

F

On electrophoresis, a paraprotein band is seen in the gamma region, which leads you to narrow down your diagnosis of the patient to I. Myeloma II. Waldenstrom's macroglobulinemia III. Hemolytic Anemia IV. Monoclonal gammopathies of uncertain significance (MGUs) V. Iron-deficiency anemia (A) I and III (B) I and IV (C) II and IV (D) I, III, and IV (E) I, II and V (F) I, II, and IV

A

Only APCs (mostly derived from monocytes) also present fragments of endocytosed foreign proteins on surface _______ molecules (A) MHC class II (B) CD (C) MHC class I (D) Interleukins

binding, protease inhibitors

Opsonins, such as C-reactive protein, are called _________ proteins, since bind to macromolecules released by damaged tissue or infective agents ________ _______ inhibit proteolytic enzymes

A

Optic vesicles appear on each side of the _______, and give rise to the retina and optic nerve (A) forebrain (B) midbrain (C) pons (D) hindbrain

Osteoblasts

Origin: Osteoprogenitor cells Function: Secrete component of initial matrix (osteoid) that allows matrix mineralization to occur Rich in RER & golgi

remodeling pathway

Originally attached fatty acids in the de novo pathway are replaced with new ones in the ______ _______, which generates the diversity & asymmetry of the acyl groups

B

Originates from left 6th arch and connects the bifurcation of the pulmonary trunk with the aorta to partially bypass the lungs (A) foramen ovale (B) Ductus arteriosus (C) ductus venosus

C

Osteoblasts release matrix vesicles rich in ________ → activity raises local PO₄³− concentration → matrix vesicles become site of formation of HA crystals (A) calcium (B) phosphate (C) alkaline phosphatase (D) Na+

B

Osteoblasts secrete _______ which binds Ca2+ ions and concentrates it locally (A) matrix vesicles (B) osteocalcin (C) calcitonin (D) PTH

B

Osteoclasts development requires (A) Entrapment in lacunae & Macrophage-colony-stimulating factor (M-CSF) (B) RANKL & Macrophage-colony-stimulating factor (M-CSF) (D) Entrapment in lacunae & mesenchymal stem cells

A

Oxidation of the cholesterol side chains by cytochrome P-450 enzyme cholesterol 24-hydroxylase (CYP46A1) in the brain & 25 hydroxylase (CYP25A1) & 27-hydroxylase (CYP27A1) in other tissues yields (A) oxysterols (B) lanosterol (C) Farnesyl pyrophosphate (D) Squalene

-slows -hyperpolarizes -increases

Parasympathetic release of ACh and activation of M2 receptors → - (increases/slows) rate of phase 4 depolarization -(depolarizes/repolarizes/hyperpolarizes) maximum diastolic potential so more inward current needed to reach threshold potential -(increases/decreases) threshold potential

5th

Pericardiocentesis is a surgical puncture of the pericardial cavity to relieve pressure of accumulated fluid in the heart. The needle is inserted into the pericardial cavity through the _____ intercostal space

B

Pericarditis is inflammation of the pericardium. Similar to MIs, it causes continuous central chest pain, but unlike MIs, pain can be relieved by: (A) open bypass surgery (B) sitting forward (C) lying down (D) taking beta blockers (E) taking baby aspirin

fibrous, serous

Pericardium is made up of a tough, CT outer layer that defines the boundaries of the mediastinum known as the ________ pericardium and a _______ pericardium that is divided into a parietal & visceral layer

B, D, A, C, E

Place the following steps of endochondral ossification in order: A.Chondrocyte hypertrophy B. ossification in bone collar C. hypertrophic chondrocytes die and osteoblasts & vasculature fill available spaces & produce woven bone D. bone collar blocks diffusion of O2 & nutrients into underlying cartilage E. secondary ossification centers later at epiphyses F. Matrix secretes osteocalcin & alkaline phosphatase which initiates calcification

C, E, G, A, D, F, H, J, I, B

Place the following steps of excitation-contraction coupling in the correct order: A. Ca2+-induced Ca2+ release B. Ca2+ reaccumulated in SR by Ca2+ ATPase → relaxation C. Cardiac action potential D. Ca2+ release from SR E. T tubules spread depolarization to cell interior F. further increase in [Ca2+] & Ca2+ binds to troponin C G. inward Ca2+ current during plateau (trigger Ca2+) H. tropomyosin moved out of the way so actin-myosin can bind I. thick and thin filaments move past each other → tension J. cross-bridge cycling

D, B, E, A, F, C

Place the following steps of intramembranous ossification in order: A. continued matrix secretion & calcification B. osteoid calcifies C. compact bone replaces woven bone D. Osteoblasts secrete osteoid E. woven bone with osteocytes in lacunae & canaliculi F. neighboring ossification centers fuse

A, B, F, E, G, H, C, I, D, J

Place the pathway of blood in the correct order, starting with the Lungs: A. Lungs B. Left Atrium C. Vena Cava D. Right Ventricle E. Aorta F. Left ventricle G. Tissues H. Systemic veins I. Right atrium J. Pulmonary artery

C

Plant sterols & cholesterol precursors are markers of (A) cholesterol transporter mutation (B) liver disease (C) cholesterol absorption & metabolism (D) de novo synthesis

A

Plaque deposits in arterial walls → decreased diameter → stiffer & less compliant → decrease in systolic, pulse, & mean pressure (A) Arteriosclerosis (B) Aortic stenosis (C) Aortic regurgitation (D) Pulmonary HTN

prothrombin time (PT)

Plasma activities of liver enzymes are markers of liver disease but do NOT exactly reflect function of the liver. Prothrombin synthesis, assessed by ______ _____ is a better indicator of synthetic function

D

Plasma cholesterol concentration depends on (A) endogenous cholesterol synthesis (B) exogenous cholesterol synthesis (C) dietary intake (D) A & C (E) A & B

D

Plasma membrane has a variety of proteins/glycoproteins involved in (A) ion & metabolite transport (B) fluid-phase & receptor-mediated uptake of macromolecules (C) cell-ligand, cell-matrix, & cell-cell interactions (D) all of the above

lysosomes, proteasomes

Plasma proteins & membrane receptors are endocytosed then hydrolyzed by acid proteases in _________ VS. intracellular proteins which are degraded by UPS in __________

A

Platelet glycocalyx adheres to collagen in vascular basal lamina or wall to form platelet plug (A) primary aggregation (B) secondary aggregation (C) Blood coagulation (D) Clot retraction (E) Clot removal

B

Platelets consist of _____ a peripheral light blue-stained zone and a _________, a central zone containing purple granules (A) granulomere, hyalomere (B) hyalomere, granulomere

1-C, 2-B, 3-A

Platelets have a light-staining peripheral zone (1) with a peripheral (2) of microtubules & microfilaments and 2 systems of membrane channels and a darker-staining central zone (3) rich in granules A. granulomere B. marginal bundle (MB) C. hyalomere

B

Platelets in plug release specific adhesive glycoprotein & ADP → further platelet aggregation and increased plug size (A) primary aggregation (B) secondary aggregation (C) Blood coagulation (D) Clot retraction (E) Clot removal

D

Positive ionotropic agents (cardiac glycosides) produce increased contractility & increased cardiac output. As a result, (A) equilibrium point shifts to a higher cardiac output & a higher R atrial pressure (B) new equilibrium point where both cardiac output & R atrial pressure are increased (C) new equilibrium where both cardiac output & R atrial pressure are decreased (D) equilibrium point shifts to a higher cardiac output & a lower R atrial P

B

Posterior intercostal arteries in the 6th and 7th intercostal spaces are the branches of the_________ (A) Abdominal aorta (B) Descending thoracic aorta (C) Internal thoracic artery (D) Subclavian artery

D

Potential space between the visceral layer (epicardium) and parietal layer lining the inner surfaces of the fibrous pericardium that allows for movement of the heart is called (A) transverse sinus (B) fibrous pericaridum (C) serous pericardium (D) pericardial cavity (E) pericarditis

B

Prednisolone is an example of a __________ glucocorticoid that lasts for (A) short-acting, < 6 hours (B) intermediate-acting, 12 to 36 hours (C) long-acting, > 36 hours (D) short-acting, < 12 hours

B

Primary bile acids are synthesized in the ______, and secondary bile acids are synthesized in the _________ (A) intestine, pancreas (B) liver, intestine (C) gall bladder, liver (D) pancreas, gall bladder

D

Primary hyperaldosteronism is an autonomous hypersecretion of aldosterone (independent of renin-angiotensin-aldosterone system) → (A) Na+ & water excretion & enhanced renin production (B) Na+ & water excretion & suppressed renin production (C) Na+ & water retention & enhanced renin production (D) Na+ & water retention & suppressed renin production

urea, liver

Primary nitrogenous excretion product in humans is _____, produced by a cycle in the _____

de novo

Primary route for ____ _____ biosynthesis of phospholipids involves activation of one of the components (either DAG or head group) w/ CTP → form high-energy intermediate, such as CDP-diacylgycerol or CDP-choline

B

Primitive hemopoiesis during embryonic development starts in the _______, and after birth takes place primarily in the ______ (A) placenta, bone marrow (B) yolk sac, bone marrow (C) placenta, spleen (D) yolk sac, spleen

A

Principal cells of _______ are pale-staining, w/ relatively few MT & distinct cell membranes that are rich in aquaporins (water channels) for passive water reabsorption (A) collecting ducts (B) connecting tubules (C) Loop of Henle (D) ureters

A

Progestin in OCPs ___________ (A) protects endometrium of uterus against proliferative effects of estrogen (B) suppresses release of FSH (C) suppresses release of LH (D) Reduces sperm motility & nidation

endometrial cancer

Progestin is necessary in combination hormone replacement therapy to protect against _______ _______, since estrogen alone leads to an increased risk of the above in the uterus of postmenopausal women

A

Propranolol has which of the following effects? (A) Decreases heart rate (B) Increases left ventricular ejection fraction (C) Increases stroke volume (D) Decreases splanchnic vascular resistance (E) Decreases cutaneous vascular resistance

vasodilators, vasoconstrictors

Prostacyclin and E-series prostaglandins are (vasoconstrictors/vasodilators) F-series prostaglandins & Thromboxane A2 are (vasoconstrictors/vasodilators)

C

Protein binding helps to I. Control distribution of molecules & their availability to tissues II. grant cell immunity III. render toxic substances less harmful (A) I only (B) I and II (C) I and III (D) II and III (E) I, II, and III

C

Protein digestion begins in the ______ w/ pepsin (A) mouth (B) esophagus (C) stomach (D) small intestine

A

Protein synthesis (A) RER (B) Mitochondria (C) Golgi apparatus (D) SER (E) Cytoskeleton

lysosomes

Protein turnover is carried out in ______ by engulfment in autophagosomes which fuse with the organelle containing acid hydrolases that degrade protein, lipids, & glycans

B

Proteoglycans & hyaluronan are examples of ________ (A) Fibrous structural proteins (B) Water-hydrated gels (C) Adhesive glycoproteins (D) Fibronectins

A

Protozoan parasite transmitted through cat feces transmitted to human through ingestion of oocyst containing water or food or consumption of cyst containing raw or undercooked meat Transmitted to fetus transplacentally (A) Toxoplasma gondii (TG) (B) Other (syphillis) (C) Rubella (D) Cytomegalovirus infection (CMV) (E) Herpes

B

Pulse pressure = __________ and usually reflects the volume of blood ejected from the left ventricle on a single beat, known as ________ (A) systolic - diastolic, cardiac output (B) systolic - diastolic, stroke volume (C) diastolic - systolic, cardiac output (D) diastolic - systolic, stroke volume

D

Pulse pressure is (A) the highest pressure measured in the arteries (B) the lowest pressure measured in the arteries (C) measured only during diastole (D) determined by stroke volume (E) decreased when the capacitance of the arteries decreases (F) the difference between mean arterial pressure and central venous pressure

IL-1, prostaglandins, fever

Pyrogens increase production of ___ → acts on anterior hypothalamus to increase local production of _______ → increase set point temperature → normal core temperature seen by hypothalamic center as "too low" → heat-generating mechanism activated to raise body temp → _______

B

RPR (Rapid Plasma Reagin) is a lab test used to screen for (A) herpes zoster virus (B) congenital syphillis (C) treponema pallidum (D) toxoplasma gondii (TG)

A

Rapid conduction of ______ leads to efficient contraction and ejection of blood (A) Bundle of His, Purkinje system, & ventricles (B) AV node (C) SA node (pacemaker) (D) Atrial internodal tracts & atria

A

Rate at which blood is pumped from the ventricle (A) cardiac output (B) venous return (C) cardiac input (D) arterial return

B

Rate at which blood is returned to the atria (A) cardiac output (B) venous return (C) cardiac input (D) arterial return

C

Rate of ________ is one determinant of heart rate (A) phase 0 depolarization (B) phase 3 repolarization (C) phase 4 depolarization (D) opening of Ca2+ channels

D

Recycling of iron and heme, the major complex containing iron, occurs most actively in which lymphoid organ(s)/tissue(s)? (A) Lymph nodes (B) Peyer's patches (C) Tonsils (D) Spleen (E) Lymphatic vessels

C

Redistribution of fatty acids in phospholipids occurs during remodeling pathway thanks to _______ and _____ (A) phosphatidic acid & DAG (B) DAG & acetyl-CoA (C) PLA2 & LPLAT (D) Ceramide & Sphingosine

E

Regulates movement, cell shape, & intracellular organization → maintain cell polarity (A) RER (B) Mitochondria (C) Golgi apparatus (D) SER (E) Cytoskeleton

B

Renal arteries branch to form smaller arteries b/w the renal lobes, w/ _________ entering the cortex to form the microvasculature (A) afferent arterioles (B) interlobular arteries (C) efferent arterioles (D) peritubular capillaries

D

Repolarization of ventricles (A) P wave (B) PR interval (C) QRS complex (D) T wave (E) QT interval

decrease, increase, increase, ACE, increase, increase, vasoconstriction

Response of Renin-Angiotensin II-Aldosterone System to acute hemorrhage Decrease in Pa → 1. (decrease/increase) in renal perfusion pressure 2. (decrease/increase) in renin 3. (decrease/increase) conversion of angiotensinogen to angiotensin I 4. ______ converts angiotensin I to angiotensin II 5. (decrease/increase) angiotensin II 6. (decrease/increase) in aldosterone, Na+-H+ exchange, and thirst AND (vasoconstriction/vasodilation) → Increase in Pa toward normal

C

Response to an acute insult is associated with wide-ranging changes in liver protein synthesis. Acute-phase proteins are those whose plasma concentrations change by > _______ of onset of an inflammatory or infective process (A) > 50% within 7 days (B) >10% within 48 hours (C) > 25% within a week (D) > 20% within 24 hours

C

Rifampin, barbiturates, carbamazepine, phenytoin, & topiramate tend to decrease effects of estrogen by (A) inhibiting secretion of estrogen (B) blocking estrogen receptors (C) inducing liver metabolism of estrogens (D) None of the above - these drugs increase the effects of estrogen

A

S2 ("dub") heart sound is caused by (A) closure of aortic & pulmonary valves at onset of ventricular diastole (B) opening of aortic & pulmonary valves at onset of ventricular diastole (C) closure of aortic & pulmonary valves at onset of ventricular systole (D) opening of aortic & pulmonary valves at onset of ventricular systole

D

S2 (dub) is made by _______ and occurs at the ________ (A) vibration and closure of AV valves (tricuspid & mitral); beginning of systole (B) vibration and closure of AV valves (tricuspid & mitral; end of systole (C) closure of semilunar (aortic & pulmonic) valves; beginning of systole (D) closure of semilunar (aortic & pulmonic) valves; end of systole

D

SREBP1c can be activated by _______ in response to oxysterols (A) liver Y receptors (LYRs) (B) HMG-CoA reductase (C) SCAP (D) liver X receptors (LXRs)

E

Scott was given a tentative diagnosis of pervasive developmental disorder (AKA autism) due to which of the following findings? (A) fair complexion (B) clumsy gross motor movements more appropriate for a 2 year old (C) hyperkinetic behavior & unpredictable temper tantrums (D) A & B (E) B & C

1-C, 2-B, 3-A

Secretion of hormones can be (1) like GH, (2) like cortisol & testosterone, or (3) like FSH and LH, which limits the utility of "random sampling" A. infradian B. circadian C. pulsatile

lemon sign

Seen in patients with spina bifida (Savanna Murphy's baby), this is a feature on the radiograph where there appears to be an indentation of the frontal bone

D

Shear is the consequence of blood traveling at different velocities within a blood vessel. It is responsible for (A) breaking up aggregates of RBCs (B) Causing aggregates of RBCs (C) decreasing blood viscosity (D) A & C

highest, lowest

Shear is the consequence of blood traveling at different velocities within a blood vessel. Shear is (lowest/highest) at blood vessel wall and (lowest/highest) at the center of blood vessel

E

Shortly after her birth a baby is diagnosed with a mutation in the erythropoietin receptor gene which leads to familial erythrocytosis (familial polycythemia). During the seventh to ninth months of fetal development, the primary effect on her red blood cell production was in which of the following? (A) Liver (B) Yolk sac (C) Spleen (D) Thymus (E) Bone marrow

C

Shunts oxygenated blood from umbilical vein (returning from placenta) to the IVC, partially bypassing the liver (A) foramen ovale (B) Ductus arteriosus (C) ductus venosus

endothelial, Kupffer

Sinusoids are lined by 2 cell types: (1) vascular ________ cells which are loosely connected with numerous gaps (2) ________ cells which are mononuclear phagocytes generally found in the gap b/w endothelial cells

C

Some attenuated viruses may replicate at a localized site (i.e. GI tract) but don't reach target tissue of disease, such as the Sabin live attenuated polio vaccine that doesn't reach _______, but has an IgA response along _____, & IgG response in _________ (A) myocytes, brain, blood (B) blood, heart, interstitial fluid (C) neurons, gut, blood (D) gut, brain, interstitial fluid

C

Specialized cell organelles with catalase, peroxidase, & other oxidative enzymes → break down long fatty acid chains → generate hydrogen peroxide (A) Proteasomes (B) Lysosomes (C) Peroxisomes (D) Golgi Body

B

Specificity of hormone response depends on the presence of intracellular proteins or receptors, which specifically recognize & selectively bind the hormone & act in concert w/ hormone ligand to regulate ________ (A) Na+ channels (B) gene expression (C) neurotransmitter signaling (D) K+ channels

B

Squalene cyclizes to ________ (A) mevalonic acid (B) lanosterol (C) Farnesyl pyrophosphate (D) Squalene

B

Static or slow increase of hCG → (A) miscarriage or nonviable uterine gestation (B) ectopic (tubal) pregnancy (C) multiple gestations or molar pregnancy

D

Steroid Hormones act via (A) G-protein linked receptors (B) transmembrane receptors (D) nuclear receptors (E) receptor tyrosine kinases

D

Steroid hormone & lipoprotein synthesis + modification of hydrophobic compounds (A) RER (B) Mitochondria (C) Golgi apparatus (D) SER (E) Cytoskeleton

D

Steroidogenesis is controlled by (A) HMG-CoA reductase (B) Liver X receptors (LXRs) (C) cholestyramine (D) cytochrome P450 monoxygenases

B

Sterol regulatory element-binding proteins (SREBP) are transcriptional regulators of _________ synthesis (A) protein (B) cholesterol (C) glucose (D) fatty acid

D

Stroke volume divided by end-diastolic volume = ejection fraction, which is a measure of (A) Atrial efficiency (B) Cardiac efficiency (C) Perfusion efficiency (D) Ventricular efficiency

Haversian System (Osteon)

Structural & functional unit of Compact Bone - bones are arranged in lamella organized concentrically around small central canals containing blood vessels & nerves

D

Structure of the liver facilitates exchange of metabolites b/w _______ and ______ (A) hepatocytes & endothelial cells (B) interstitial fluid & macrophages (C) hepatocytes & RBCs (D) hepatocytes & plasma

D

Sudden onset with: - Loss of consciousness - Bilateral sharp tonic contraction of muscles - Generalized from onset, followed by clonic contractions Responsive to pharmacotherapy (A) Simple partial seizure (B) Complex partial seizure (C) Secondary Generalized seizure (D) Generalized tonic-clonic (grand mal) seizures (E) Petit Mal (Absence seizures)

C

Sudden, brief, & violent spasms of 1 or more muscle groups, which are refractory to pharmacotherapy are known as ________ (A) Atonic ("Drop Attacks") (B) Status Epilepticus (C) Myoclonic Seizures (D) Simple partial seizures

C

Surfaces of ALL NUCLEATED cells bear fragments of their constituent proteins on _______ molecules (A) MHC class II (B) CD (C) MHC class I (D) Interleukins

A

Sympathetic activates ______ receptors → adenylyl cyclase → production of cAMP → activation of protein kinase A → phosphorylation of proteins that increase ________ (A) beta-1, contractility (B) beta-1, HR (C) alpha-1, conduction velocity (D) beta-2, conduction velocity

D

Sympathetic innervation of vascular smooth muscle is an example of (A) Autoregulation (B) Active hyperemia (C) Reactive hyperemia (D) Hormonal (extrinsic) control of blood flow

increase, decrease

Sympathetic → (increase/decrease) HR Parasympathetic → (increase/decrease) HR

B

Synthesis of 1 molecule of cholesterol requires: (A) 8 moles acetyl-CoA, 16 moles ATP, 6 moles NADP (B) 18 moles acetyl-CoA, 36 moles ATP, 16 moles NADP (C) 28 moles acetyl-CoA, 46 moles ATP, 6 moles NADP (D) 36 moles acetyl-CoA, 24 moles ATP, 8 moles NADP

lipoproteins

Synthesis of cholesterol de novo & delivery by _________ are reciprocally related

adrenal cortex, testis, and ovary

Synthesis of steroid hormones occurs in what 3 organs?

D

Systolic pressure is (A) lowest, Parteries during ventricular relaxation (B) lowest, Parteries after blood ejected from L ventricle during systole (C) highest, Parteries during ventricular relaxation (D) highest, Parteries after blood ejected from L ventricle during systole

D

T cells function in _______ immunity (A) adaptive (B) innate (C) humoral (D) cell-mediated

B

T lymphoblasts, or thymocytes, attach in the ______ to a cytoreticulum composed of interconnected TECs (thymic epithelial cells) (A) Spleen (B) Thymus (C) Bone marrow (D) LN

true

T/F As cyclin levels rise & fall, activity of associated CDKs also rise and fall with it

false (the domains are distinct from each other)

T/F DNA-binding domain & drug-binding domain on cytostolic receptor that glucocorticoids binds are the same

false (opposite)

T/F Epiphyseal plate changes thickness but is NOT displaced away from center of diaphysis as length of bone increases

true

T/F For each miscarriage a woman has, their chance of having a baby with aneuploidy increases

false (glucocorticosteroids add a supplemental benefit after administering epinephrine)

T/F Glucocorticosteroids replace epinephrine as the primary drug of choice in the case of severe anaphylaxis

true

T/F Lymph flow is unidirectional, which is aided by one-way flap valves that permit interstitial fluid to enter, but not leave lymph vessels

false (constant state of synthesis, turnover, & remodeling)

T/F Phospholipids go through finite stages of of synthesis, turnover, & remodeling

false (next beat)

T/F Postextrasystolic potentiation means that when an extrasystole occurs, tension on the current beat is greater than normal

false (no sustained plateau)

T/F SA node has a unstable RMP and a sustained plateau

true

T/F TECs secrete many cytokines, compartmentalize the thymus into a cortex & a medulla, & in the cortex surround blood vessels in the blood-thymus barrier

false (true capillaries do NOT have smooth muscle)

T/F True capillaries have smooth muscle in addition to their single layer of endothelial cells surrounded by basement membrane

true

T/F There is no drug-free (placebo) period with continuous dosing OCPs

true

T/F - the hematopoietic stem cell is the common precursor of ALL formed elements of blood

false (ultrasound)

T/F Every pregnant woman has an amniocentesis performed

A

TFs activated by lipid-soluble ligands that easily pass through plasma membranes (A) intracellular receptors (B) cell surface receptors (C) signal transduction pathways

A

Tall epithelial cells of the macula densa & specialized smooth muscle cells in adjacent afferent arteriole (juxtaglomerular cells) that secrete ______, comprise a JGA (juxtaglomerular apparatus) important for regulating BP (A) renin (B) aldosterone (C) erythropoietin (D) ADH

D

Temporary cessation of breathing, especially during sleep (A) tachypnea (B) dyspnea (C) orthopnea (D) apnea

D

Testosterone is reduced to dihydrotestosterone in target tissues, which is the mediator of most actions. This irreversible conversion is only in tissue containing (A) 5α-kinase (B) 3β-phosphatase (C) 5β-reductase (D) 5α-reductase (E) 3β-kinase

1. stenosis 2. VSD 3. overriding 4. right

Tetralogy of Fallot occurs when AorticoPulmonary (AP) septum fails to align properly with the AV septum → 1. pulmonary _______ (obstruction to R ventricular outflow) 2. _____ (allows blood to move from L to R ventricle) 3. ________ aorta (dextraposition of aorta) 4. _______ ventricular hypertrophy Characterized by right-to-left shunting of blood & cyanosis

vertex, breech

The "normal" presentation of fetus during delivery where the highest part of the fetal head arrives first at mother's pelvic brim is __________ presentation _______ presentation occurs when the fetus's buttox arrives first at mother's pelvic brim

parietal, visceral

The (parietal/visceral) layer of serous pericardium lines the inner surface of the fibrous pericardium The (parietal/visceral) layer of serous pericardium adheres to the heart & forms the outer layer (epicardium) of the heart wall

left

The (right/left) ventricle must develop greater pressure to eject blood and therefore has a thicker wall

synthetic

The 2 forms of glucocorticoids as medications are natural hormone hydrocortisone & synthetic compounds. Which one is longer acting & more potent?

B

The 2 stage thymic selection process of positive followed by negative selection leads to _______ immune tolerance, producing functional T cells that do not bind to proteins of the host. (A) T cell (B) central (C) peripheral (D) primary

B

The Bacilli Calmette-Guérin (BCG) vaccine against TB is attenuated with (A) Salmonella typhi (B) Mycobacterium bovis (C) Neisseria meningitidis (D) Yersinia pestis

C

The Frank-Starling relationship states that: (A) volume heart ejects in systole > volume it receives in venous return (B) volume heart ejects in systole < volume it receives in venous return (C) volume heart ejects in systole = volume it receives in venous return (D) no relationship between volume heart ejects in systole and volume it receives in venous return

D

The Hb-haptoglobin complex, heme-hemopexin complex, and methemalbumin complex were evolved to prevent major losses of ______ and to complex free ______, which is toxic to many tissues (A) copper, O2 (B) iron, O2 (C) copper, heme (D) iron, heme

B

The ______ is a groove on the external surface of the heart that marks the division between the atria and ventricles (A) sulcus terminalis (B) coronary sulcus (C) marginal groove (D) coronary sinus (E) crista terminalis

A

The _______ is associated with appearance of optic vesicles and includes the telencephalon & diencephalon (A) prosencephalon (B) mesencephalon (C) midbrain (D) rhombencephalon

variable (Fab)

The _______ region of antibodies, which is made of regions from 1 heavy & 1 light chain function as the antigen-binding site

A

The ________ is made up of unipolar neurons & capsular (satellite) cells (A) posterior root ganglion (B) anterior root ganglion (C) lateral root ganglion (D) inferior root ganglion

D

The _________ is for the detection of specific RNA sequences in complex samples (A) southern blot (B) western blot (C) eastern blot (D) northern blot

C

The _________ is used for the detection of specific posttranslational modifications of proteins (A) southern blot (B) western blot (C) eastern blot (D) northern blot

D

The _________ nerve provides parasympathetic innervation to thoracic viscera The _________ nerve provide motor & sensory innervation to the diaphragm & associated membranes (A) glossopharyngeal, phrenic (B) phrenic, glossopharyngeal (C) phrenic, vagus (D) vagus, phrenic (E) glossopharyngeal, vagus

C

The _________ of some hepatic enzymes produces a wide-ranging capability for drug metabolism (A) high substrate specificity (B) polarity (C) low substrate specificity (D) hydrophobicity

C

The ability to identify, isolate, expand, & transplant stem cells has lead to regenerative medicine which involves generating __________ cells that resemble ES cells derived from the patient in whom they will be implanted (A) multipotent (B) totipotent (C) pluripotent (D) tissue stem cells

B

The afterload of the left ventricle = Velocity of shortening ______ after afterload increases (A) arterial pressure, increases (B) aortic pressure, decreases (C) capillary pressure, increases (D) vena cava pressure, decreases

A

The amount of O2 (O2 pressure) is highest in ______ it decreases in tissue capillaries, where exchange takes place between blood and tissues. (A) lung capillaries (B) arterial blood (C) venous blood (D) systemic capillaries

superior, C, F, H

The arch of the aorta is found in the _____ mediastinum and gives rise to which 3 of the following arteries? (A) R pulmonary artery (B) R coronary artery (C) Brachiocephalic artery (D) L coronary artery (E) R common carotid artery (F) L common carotid artery (G) R subclavian artery (H) L subclavian artery

C

The ascending aorta is located in the ___________ mediastinum and gives off the right and left _______ artery (A) middle, brachiocephalic (B) superior, brachiocephalic (C) middle, coronary (D) superior, coronary (E) middle, common carotid (F) superior, common carotid

C

The assumption is that in the steady state, cardiac output of the L and R ventricles are equal. The Fick principle states that there is ___________, so that in the steady state, rate of O2 consumption by body must equal amount of O2 leaving the lungs in the pulmonary vein MINUS amount of O2 returning to lungs in pulmonary artery (A) Balancing of gases (B) conservation of energy (C) conservation of mass (D) uniform resistance

A

The azurophilic granules of ________ contain proteases & antibacterial proteins, including MPO, lysozyme, and defensins (A) neutrophils (B) lymphocytes (C) eosinophils (D) basophils (E) monocytes

C

The basement membrane is the specialized & organized ECM around epithelial cells, endothelial cells, & smooth muscle cells synthesized by (A) pluripotent stem cells (B) mesenchymal cells (fibroblasts) (C) overlying epithelium & underlying mesenchymal cells (D) laminin

D

The blood volume of ________ is stressed volume, while the blood volume of ________ is unstressed volume (A) veins, arteries (B) capillaries, arteries (C) arteries, capillaries (D) arteries, veins

B

The bone matrix is 90% (A) hydroxyapatite (B) collagen type I (C) collagen type IV (D) fibroblasts

D

The buffy coat is a thin layer of leukocytes and platelets between the supernatant plasma and RBCs in hematocrit. The buffy coat is important because it allows you to (A) study the leukocytes (B) diagnose leukemia (C) identify parasites (D) all of the above

B

The calyces, renal pelvis, ureters, & urinary bladder are lined by urothelium (__________ epithelium), which protects underlying cells from hypertonic, potentially toxic effects of urine (A) simple squamous (B) transitional (C) pseudostratified columnar (D) simple cuboidal

conducting system

The cardiac plexus receives superior, middle, & inferior cervical & thoracic cardiac nerves from sympathetic trunks & vagus nerves. It is divided into the superficial & deep cardiac plexus and innervates the _________ _______ of the heart

sympathetic, parasympathetic

The cardiac plexus supplies the heart with: (sympathetic/parasympathetic) fibers which INCREASE HR & force of heart beat and dilate coronary arteries (sympathetic/parasympathetic) fibers which DECREASE HR and constrict coronary arteries

increase

The carotid sinus baroreceptor response to a DECREASE in Pa is to DECREASE the parasympathetic outflow to heart which leads to (increase/decrease) in heart rate

1. increase, increase 2. increase, increase 3. increase, increase 4. increase, stressed

The carotid sinus baroreceptor response to a DECREASE in Pa is to INCREASE the sympathetic outflow which leads to 1. (increase/decrease) HR → (increase/decrease) cardiac output 2. (increase/decrease) contractility & stroke volume → (increase/decrease) cardiac output 3. (increase/decrease) vasoconstriction of arterioles → (increase/decrease) in TPR 4. (increase/decrease) in vasoconstriction of veins → increase in (stressed/unstressed) V

C

The cavity in the mesenchyme that fills with CSF becomes the ______ (A) cerebral aqueduct (B) 4th ventricle (C) subarachnoid space (D) ligamentum denticulatum

D

The cerebral aqueduct develops from a cavity in what part of the brain? (A) cerebellum (B) pons (C) medulla oblongata (D) midbrain (mesencephalon)

C

The change indicated by the dashed lines on the cardiac output/venous return curves shows (A) decreased cardiac output in the "new" steady state (B) decreased venous return in the "new" steady state (C) increased mean systemic pressure (D) decreased blood volume (E) increased myocardial contractility

C

The characteristic tissue-specific patterns of expression of ______ help identify cell origin of poorly differentiated tumors (A) Actin (B) Myosin (C) Intermediate filaments (D) Microtubules

excitatory, inhibitory

The commonality among all types of seizure disorders is the disrupted balance between ______ drive (via gluamatergic signaling) & ________ drive (via GABAergic signaling) at the synaptic level → seizure activity

C

The dashed line in the figure illustrates the effect of (A) increased total peripheral resistance(TPR) (B) increased blood volume (C) increased contractility (D) a negative inotropic agent (E) increased mean systemic pressure

A

The double-bubble sign on baby girl weisen's chest x-ray showed (A) duodenal atresia (B) blood in the stomach (C) esophageal atresia (D) enlarged heart

ligamentum arteriosum

The ductus arteriosus becomes the _______ ________, which connects the L pulmonary artery to the concavity of the arch of the aorta

1. pharyngoesophageal 2. thoracic 3. diaphragmatic

The esophagus extends behind the trachea and has 3 constrictions: 1. upper or ________ at the beginning of the esophagus at level of cricoid cartilage 2. middle or _____ constriction where it is crossed by the aortic arch & L main bronchus 3. inferior or ______ constriction at the esophageal hiatus of the diaphragm (T10)

C

The filum terminale is an extension of _____ (A) dura mater (B) arachnoid mater (C) pia mater (D) spinal nerve roots

A

The first committed step in the pathway of cholesterol synthesis is the formation of __________ (A) mevalonic acid (B) HMG-CoA reductase (HMGR) (C) Farnesyl pyrophosphate (D) Squalene

A

The first heart sound corresponds to point (A) 1 (B) 2 (C) 3 (D) 4

A, D

The first step in metabolism of carbon skeletons of AAs is loss of amino groups, followed by either _________ such as for alanine, glutamate, & asparate OR __________ such as for branched-chain or aromatic AAs (A) direct metabolism in a central pathway (B) breakdown in urea cycle in liver (C) metabolized in kidney to be excreted (D) 1 or more intermediate conversions to yield a metabolite in one of the central pathways

B

The following are signs of _______ 1. Severe nausea, vomiting, & diarrhea leading to dehydration 2. Hypotension & tachycardia 3. Tachypnea (A) chronic adrenal insufficiency (B) acute adrenal insufficiency (adrenal crisis) (C) Cushing disease (D) overactive adrenal gland

F

The following is the plasma marker of impairment for the function of? Altered biological half-time of a drug (A) Heme catabolism (B) Carbohydrate metabolism (C) Protein synthesis (D) Protein catabolism (E) Lipid metabolism (F) Drug metabolism (G) Bile acid metabolism

D

The following is the plasma marker of impairment for the function of? ↑ Ammonia ↓ Urea (A) Heme catabolism (B) Carbohydrate metabolism (C) Protein synthesis (D) Protein catabolism (E) Lipid metabolism (F) Drug metabolism (G) Bile acid metabolism

A

The following is the plasma marker of impairment for the function of? ↑ Bilirubin (A) Heme catabolism (B) Carbohydrate metabolism (C) Protein synthesis (D) Protein catabolism (E) Lipid metabolism (F) Drug metabolism (G) Bile acid metabolism

G

The following is the plasma marker of impairment for the function of? ↑ bile acids (A) Heme catabolism (B) Carbohydrate metabolism (C) Protein synthesis (D) Protein catabolism (E) Lipid metabolism (F) Drug metabolism (G) Bile acid metabolism

E

The following is the plasma marker of impairment for the function of? ↑ triglycerides ↑ cholesterol (A) Heme catabolism (B) Carbohydrate metabolism (C) Protein synthesis (D) Protein catabolism (E) Lipid metabolism (F) Drug metabolism (G) Bile acid metabolism

C

The following is the plasma marker of impairment for the function of? ↓ albumin prolonged prothrombin (PT) time (A) Heme catabolism (B) Carbohydrate metabolism (C) Protein synthesis (D) Protein catabolism (E) Lipid metabolism (F) Drug metabolism (G) Bile acid metabolism

B

The following is the plasma marker of impairment for the function of? ↓ glucose (A) Heme catabolism (B) Carbohydrate metabolism (C) Protein synthesis (D) Protein catabolism (E) Lipid metabolism (F) Drug metabolism (G) Bile acid metabolism

B

The following statement concern the neural crest cell: (A) They are formed from the medial margin of the neural plate (B) They give rise to the posterior root ganglia. (C) They do not form the neurons of the autonomic ganglia. (D) The Schwann cells of peripheral nerves are not formed from neural crest cells. (E) They form the cells of the suprarenal cortex.

A

The following statement concerns the condition of spina bifida: (A) It is one of the more common congenital anomalies of the central nervous system. (B) The most common form of spina bifida is syringomyelocele. (C) The condition occurs most often in the cervical and upper thoracic regions. (D) In a myelocele, the neural tube closes in the region of the defect. (E) Most cases of spina bifida occulta require explorative surgery

C

The following statement concerns the developing spinal cord: (A) The alar plates form the neurons in the anterior gray columns. (B) The nerve cells of the sympathetic outflow are not formed from the basal plates. (C) In the adult, the lower end of the spinal cord lies at the level of the lower border of the first lumbar vertebra. (D) At birth, the lower end of the spinal cord lies at the level of the third sacral vertebra. (E) The meninges surrounding the spinal cord are developed from the endoderm

E

The following statement concerns the development of myelination in the brain: (A) Myelination begins at birth. (B) The sensory fibers are myelinated last. (C) The process of myelination is haphazard. (D) Myelination of the nerve tracts is largely complete by the fourth year of life. (E) Myelination is carried out by oligodendrocytes and not by neurons.

A

The following statement concerns the development of the brainstem: (A) The cerebellum is formed from the dorsal part of the alar plates of the metencephalon. (B) The neurons of the deep cerebellar nuclei are derived from the matrix cells lining the cavity of the midbrain vesicle. (C) The neuroblasts in the dorsal plates will form the nuclei of the trochlear and oculomotor nerves. (D) The neuroblasts of the superior and inferior colliculi are also formed from the neurocytes in the basal plates. (E) The pons arises from the alar part of the metencephalon with cellular contributions from the alar part of the myelencephalon

E

The following statement concerns the neural tube: (A) It is lined by stratified squamous cells (B) The neuroblasts migrate medially to form the inter- mediate zone (C) The repeated division of the matrix cells does not increase the length of the diameter of the tube (D) The ventricular zone will form the gray matter of the spinal cord. (E) The nerve fibers in the marginal zone become myelinated and form the white matter of the spinal cord.

D

The glomerular filter has 3 parts: 1. ________ capillary endothelium 2. Thick _____________ 3. ________ b/w pedicels covered by thin filtration slit diaphragms (A) fenestrated, podocyte layer, occluding junctions (B) continuous, mesangial cell layer, gap junctions (C) sinusoidal, stratified epithelial layer, mesangial cells (D) fenestrated, fused basal laminae, slit pores

E

The greatest pressure decrease in the circulation occurs across the arterioles because (A) they have the greatest surface area (B) they have the greatest cross-sectional area (C) the velocity of blood flow through them is the highest (D) the velocity of blood flow through them is the lowest (E) they have the greatest resistance

D

The law of Laplace states that pressure correlates directly with _____ and wall thickness correlates inversely with _______ (A) tension, tension (B) tension, pressure (C) radius, tension (D) tension, radius

C

The liver plays a central role in ____________, specifically in maintaining the circulating concentration of glucose (A) gluconeogenesis (B) glycolysis (C) glucose metabolism (D) oxidative phosphorylation

A

The low-resistance pathways between myocardial cells that allow for the spread of action potentials are the (A) gap junctions (B) T tubules (C) sarcoplasmic reticulum (SR) (D) intercalated disks (E) mitochondria

fibrinogen

The main difference between plasma and serum is the absence of _______ in serum

B

The main limitation of hormone replacement therapy for postmenopausal women involves (A) expense & time required (B) temporal related treatments that are short-term in use (C) long-term development of tolerance so therapy no longer works (D) short-term side effects including cardiovascular disease, breast cancer, & dementia

B

The main mineralcorticoid is _______. Angiotensin II stimulates its synthesis not ACTH like with glucocorticoids (A) cortisol (B) aldosterone (C) prednisolone (D) estrogen

B

The major lubricant for diarthrotic joints is synthesized by cells located in which joint structure? (A) Nucleus pulposus (B) Synovial membrane (C) Articular cartilage (D) Annulus fibrosus (E) Fibrous capsule

B

The medial group of motor neurons leaves the anterior surface of the spinal cord and travels to muscles. The lateral group forms the lateral gray horn, which is responsible for (A) parasympathetic outflow (B) sympathetic outflow (C) somatic outflow (D) reflexes

B, C, E, H, I

The middle mediastinum is located between the right & left pleural cavities and contains which of the following? (select all that apply - 5) (A) Thoracic duct (B) Arch of azygous vein (C) Main bronchi (D) Esophagus (E) Roots of great vessels (F) Trachea (G) Thymus (H) Pericardium & Heart (I) Phrenic nerves (J) Lungs (K) Brachiocephalic veins

anterior interventricular, LAD

The most common artery blocked in MI is the ______ _______ artery, aka _____ which is a branch of the left coronary artery

D

The neural retina is derived from the (A) alar plate (B) choroid (C) neural crest (D) neural tube (E) telencephalic vesicle wall

descending aorta azygous & hemiazygous veins thoracic duct esophagus sympathetic trunk/ganglia

The posterior mediastinum is posterior to the pericardium b/w the mediastinal pleura and contains what? (DATES)

B

The precapillary sphincter is a band of ________ at the junction of arterioles & capillaries (A) hyaline cartilage (B) smooth muscle (C) skeletal muscle (D) fibrocartilage

D

The presence of howell-jolly bodies on a blood smear is seen in Sickle Cell anemia due to ______ not functioning (A) kidney (B) liver (C) lungs (D) spleen

A

The primary cause of trisomy 21 (Down Syndrome) is (A) maternal nondisjunction (B) sexually inherited disorder (C) paternal nondisjunction (D) inborn error of metabolism

C

The principal progestational hormone secreted into the bloodstream is (A) estrone (B) testosterone (C) progesterone (D) estradiol

right ventricle

The pulmonary trunk originates from the conus arteriosus of the ______ ______ and bifurcates into the right and left pulmonary arteries in the concavity of the aortic arch

A

The pulmonary valve is most audible (A) Over the medial end of the second left intercostal space (B) Over the medial end of the second right intercostal space (C) In the left fourth intercostal space at the midclavicular line (D) In the left fifth intercostal space at the mid- clavicular line (E) Over the right half of the lower end of the body of the sternum

B

The rate-limiting enzyme in the pathway is (A) squalene synthase (B) HMG-CoA reductase (HMGR) (C) oxidosqualene cyclase (D) 25 hydroxylase

pharmacokinetics, pharmacodynamics

The response to any particular drug is influenced by the drug's kinetic properties _________ and its effects _________

E

The right AV valve is most audible (A) Over the medial end of the second left intercostal space (B) Over the medial end of the second right intercostal space (C) In the left fourth intercostal space at the midclavicular line (D) In the left fifth intercostal space at the mid- clavicular line (E) Over the right half of the lower end of the body of the sternum

B

The right and left pulmonary veins drain into the: (A) Right atrium (B) Left atrium (C) Pulmonary trunk (D) Coronary sinus (E) Left ventricle

B

The right margin of the heart is formed by the ______________ (A) Left atrium (B) Right atrium (C) Left ventricle (D) Right ventricle (E) Arch of aorta

spinal cord

The rostral end of the neural tube elongates during development to become the ______ ________

B

The sliding filament theory states that when cross bridges between _____ & ______ form and then break, thick & thin filaments move past each other (cross bridge cycling) → tension produced (A) troponin C & Ca2+ (B) actin & myosin (C) actin & titin (D) myosin & tropomyosin

B

The slope of the vascular curve is determined by (A) total blood volume (B) resistance of arterioles (C) pressure gradient across vasculature (D) rate of atrial contraction

C

The specific granules of _________ contain major basic proteins, which are arginine-rich factors giving granules their acidophilia (A) neutrophils (B) lymphocytes (C) eosinophils (D) basophils (E) monocytes

C

The superior vena cava is formed by the union of_________ (A) Left and right internal thoracic veins (B) Left and right subclavian veins (C) Left and right brachiocephalic veins (D) Left and right internal jugular veins

E, D

The sympathetic nervous system generates heat by stimulating (1) ____ which increases metabolic rate & (2) ____ which causes vasoconstriction to reduce blood flow to skin surface & reduce heat loss (A) anterior hypothalamus (B) posterior hypothalamus (C) Na+-K+ ATPase (D) alpha 1 receptors in vascular smooth muscle of skin blood vessels (E) beta receptors in brown fat

positive, negative

The sympathetic nervous system has a (positive/negative) effect on contractility (ionotropic effect) of myocardium The parasympathetic nervous system has a (positive/negative) effect on contractility (ionotropic effect) of atria

B

The sympathetic nervous system increases conduction velocity by acting on the ____ receptor to increase ____ → increased inward current → increased conduction velocity; it also shortens ERP (A) beta 1, If (B) beta 1, ICa (C) M2, If (D) beta 2, ICa

A

The sympathetic system _______HR by acting on the beta 1 receptor to (A) increase HR, increase If & ICA (B) decrease HR, decrease If only (C) increase HR, increase If only (D) decrease HR, decrease If & ICA

D

The synthesis of bile acids & steroid hormones from cholesterol involves several hydroxylation reactions catalyzed by _________ (A) HMG-CoA reductase (B) Liver X receptors (LXRs) (C) cholestyramine (D) cytochrome P450 monoxygenases

C

The tendency for blood flow to be turbulent is increased by (A) increased viscosity (B) increased hematocrit (C) partial occlusion of a blood vessel (D) decreased velocity of blood flow

B

The tendency for edema to occur will be increased by (A) arteriolar constriction (B) increased venous pressure (C) increased plasma protein concentration (D) muscular activity

C

The thoracic duct opens into the junction between ________________ (A) Right internal jugular vein and right subclavian vein (B) Left and right pulmonary arteries (C) Left internal jugular vein and left subclavian vein (D) Left and right pulmonary veins

B

The urea cycle is a hepatic pathway for disposal of excess ________ (A) iron (B) nitrogen (C) copper (D) potassium

nitrogen

The urea cycle is essential for removal of ________ generated by AA metabolism

D

The urea cycle is split b/w the _________ & the _______ (A) ER & golgi (B) SER & plasma membrane (C) cytosol & RER (D) mitrochondrial matrix & cytosol

B

The vascular function (venous return) curve depicts a relationship b/w blood flow through vascular system (venous return) and (A) L atrial P (B) R atrial P (C) L ventricular P (D) L ventricular V

D

The ventricles are completely depolarized during which isoelectric portion of the electrocardiogram (ECG)? (A) PR interval (B) QRS complex (C) QT interval (D) ST segment (E) T wave

D

The wall of the neural tube is made up of ________ epithelial cells known as matrix cells (A) simple squamous (B) simple cuboidal (C) stratified squamous (D) pseudostratified columnar

B

The x-axis in the figure could have been labeled (A) end-systolic volume (B) end-diastolic volume (C) pulse pressure (D) mean systemic pressure (E) heart rate

C

Therapeutic use of ______: -Generalized tonic-clonic seizures & both simple & complex partial seizures -Lacks dysmorphic side effects common to phenytoin -Also used for trigeminal neuralgia (A) Hydrantoins - Phenytoin (B) Barbiturates - Phenobarbital (C) Carbamazepine (D) Valproic Acid (E) Succinimides - Ethosuximide

A

To determine axis of deviation on an ECG, you must look at the QRS complex of lead 1 and AVF to determine if its mostly upright or mostly downward. If both are pointed upward, it is _________. If lead 1 QRS (R thumb) is up and AVF QRS (L thumb) is down, it is _________. If lead 1 QRS (R thumb) is down and AVF QRS (L thumb) is up, it is ________ (A) normal, left axis deviation, right axis deviation (B) normal, right axis deviation, left axis deviation (C) right axis deviation, left axis deviation, normal (D) left axis deviation, right axis deviation, normal

B

Transports lipids and some metal ions (A) albumin (B) alpha-globulins (C) beta-globulins (D) gamma-globulins (E) fibrinogen

red line, red flare, wheal

Trauma to skin results in the "triple response", which includes what 3 things?

A

Treated w/ large doses of glucocorticoids for long periods → (A) decreased resistance to infection & poor wound healing response (B) liver disease & hepatosplenomegaly (C) ulcerative colitis (D) upset stomach & skin irritation

arteries, vein

Umbilical (arteries/veins) become medial umbilical ligaments after birth The right is obliterated during embryonic period, but the left umbilical (artery/vein) forms the ligamentum teres hepatis after birth

D

Under conditions of homeostasis, what 2 properties characterize stem cells? (A) self-renewal (B) surface receptors (C) Division/differentiation (D) A & C (E) All of the above

D

Unlike integral or peripheral membrane proteins, GPI-anchored proteins may be released from the cell surface by ________ in response to regulatory processes (A) LPLAT (B) PLA2 (C) DAG (D) phospholipase C

B

Uses inactivated pathogens (A) live attenuated vaccines (B) inactivated vaccines (C) subunit vaccines (D) all of the above

C

Uses molecular components of bacteria or viruses as vaccines, including: -Capsular polysaccharides & surface proteins of bacteria -Viral envelope glycoproteins (A) live attenuated vaccines (B) inactivated vaccines (C) subunit vaccines (D) all of the above

B

V regions in the L & H chains form a functional pocket where epitope fits, known as the _________ (A) constant region (Fc) (B) antibody recognition region (Fab) (C) variable (V) region (D) J chain

Haversian Canal (Central Canal)

VERTICAL contain blood vessels & nerves

A

Vascular tufts of ______, which is a monolayer of ependymal cells of the roof plate covered with pia mater, project into the 4th ventricle to form the choroid plexus (A) tela choroidea (B) conus medullaris (C) foramina of luschka (D) hypoglossal nucleus

A

Volume of blood ejected by ventricle on each beat =end-diastolic volume - end-systolic volume (A) stroke volume (B) ejection fraction (C) cardiac output (D) afterload

out, in

Water moves in and out by osmosis depending on relative solute concentrations HYPERtonicity → net movement of water ______ HYPOtonicity → net movement of water ______

C

What 3 factors cause blood vessels to sickle? (Can be treated with O2 and fluids) (A) hyperoxia, dehydration, alkalosis (B) hypoxia, excessive hydration, alkalosis (C) hypoxia, dehydration, acidosis (D) trauma, dehydration, acidosis (E) hypoxia, trauma, dehydration

C

What are general therapeutic uses of Adrenal Glucocorticosteroids? I. Replacement in adrenal insufficiency II. Suppress acute or chronic inflammation III. Used in dentistry to reduce signs & symptoms of excess inflammatory reactions IV. Cough suppressant (A) I & II (B) II & III (C) I, II, & III (D) I, II, III, & IV

fragments of bone marrow megakaryocytes

What are platelets derived from?

polar, conjugation

What are the 2 phases of drug metabolism by hepatic enzymes? 1. addition of a _______ group 2. _______

azurophilic and specific

What are the 2 types of granules found in granulocytes?

RBCs, WBCs, platelets

What are the 3 major cell lineages among blood cells?

modified AAs, peptides, glycoproteins, steroids

What are the 4 structures hormones can be?

Phenylalanine, Valine, Threonine Tryptophan, Isoleucine, Methionine Histidine, Arginine, Leucine, Lysine

What are the essential AAs? (PVT TIM HALL)

B

What are the most frequent seizures in children and what causes them? (A) Petit mal, inherited disorder (B) Febrile seizures, fever (C) Grand Mal, toxins (D) Partial, congenital defect

bone marrow & thymus

What are the primary lymphoid organs?

D

What cell in circulating blood is the precursor to microglia and most antigen-presenting cells? (A) Eosinophil (B) Basophil (C) Lymphocyte (D) Monocyte (E) Mast cell

E

What contains the highest proportion of the blood in the cardiovascular system? (A) arterioles (B) arteries (C) capillaries (D) venules (E) veins

direction of pressure gradient

What determines the direction of blood flow (Q)?

Toxoplasma Other (syphillis) Rubella Cytomegalovirus Herpes Virus

What does TORCH stand for?

increase, increase

What effect do the following changes have on reynolds number (NR)? 1. decrease in hematocrit → decrease in viscosity → 2. narrowing of vessel → increase in velocity →

B

What ensures accurate DNA replication before cell division? (A) G1-S checkpoint (B) G2-M checkpoint (C) CDK inhibitors (D) cyclins

B

What enzyme trims pre-miRNA to form mature double-stranded miRNA with 21 to 30 nucleotides? (A) RISC (B) Dicer (C) XIST (D) CRISPR

E

What factor(s) determine blood flow (Q) (A) blood type (B) pressure difference between 2 ends of the vessel (C) Resistance of vessel to blood flow (D) A & C (E) B & C

B

What fate often awaits granulocytes that have entered the marginating compartment? (A) Undergo mitosis (B) Crossing the wall of a venule to enter connective tissue (C) Cannot reenter the circulation (D) Differentiate into functional macrophages (E) Begin to release platelets

estrogen

What hormone causes the LH surge during ovulation?

B

What is a possible reason for Scott's false negative on the Guthrie test at birth? (A) baby acquired PKU later (B) baby was nursing (instead of formula) (C) baby was on formula (instead of nursing) (D) he had a mutation that does not allow for detection of PKU by the heel test

D

What is responsible for the following response changes in pCO2 or pH → stimulate medullary chemoreceptors → changes in outflow of medullary cardiovascular centers (A) Renin-Angiotensin II-Aldosterone System (B) Baroreceptors in carotid sinus (C) Peripheral Chemoreceptors in carotid & aortic bodies (D) Central Chemoreceptors

C

What is responsible for the following response: ↓ in arterial PO2 → increased firing rate from afferent nerves → arteriolar vasoconstriction & ↑ in parasympathetic outflow to heart → ↑ HR (A) Renin-Angiotensin II-Aldosterone System (B) Baroreceptors in carotid sinus (C) Peripheral Chemoreceptors in carotid & aortic bodies (D) Central Chemoreceptors

D

What is the 1st vaccine given soon after birth? (A) HPV vaccine (B) MMR (C) Diphtheria (D) HBV vaccine (E) Varicella Zoster

D

What is the approximate life span of a circulating erythrocyte? (A) 8 days (B) 20 days (C) 5 weeks (D) 4 months (E) 1 year

A

What is the earliest stage at which specific granulocyte types can be distinguished from one another? (A) Myelocyte (B) Band form (C) Reticulocyte (D) Metamyelocyte (E) Promyelocyte

A

What is the first bone tissue to appear in embryonic development and in fracture repair? (A) woven bone (B) lamellar bone (C) compact (cortical) bone (D) cancellous (trabecular) bone

D

What is the lifespan of neutrophils? (A) about 10 days (B) 13 to 20 days (C) 120 days (D) 1 day

A

What is the lifespan of platelets? (A) about 10 days (B) 13 to 20 days (C) 120 days (D) 1 day

E

What is the longest phase of the SA node which accounts for its automaticity? (A) phase 0 (B) phase 1 (C) phase 2 (D) phase 3 (E) phase 4

B

What is the main difference between partial and generalized seizures? (A) partial comes with less symptoms and is easier to treat than generalized (B) partial involves one side of brain at onset, whereas generalized involves both sides (C) generalized affects more of the population than partial (D) partial originates in the frontal lobe while generalized originates in the brainstem

B

What is the principal function of cutaneous sympathetic nerves? (A) pain regulation (B) temperature regulation (C) pressure regulation (D) volume regulation

A

What is the site of highest resistance in the cardiovascular system? (A) arterioles (B) arteries (C) capillaries (D) venules (E) veins

D

What is usually used as a first-line screen for thyroid diseased? (A) serum fT3 (B) serum TRH (C) serum fT4 (D) serum TSH

B

What layer of the adrenal cortex produces glucocorticoids? (A) Zona glomerulosa (outer) (B) Zona fasciculata (middle) (C) Zona reticularis (inner) (D) Entire adrenal cortex

A

What leads to the changes detailed in the chart? (A) Hemorrhage (B) Exercise (C) Standing (D) All of the above

B

What leads to the changes detailed in the chart? (A) Hemorrhage (B) Exercise (C) Standing (D) All of the above

C

What leads to the changes detailed in the chart? (A) Hemorrhage (B) Exercise (C) Standing (D) All of the above

status epilepticus

What might develop in the patients with the following: 1. with convulsive disorders 2. with acute disease affecting brain 3. after abrupt withdrawal of depressant or antiseizure meds 4. After local anesthesia administration

A

What monitors integrity of DNA before DNA replication? (A) G1-S checkpoint (B) G2-M checkpoint (C) CDK inhibitors (D) cyclins

D

What produces the greatest response in carotid sinus baroreceptors given their sensitivity to this change? (A) changes in blood volume (B) changes in hormone balance (C) changes in PO2 (D) changes in Pa (mean arterial pressure)

C

What regulates blood flow through capillaries? (A) contraction & relaxation of atria (B) hormone secretion (C) contraction & relaxation of arterioles & precapillary sphincters (D) Hematopoiesis

B

What type of capillaries are found in red bone marrow? (A) fenestrated (B) sinusoidal (C) continuous

adult stem cells

What type of cell is described below? -Associated with differentiated cells of a given tissue -Found in stem cell niches -Can only produce cells that are normal constituents of the tissue in which they reside (AKA tissue stem cells)

embryonic stem cells

What type of cell is described below? -Mostly undifferentiated -Inner cell mass of blastocysts -Limitless cell renewal capacity -Can give rise to every cell in the body (totipotent)

E

What type of epithelium lines the prostatic urethra? (A) Simple columnar (B) Pseudostratified columnar (C) Stratified squamous (D) Simple squamous (E) Transitional (urothelium)

B

What type of epithelium lines the urethra in both genders? (A) simple squamous (B) transitional (urothelium) (C) pseudostratified columnar (D) simple cuboidal

D

What would a bulging anterior fontanel indicate? (A) spina bifida (B) myelomeningocele (C) normocephalus (D) hydrocephalus

B

When AP septum fails to develop in a spiral fashion → aorta arises from R ventricle & pulmonary trunk arises from L ventricle → right-to-left shunting of blood & cyanosis (A) Tetralogy of Fallot (B) Transposition of great vessels (C) Atrial Septal Defect (ASD) (D) Ventricular Septal Defect (VSD)

A

When RBCs are hemolyzed, Hb is released into plasma and dissociates into dimers that bind to _______, forming a complex metabolized in the liver and reticuloendothelial system faster than the protein alone (A) haptoglobin (B) myoglobin (C) albumin (D) TBG

B

When a person moves from a supine position to a standing position, which of the following compensatory changes occurs? (A) Decreased heart rate (B) Increased contractility (C) Decreased total peripheral resistance (TPR) (D) Decreased cardiac output (E) Increased PR intervals

higher, turbulent

When a thrombi occurs, there is a blood clot in the vessel → narrowed diameter → decrease in velocity →(lower/higher) NR & (laminar/turbulent) blood flow

↑ ↑ ↑ ↑ ↑ ↑

When an individual moves from a supine to standing position, compensatory mechanisms will attempt to increase BP to normal, carotid sinus baroreceptors will decrease firing rate of carotid sinus nerves →→ -arterial BP (↑/↓) -cardiac output (↑/↓) -stroke volume (↑/↓) -Central venous pressure (↑/↓) -HR (↑/↓) -TPR (↑/↓)

↓ ↓ ↓ ↓

When an individual moves from a supine to standing position, the initial response is: -arterial BP (↑/↓) -cardiac output (↑/↓) -stroke volume (↑/↓) -Central venous pressure (↑/↓)

increased, decreased, increased

When carotid sinus baroreceptors detect a decrease in Pa as a result of acute blood loss (hemorrhage) → (increased/decreased) sympathetic outflow to heart & blood vessels & (increased/decreased) parasympathetic outflow to heart → (increased/decreased) HR, contractility, TPR, & venous return → return Pa to normal

1. increase, increase 2 decrease, decrease

When carotid sinus baroreceptors detect: 1. (decrease/increase) in Pa → INCREASED stretch → ((decrease/increase) firing rate in afferent nerve 2. (decrease/increase) in Pa → DECREASED stretch → ((decrease/increase) firing rate in afferent nerve

low, high

When cholesterol concentration is ________, the SCAP/SREBP complex dissociates from Insig-1 & tavels to golgi When cholesterol concentration is _______, cholesterol binding to SCAP stabilizes SCAP/SREBP/Insig-1 complex to block its movement to golgi → decrease in nuclear SREBP & transcription stays repressed → cholesterol synthesis inhibited

B

When testing using the Vasalva maneuver, if baroreceptor reflex is intact, a decrease in Pa will be sensed by baroreceptors → increase in sympathetic outflow to heart and blood vessels and _______ would be noted. When the maneuver is stopped, there is a rebound increase in venous return, cardiac output, & Pa. Increase in Pa is sensed by the baroreceptors and a _______ would be noted (A) increased breathing rate, decreased breathing rate (B) increased HR, decreased HR (C) spike in blood pressure, drop in blood pressure (D) heart murmur, return to normal heart sounds

A

Where proteins & lipids are sent to other organelles or to secretory vesicles (A) trans golgi network (B) RER (C) cis golgi network (D) lysosomes

C

Where proteins are recycled back to ER (A) trans golgi network (B) RER (C) cis golgi network (D) lysosomes

D

Which "zone" of endochondral ossification in the growing femur of an adolescent is the farthest from that bone's secondary ossification center? (A) Zone of hypertrophy (B) Zone of reserve cartilage (C) Zone of calcified cartilage (D) Zone of ossification (E) Zone of proliferation

A

Which agent is released or secreted after a hemorrhage and causes an increase in renal Na+ reabsorption? (A) Aldosterone (B) Angiotensin I (C) Angiotensinogen (D) Antidiuretic hormone (ADH) (E) Atrial natriuretic peptide

A

Which antibody can go to fetal circulation in pregnant women? (A) IgG (B) IgM (C) IgA (D) IgD (E) IgE

C

Which antimicrobial chemical of innate immunity is described below? enzyme made by neutrophils & cells of epithelial barriers, which hydrolyzes bacterial cell wall components, killing those cells (A) Hydrochloric acid (HCl) & organic acids (B) Defensins (C) Lysozyme (D) Complement (E) Interferons

A

Which antimicrobial chemical of innate immunity is described below? lower the pH locally to either kill entering microorganisms directly or inhibit their growth (A) Hydrochloric acid (HCl) & organic acids (B) Defensins (C) Lysozyme (D) Complement (E) Interferons

E

Which antimicrobial chemical of innate immunity is described below? paracrine factors from leukocytes & virus-infected cells that signal NK cells to kill such cells & adjacent cells to resist viral infection (A) Hydrochloric acid (HCl) & organic acids (B) Defensins (C) Lysozyme (D) Complement (E) Interferons

B

Which antimicrobial chemical of innate immunity is described below? short cationic polypeptides produced by neutrophils & various epithelial cells that kill bacteria by disrupting the cell walls (A) Hydrochloric acid (HCl) & organic acids (B) Defensins (C) Lysozyme (D) Complement (E) Interferons

D

Which antimicrobial chemical of innate immunity is described below? system of proteins in blood plasma, mucus, & macrophages that react w/ bacterial surface components to aid removal of bacteria (A) Hydrochloric acid (HCl) & organic acids (B) Defensins (C) Lysozyme (D) Complement (E) Interferons

B

Which biochemical component of the erythrocyte cell surface is primarily responsible for determining blood type (eg, the A-B-O system). (A) Fatty acid (B) Carbohydrate (C) Nucleic acid (D) Protein (E) Cholesterol

D

Which cell is a modified smooth muscle cell that secretes renin? (A) Macula densa cells (B) Mesangial cells (C) Podocytes (D) Juxtaglomerular cells (E) Endothelial cells

D

Which cell type comprises the visceral layer of Bowman capsule? (A) Endothelial cells (B) Juxtaglomerular cells (C) Mesangial cells (D) Podocytes (E) Extraglomerular mesangial (or Lacis) cells

A

Which cell type gives rise to both memory and effector cells and is primarily associated with humoral immunity? (A) B lymphocyte (B) NK cell (C) Macrophage (D) T lymphocyte (E) Reticular cell

B

Which cell type has cytoplasmic granules that contain heparin and histamine? (A) Eosinophils (B) Basophils (C) Lymphocytes (D) Monocytes (E) Neutrophils

D

Which cell type is capable of further mitosis after leaving the hemopoietic organ in which it is formed? (A) Basophil (B) Eosinophil (C) Reticulocyte (D) Lymphocyte (E) Neutrophil

A

Which component of bone impedes the distribution of nutrients and oxygen to osteocytes? (A) ECM (B) Canaliculi (C) Periosteum (D) Cell processes (E) Haversian canals

A

Which cytokine is described below? Cytokine(s) whose main function is growth and differentiation factors for leukocyte progenitor cells in bone marrow (A) GM-CSF, M-CSF (B) TNF-α, TGF-β, IL-1 (C) IL-12 (D) IL-2, IL-4 (E) IL-5 (F) interferon-γ, IL-4 (G) IL-10 (H) interferon-α, interferon-β (I) IL-8

E

Which cytoplasmic components are the main constituents of the dark precipitate that forms in reticulocytes upon staining with the dye cresyl blue? (A) Golgi complexes (B) Hemoglobin (C) Nucleoli (D) Nuclear fragments (E) Polyribosomes

E

Which description is true of all secondary (peripheral) lymphoid organs? (A) Capable of antigen-independent lymphopoiesis (B) Contain crypts (C) Contain epithelial-reticular cells (D) Lack connective tissue capsules (E) Contain lymphoid nodules

A

Which enzyme is deficient in CAH, such as in the case for baby boy (girl) Colman? (A) 21 α-hydroxylase (B) 17 α-hydroxylase (C) 1 1β-hydroxylase (D) Aromatase (E) 3β-Hydroxysteroid dehydrogenase

A

Which enzyme should be where the blue box is located? (A) 21 α-hydroxylase (B) 17 α-hydroxylase (C) 1 1β-hydroxylase (D) Aromatase (E) 3β-Hydroxysteroid dehydrogenase

E

Which enzyme should be where the green box is located? (A) 21 α-hydroxylase (B) 17 α-hydroxylase (C) 1 1β-hydroxylase (D) Aromatase (E) 3β-Hydroxysteroid dehydrogenase

D

Which enzyme should be where the orange box is located? (A) 21 α-hydroxylase (B) 17 α-hydroxylase (C) 1 1β-hydroxylase (D) Aromatase (E) 3β-Hydroxysteroid dehydrogenase

B

Which enzyme should be where the pink box is located? (A) 21 α-hydroxylase (B) 17 α-hydroxylase (C) 1 1β-hydroxylase (D) Aromatase (E) 3β-Hydroxysteroid dehydrogenase

C

Which enzyme should be where the red box is located? (A) 21 α-hydroxylase (B) 17 α-hydroxylase (C) 1 1β-hydroxylase (D) Aromatase (E) 3β-Hydroxysteroid dehydrogenase

E

Which epiphyseal plate region of activity is described below? -Bone tissue first appears -Capillaries & osteoprogenitor cells invade chondrocytic lacunae -Osteoblasts form layer & secrete osteoid →woven bone→remodeled as lamellar bone (A) Zone of reserve cartilage (B) Proliferative zone (C) Zone of Hypertrophy (D) Zone of calcified cartilage (E) Zone of ossification

C

Which epiphyseal plate region of activity is described below? -Swollen, terminally differentiated chondrocytes -Compress matrix & secrete type X collagen to stiffen it (A) Zone of reserve cartilage (B) Proliferative zone (C) Zone of Hypertrophy (D) Zone of calcified cartilage (E) Zone of ossification

B

Which epiphyseal plate region of activity is described below? Cartilage cells divide repeatedly, enlarge, & secrete more type II collagen & proteoglycans; become organized into columns parallel to long axis of bone (A) Zone of reserve cartilage (B) Proliferative zone (C) Zone of Hypertrophy (D) Zone of calcified cartilage (E) Zone of ossification

D

Which epiphyseal plate region of activity is described below? Chondrocytes about to undergo apoptosis release matrix vesicles & osteocalcin → begin matrix calcification by formation of HA crystals (A) Zone of reserve cartilage (B) Proliferative zone (C) Zone of Hypertrophy (D) Zone of calcified cartilage (E) Zone of ossification

A

Which epiphyseal plate region of activity is described below? typical hyaline cartilage (A) Zone of reserve cartilage (B) Proliferative zone (C) Zone of Hypertrophy (D) Zone of calcified cartilage (E) Zone of ossification

D

Which function is carried out by all lymphoid tissues and organs? (A) Filtration of lymph (B) Filtration of blood (C) Extramedullary hemopoiesis (D) Production of lymphocytes (E) Destruction of old erythrocytes

cortisol

Which hormone has the following actions: 1. negative feedback to hypothalamus & anterior pituitary 2. metabolic homeostasis 3. fluid/electrolyte homeostasis 4. anti-inflammatory/immunosuppressive effects

C

Which hormone is responsible for: -maintenance of pregnancy -decreases hepatic secretion of very LDL (low-density lipoprotein) & HDL -stimulates hypothalamic respiratory center -enhances Na+ excretion by the kidneys (A) estrogens (B) GH (C) Progestins (D) Androgens

A

Which hormone is responsible for: -the development, growth, & maintenance of secondary sex characteristics -uterine growth -pulsatile release of LH from pituitary -thickening of vaginal mucosa (A) estrogens (B) GH (C) Progestins (D) Androgens

C

Which intermediate filament is described below: axons of neurons → strength & rigidity (A) Vimentin (B) Desmin (C) Neurofilaments (D) Glial fibrillary acidic protein (E) Cytokeratins (F) Lamins

E

Which intermediate filament is described below: epithelial cells → histochemical markers for various epithelia (A) Vimentin (B) Desmin (C) Neurofilaments (D) Glial fibrillary acidic protein (E) Cytokeratins (F) Lamins

D

Which intermediate filament is described below: glial cells supporting neurons (A) Vimentin (B) Desmin (C) Neurofilaments (D) Glial fibrillary acidic protein (E) Cytokeratins (F) Lamins

A

Which intermediate filament is described below: mesenchymal cells (fibroblasts, endothelium) anchoring intracellular organelles (A) Vimentin (B) Desmin (C) Neurofilaments (D) Glial fibrillary acidic protein (E) Cytokeratins (F) Lamins

B

Which intermediate filament is described below: muscle cells, form scaffold for myosin-actin contraction (A) Vimentin (B) Desmin (C) Neurofilaments (D) Glial fibrillary acidic protein (E) Cytokeratins (F) Lamins

F

Which intermediate filament is described below: structure of nuclear lamina & help regulate nuclear gene transcription (A) Vimentin (B) Desmin (C) Neurofilaments (D) Glial fibrillary acidic protein (E) Cytokeratins (F) Lamins

D (A-C are encapsulated bacteria; D is caused by a virus)

Which is LEAST risk for infection in sickle cell anemia? (A) streptococcus pneumoniae (B) neisseria meningiditis (C) hemophilus influenze type B (HIB) (D) hepatitis B

E

Which is LEAST suggestive of sickle cell anemia? (A) target cells (B) Howell Jolly Bodies (C) auto-splenectomy (D) vaso-occlusion (E) increased haptoglobin

E

Which is NOT typically screened for in the neonate? (A) PKU (B) Hypothyroid (C) Sickle Cell (D) galactosemia (E) down syndrome

D

Which is correct regarding blood cells? (A) RBCs live 10 days (B) Platelets live 1 day (C) WBCs live 120 days (D) RBCs, WBCs, & platelets have a common ancestry in the bone marrow

A

Which is the best technique to separate oxygenated normal hemoglobin A (HbA) from oxygenated sickle cell hemoglobin (HbS), assuming no protein aggregation? (A) Native gel electrophoresis (B) SDS-PAGE (C) Gel filtration (D) Affinity chromatography with a C-terminal antibody (E) Ultracentrifugation

C

Which is true about a karyotype? (A) does not determine gender (B) if down syndrome, denotes X-linked inheritance (C) if down syndrome, abnormality is due to nondisjunction (D) is tested in all pregnancies (E) is performed via maternal US

D

Which is/are a physical exam sign of a patient with Down Syndrome? (A) Trisomy 21 (B) Polyhydramnios (C) Double bubble (D) excess nuchal skin (E) all of the above

D

Which of the following agents or changes has a negative inotropic effect on the heart? (A) Increased heart rate (B) Sympathetic stimulation (C) Norepinephrine (D) Acetylcholine (ACh) (E) Cardiac glycosides

B, E

Which of the following are agranulocytes? (select all that apply) (A) neutrophils (B) lymphocytes (C) eosinophils (D) basophils (E) monocytes

A, C, D

Which of the following are granulocytes? (select all that apply) (A) neutrophils (B) lymphocytes (C) eosinophils (D) basophils (E) monocytes

D

Which of the following are implications to consider for dentistry when treating patients taking glucocorticoids? (A) decreased resistance to infection (B) poor wound healing (C) suppression of pituitary adrenal function (D) all of the above

D

Which of the following are pathways for macromolecule transport to lysosomes? I. Autophagy II. Phagocytosis III. Fluid-phase pinocytosis IV. receptor-mediated endocytosis (A) I & III (B) II & IV (C) I, II, & IV (D) I, II, III, & IV

C

Which of the following are produced by the adrenal medulla? (A) glucocorticoids (B) mineralcorticoids (C) catecholamines (D) A & B

D

Which of the following blood cells differentiate outside of the bone marrow? (A) Neutrophils (B) Basophils (C) Eosinophils (D) T lymphocytes (E) Megakaryocytes

E

Which of the following can be used to describe megakaryocytes? (A) Multinucleated (B) Formed by fusion of haploid cells (C) Precursors to bone marrow macrophages (D) A minor but normal formed element found in the circulation (E) Possess dynamic cell projections from which one type of formed element is released

A

Which of the following cells move to the site of injury in chemotaxis? (A) PML (B) Lymphocyte (C) T-Lymphocyte (D) Basophil (E) Mast cell

D

Which of the following changes will cause an increase in myocardial O consumption? (A) Decreased aortic pressure (B) Decreased heart rate (C) Decreased contractility (D) Increased size of the heart (E) Increased influx of Na+ during the upstroke of the action potential

A

Which of the following coordinates the mechanism for dissipating heat by shunting blood to venous plexuses near skin surface? (A) anterior hypothalamus (B) posterior hypothalamus (C) Na+-K+ ATPase (D) alpha 1 receptors in vascular smooth muscle of skin blood vessels (E) beta receptors in brown fat

B

Which of the following does NOT contain heme? (A) Hemoglobin (B) Platelets (C) Myoglobin (D) Cytochromes

D

Which of the following does NOT increase catecholamine secretion? (A) ACTH (B) glucocorticoids (C) sympathetic nerve stimulation (D) mineralocorticoids

A, B, C

Which of the following drain into the coronary sinus? (Select all that apply - 3) (A) Middle cardiac vein (B) Small cardiac vein (C) Oblique cardiac vein (D) Anterior cardiac vein (E) Right pulmonary vein

B

Which of the following findings is associated with Gaucher Disease? (A) cherry red macula (B) tissue paper cytoplasm (C) globoid cells (D) X-linked

C

Which of the following increase ADH secretion I. Increases in serum osmolarity II. Decreases in serum osmolarity III. Increases in blood V & BP IV. Decreases in blood V & BP (A) I & III (B) II & IV (C) I & IV (D) II & III

B

Which of the following induces UGT enzymes CYP2C & CYP3A which can affect OCP concentrations and also has toxicity affects of sedation and potentially causing megaloblastic anemia & osteomalacia (A) Hydrantoins - Phenytoin (B) Barbiturates - Phenobarbital (C) Carbamazepine (D) Valproic Acid (E) Succinimides - Ethosuximide

D

Which of the following is NOT a content of the superior mediastinum? (A) Remnants of thymus gland (B) LNs (C) fat & CT (D) SVC

D

Which of the following is NOT a mechanism for removal of amino group from AAs? (A) Transamination (B) Oxidative deamination (C) Removal of a molecule of water by a dehydratase (D) Reductive deamination

B

Which of the following is NOT a secondary lymphoid organ? (A) lymph nodes (B) thymus (C) MALT (D) tonsils (E) Spleen

C

Which of the following is NOT produced by the adrenal cortex? (A) glucocorticoids (B) gonadal hormones (C) catecholamines (D) mineralocorticoids

A

Which of the following is a content of the middle mediastinum? (A) Pericardium (B) Vagus nerve (C) Esophagus (D) Thymus (E) Descending thoracic aorta

E

Which of the following is a length-dependent mechanism that alters tension? (A) Maximal overlap of thick & thin filaments (B) Increasing muscle length which increases Ca2+ sensitivity to troponin C (C) Increasing muscle length → increases Ca2+ release from SR (D) A & B (E) All of the above

B

Which of the following is an effect of histamine? (A) Decreased capillary filtration (B) Vasodilation of the arterioles (C) Vasodilation of the veins (D) Decreased Pc (E) Interaction with the muscarinic receptors on the blood vessels

A

Which of the following is not one of the responses promoted by exposure of Fc portion following antigen binding? (A) Agglutination (B) Complement fixation (C) Opsonization (D) Activation of NK cells

B

Which of the following is the correct path of blood flow through the vascular bed? (A) arteriole → artery → capillaries → vein → venule (B) artery → arteriole → capillaries → venule → vein (C) capillaries → arteriole → artery → venule → vein (D) arteriole → venule → capillaries → veins → arteries

A

Which of the following is the rate limiting step in heme synthesis? (A) glycine + succinyl-coA → 5-ALA (B) 2 5-ALA → PBG (C) 4 PBG → uropophyrinogen III → copropophyrinogen III (D) Decarboxylation & oxidation of sides chains in uropophyrinogen III → protoporphyrin IX (E) ferrochelatase adds Fe2+ to protoporphyrin IX → heme

B

Which of the following is the result of an inward Na+ current? (A) Upstroke of the action potential in the sinoatrial (SA) node (B) Upstroke of the action potential in Purkinje fibers (C) Plateau of the action potential in ventricular muscle (D) Repolarization of the action potential in ventricular muscle (E) Repolarization of the action potential in the SA node

C (saturated at sn-1; unsaturated at sn-2)

Which of the following is true regarding glycerophospholipids & sphingolipids? (A) Saturated fatty acids are esterified at the sn-2 position (B) Unsaturated fatty acids are esterified at the sn-1 position (C) glycerophospholipids are saponifiable, but sphingolipids have no alkaliable ester bonds (D) All of the above

D

Which of the following is true regarding the metabolic hypothesis I. vasodilator metabolites produced as a result of metabolic activity in tissue II. Based on observation that the tissue supply of O2 is matched to the tissue demand for O2 III. based on observation that vascular smooth muscle contracts when it is stretched IV. Explains autoregulation but not active or reactive hyperemia (A) I & III (B) I, II, & IV (C) III & IV (D) I & II

C

Which of the following is true regarding the myogenic hypothesis I. vasodilator metabolites produced as a result of metabolic activity in tissue II. Based on observation that the tissue supply of O2 is matched to the tissue demand for O2 III. based on observation that vascular smooth muscle contracts when it is stretched IV. Explains autoregulation but not active or reactive hyperemia (A) I & III (B) I, II, & IV (C) III & IV (D) I & II

C

Which of the following most accurately describes compact bone? (A) Predominant bone type in the epiphyses of adult long bones (B) Also known as cancellous bone (C) Characterized by the presence of osteons (D) Lines the medullary (marrow) cavity (E) Forms the diploë in cranial bones

D

Which of the following most accurately describes the endosteum? (A) Composed of two layers: osteogenic and fibrous (B) Continuous with the joint capsule (C) Attached to the bone surface by collagen bundles called Sharpey fibers (D) Lines the medullary cavity (E) Contains mature osteocytes

E

Which of the following parameters is decreased during moderate exercise? (A) Arteriovenous O2 difference (B) Heart rate (C) Cardiac output (D) Pulse pressure (E) Total peripheral resistance (TPR)

D

Which of the following substances crosses capillary walls primarily through water-filled clefts between the endothelial cells? (A) O2 (B) CO2 (C) CO (D) Glucose

D

Which of the following use channel & carrier proteins? (A) small, nonpolar molecules (B) hydrophobic molecules (C) Large, polar molecules (D) low MW polar molecules

B

Which of the following would you most likely treat with an injection of prednisolone or dexamethasone ( glucocorticoids)? (A) herpetic ulcers (B) TMJ disorders - refractory cases or severe acute pain (C) angular cheilitis (D) allergic reactions

A

Which of the following would you most likely treat with topical application of glucocorticoids? (A) oral ulcerations, such as denture-induced ulcers or angular cheilitis (B) TMJ disorders - refractory cases or severe acute pain (C) Postoperative sequelae (D) allergic reactions

E

Which phase of the ventricular action potential coincides with diastole? (A) Phase 0 (B) Phase 1 (C) Phase 2 (D) Phase 3 (E) Phase 4

C

Which process occurs during granulopoiesis but not during erythropoiesis? (A) Cells lose their capacity for mitosis (B) Euchromatin content increases (C) Nucleus becomes increasingly lobulated (D) Overall cell diameter decreases (E) Overall nuclear diameter decreases

D

Which receptor mediates slowing of the heart? (A) α1 Receptors (B) β1 Receptors (C) β2 Receptors (D) Muscarinic receptors

C

Which structure is partly encapsulated and covered by nonkeratinized stratified squamous epithelium? (A) Appendix (B) Lymph node (C) Palatine tonsil (D) Peyer's patch (E) Thymic (Hassal's) corpuscle

B

Which structure would be most heavily labeled by an immunohistochemical method targeting the CD8 surface antigen? (A) Germinal centers (B) Paracortex (C) Peyer's patch (D) Sheathed arterioles (E) Splenic cords

D

Which substance is NOT involved in PKU? (A) phenylalanine (B) tyrosine (C) melanin (D) homocystine (E) myelin

C

Which type of epithelium lines the thick ascending limb of the loop of Henle? (A) Pseudostratified columnar (B) Simple columnar (C) Simple cuboidal (D) Simple squamous (E) Transitional (urothelium)

B

Why are the OCP formulations made low-dose? (A) the effects on ovulation do not occur at high doses (B) many of the systemic side-effects are dose-dependent (C) side effects on uterus are dose-dependent (D) easier and cheaper to make

C

Why did Caroline Weisen have an amniocentesis performed? (A) age and chronic condition (B) chronic condition and prior gestation history (C) age and prior gestation history (D) age and current medical condition

C

Why was an orogastric tube placed in baby girl Weisen? (A) fix her right ventricular hypertrophy (B) inflate the stomach (C) decompress the stomach (D) provide nutrition

C

Why would estrogen + progestins be used in hormone replacement therapy in menopausal women? (A) reinitiate menopause (B) initiate ovulation (C) reduce bone loss/fractures (D) increase osteoclast activity

A

Wilson's disease is a condition associated w/ liver & CNS damage; it results from abnormal tissue ______ disposition (A) copper (B) heme (C) iron (D) fat

B

Wound to the heart or pericardial effusion → rapid accumulation of fluid or blood in pericardial cavity → acute compression of heart → reduced cardiac output w/ increased HR (A) MI (B) Cardiac tamponade (C) Pericarditis (D) Pericardial Effusion (E) Pericardiocentesis

peroxisomes

X-linked adrenoleukodystrophy is caused by a defect in ______ → fatty acids not broken down → accumulation of fatty acids in blood → damage to adrenal glands & myelin

spina bifida

____ _____ usually occurs in the sacrolumbar region and results from failure of the posterior neuropore to close

C

____ enforces checkpoints by modulating CDK-cyclin complex activity (A) G1-S checkpoint (B) G2-M checkpoint (C) CDK inhibitors (D) cyclins

A

____ is a globulin that binds thyroid hormones (A) TBG (thyroxine-binding globulin) (B) albumin (C) transferrin (D) Ferritin (E) ceruloplasmin

A

____ is dependent on protein concentration of interstitial fluid, which is normally very low b/c very little protein is filtered. Increases in this factor ______ filtration out of capillary (A) πi (interstitial fluid oncotic pressure), favors (B) Pi (interstitial fluid hydrostatic P), favors (C) πi (interstitial fluid oncotic pressure), opposes (D) Pi (interstitial fluid hydrostatic P), opposes

E

____ is found in ______ and is a scaffold for subsequent ECM deposition, angiogenesis, & reepithelization in healing wounds (A) fibronectin, basement membrane (B) integrin, basement membrane (C) laminin, basement membrane (D) laminin, interstitial ECM (E) fibronectin, interstitial ECM

A

____ release is episodic and under the influence of the hypothalamus, with 2/3 of total 24-hr secretion occurring at night (A) Growth Hormone (GH) (B) LH (C) cortisol (D) GnRH (E) Estrogen (F) Testosterone

B

____, such as flutamide are denoted by the suffix "-utamide" and are used to treat prostate cancer in combination with GnRH antagonists (A) inhibitors of testosterone synthesis & secretion (B) Androgen receptor agonists (C) 5α-reductase inhibitors (D) SERMs

C

_____ & _____ are active during inflammatory response & in signal transduction, while _____ is the lysophospholipase that removes 2nd acyl group after action of PLA1 or PLA2 (A) PLA2 & phospholipase C, LPLAT (B) PLA2 & PLB; phospholipase C (C) PLA2 & phospholipase C; PLB (D) LPLAT & PLA2; PLA1

A

_____ are live viral vaccines that induce innate, humoral, & cell-mediated responses similar to those induced by pathogen itself (A) attenuated vaccines (B) inactivated vaccines (C) subunit vaccines (D) all of the above

A

_____ describes the volume of blood a vessel can hold at a given pressure (= V/P) The higher it is, the more volume it can hold at a given pressure (A) Compliance (Capacitance) (B) Shear (C) Flow (Q) (D) Resistance (R) (E) Elasticity

B

_____ distribute cholesterol to peripheral tissues. _____ removes cholesterol from cells (A) LDL; VLDL & HDL (B) LDL & VLDL; HDL (C) HDL; LDL & VLDL (D) HDL & VLDL; LDL

D

_____ gives tissues the ability to recoil & recover shape, while ______ imparts tensile strength (A) nonfibrillar collagens, elastin (B) elastin, proteoglycans (C) fibronectin, integrins (D) elastin, fibrillar collagens

D

_____ hormones have a critical effect on normal skeletal & CNS development (A) Anterior pituitary (B) Posterior pituitary (C) Adrenal (D) Thyroid (E) Hypothalamus

B

_____ is condensed mesenchyme separating the occipital lobe from the cerebellum (A) falx cerebri (B) tentorium cerebelli (C) 4th ventricle (D) pontine nuclei

C

_____ is found in ______ and connects cells to underlying ECM components - mediates attachment & modulates cell proliferation, differentiation, & motility (A) fibronectin, basement membrane (B) integrin, basement membrane (C) laminin, basement membrane (D) laminin, interstitial ECM (E) fibronectin, interstitial ECM

D

_____ is found in the mitochondria, while _____ is found in the mitochondria & cytoplasm, and is therefore more sensitive due to its cytoplasmic location. Both increase in liver disease. (A) AST, ALP (B) ALP, AST (C) ALP, ALT (D) AST, ALT

E

_____ is located on the extracellular side of the plasma membrane and is involved in cell-cell and cell-matrix interactions. (A) phosphatidylserine (B) phosphatidylinositol (C) glycolipids (D) sphinglomyelin (E) C & D

A

_____ is located on the inner surface of the plasma membrane → negative charge involved in electrostatic interactions. If flipped to extracellular side → "eat me" signal for phagocytes (A) phosphatidylserine (B) phosphatidylinositol (C) glycolipids (D) sphinglomyelin (E) C & D

B

_____ is located on the inner surface of the plasma membrane. If phosphorylated it is an electrostatic binding site for intracellular proteins. If hydrolyzed it generated intracellular 2nd signals. (A) phosphatidylserine (B) phosphatidylinositol (C) glycolipids (D) sphinglomyelin (E) C & D

hCG (human chorionic gonadotropin)

_____ is secreted by the gestational trophoblast and maintains the corpus luteum until ~ 9 weeks of pregnancy, when the trophoblast can produce progesterone

D

_____ is the major iron storage protein found in almost all cells of the body (A) TBG (thyroxine-binding globulin) (B) albumin (C) transferrin (D) Ferritin (E) ceruloplasmin

C

_____ is the total volume ejected by the ventricle per unit time =stroke volume x HR (A) stroke volume (B) ejection fraction (C) cardiac output (D) afterload

hypathalamo-anterior-pituitary

_____-_____-______- axis is a critical link between the brain and endocrine glands which orchestrates synthesis & action of thyroid hormones, glucocorticoids, sex steroids, growth hormone, and prolactin

A

______ & _______ _______ of cytochrome P-450 enzymes underpin mechanisms of drug interactions (A) induction & competitive inhibition (B) nonsense & frameshift mutations (C) reversible & non-competitive inhibition (D) all of the above

D

______ & _______ are 2 peptide hormones synthesized in the cell bodies of the hypothalamic neurons that are subsequently secreted by the posterior pituitary (A) oxytocin & ACTH (B) prolactin & vasopressin (ADH) (C) prolactin & ACTH (D) oxytocin and vasopressin (ADH)

fosphenytoin

______ (water-soluble analogue) is useful in adult patients with partial or generalized seizures that require IV or intramuscular administration

amino acids

______ ______ are a source of energy from the diet & during fasting

banana sign

______ _______ is a flattened and curved shape of cerebellar hemispheres seen in axial section in sonography of fetal skull, which is a sign of Arnold-Chiari deformity as seen in Savanna Murphy's baby

fossa ovalis

______ _______ is an oval-shaped depression in the interatrial septum that represents the site of the foramen ovale

B

______ acts to prevent another LH surge by secreted estrogen during the luteal phase (A) estradiol (B) progesterone (C) FSH (D)LH

A

______ are proteins produced in response to foreign substances and secreted by B cells (A) immunoglobulins (B) immunogens (C) antigens (D) antibodies

Right, Left

______ axis deviation on an ECG points towards a pulmonary problem ______ axis deviation on an ECG is associated more with a heart problem

C

______ can inhibit osteoclast activity → lower blood Ca2+ levels (A) GH (somatrotropin) (B) PTH (C) Calcitonin (D) Ca2+

A

______ circulates unbound in plasma, has a half-life or ~ 10 minutes, is increased by stress, and is inhibited by negative feedback of cortisol (A) ACTH (B) CRH (C) Cortisol (D) GnRH (E) TRH

C

______ contract to tighten the chordae tendinae → prevent the cusps of the tricuspid valve from being everted into atrium by the P developed by pumping action of the heart → prevents regurgitation of ventricular blood into R atrium (A) conus arteriosus (B) pectinate muscles (C) papillary muscles (D) trabeculae carnae cordis

D

______ do not have nuclei or intracellular organelles, have 120 day lifespans, & function to transport O2 and remove CO2 & H+ (A) platelets (B) neutrophils (C) lymphocytes (D) RBCs

C

______ hormones, like IL-2, act on cells of their synthetic origins and can influence their own hormonogenesis (A) Endocrine (B) Paracrine (C) Autocrine

B

______ increase the expression of LDL receptor gene & affect fatty acid synthesis (A) HMG-CoA reductase (B) SREBPs (C) SCAP (D) LXRs

B

______ indirectly stimulates osteoclasts to elevate blood levels of Ca2+ (A) GH (B) PTH (C) Calcitonin (D) Ca2+

D

______ is a 198-AA polypeptide hormone secreted solely by lactotrope cells of the anterior pituitary (A) oxytocin (B) estradiol (C) testosterone (D) prolactin (E) hCG

C

______ is a 28-AA peptide hormone with a fatty acid chain that is also a potent inducer of GH secretion (A) GnRH (B) CRH (C) Ghrelin (D) Somatostatin (GHIH) (E) GHRH

A

______ is a 39-AA polypeptide synthesized from a 241-AA precursor molecule, pro-opiomelanocortin. Released from anterior pituitary. (A) ACTH (B) CRH (C) Cortisol (D) GnRH (E) TRH

D

______ is a full estrogen receptor antagonist which binds competitively to estrogen receptor → change in conformation of receptor → reduced estrogen binding (A) Raloxifene (B) Dihydrotestosterone (C) Clomiphene (D) Fulvestrant

C

______ is a globulin that transports iron, preventing the toxic effects of Fe3+ (A) TBG (thyroxine-binding globulin) (B) albumin (C) transferrin (D) Ferritin (E) ceruloplasmin

C

______ is a non-steroidal drug used in treatment of anovulatory fertility that inhibits negative feedback of estrogen on hypothalamus & pituitary → increase in release of gonadotropins → ovulation (A) Raloxifene (B) Dihydrotestosterone (C) Clomiphene (D) Fulvestrant

B

______ is a plasma protein that serves as an osmotic regulator and is a major transport protein (A) TBG (B) albumin (C) transferrin (D) Ferritin (E) ceruloplasmin

A

______ is a protease secreted by the kidney that regulates BP by cleaving circulating angiotensinogen to angiotensin I (A) renin (B) aldosterone (C) erythropoietin (D) ADH

B

______ is an enzyme released from cardiac tissue & hepatocellular cells whenever there is cell lysis used to diagnose liver damage (A) ALT (Alanine Transaminase) (B) AST (Aspartate Transaminase) (C) Alkaline phosphatase (D) ALP (Alkaline Phosphatase)

A

______ is closed during ventricular systole and is most audible over the right lower part of the body of the sternum (A) Tricuspid (Right AV) valve (B) Pulmonary valve (C) Mitral (Biscuspid/Left AV) valve (D) Aortic Valve

GnRH (gonadotropin-releasing hormone)

______ is essential for secretion of FSH & LH

MALT, lymph nodes

______ is found in the mucosa of most tracts but is concentrated in the palatine, lingual, & pharyngeal tonsils, Peyer patches, & the appendix, whereas _________ ________ are completely encapsulated & occur along the lymphatic vessels; each has several afferent lymphatics & ONE efferent lymphatic

D

______ is made up of anterior & posterior roots of spinal nerves & the filum terminale and is enclosed in the subarachnoid space until S2. It is the site of spinal taps. (A) ligamentum denticulate (B) cervical enlargement (C) tela choiroidea (D) cauda equina

E

______ is released when atrial receptors detect decrease in blood V as in hemorrhage → causes both vasoconstriction & increased water reabsorption → increase BP (A) norepinephrine (B) epinephrine (C) angiotensin II (D) Aldosterone (E) ADH

B

______ is secreted by the hypothalamus into hypophyseal portal blood → acts via G protein-coupled receptors → activates cAMP → ACTH synthesis & secretion (A) ACTH (B) CRH (C) Cortisol (D) GnRH (E) TRH

growth hormone (GH)

______ is synthesized by the somatropic cells of the anterior pituitary & stored within granules. Its overall function is to promote growth of bone, cartilage, & soft tissue

A

______ is the clinical presentation of hypercortisolism - usually iatrogenic, caused by exogenous glucocorticoid therapy (e.g. prednisolone, dexamethasone) (A) Cushing Syndrome (B) Acromegaly (C) Marfan Syndrome (D) Grave's Disease (E) Addison's Disease

B

______ reposition nucleosomes on DNA to expose or hide gene regulatory elements (A) DNA methylation (B) Chromatin remodeling complexes (C) chromatin writer complexes (D) phosphorylation

A

______ should not be taken while pregnant as it can cause fetal hydantoin syndrome, which is characterized by cleft lip, cleft palate, congenital heart disease, delay in psychomotor development, pre- and post-natal growth deficiencies, impaired intellectual performance, & genitourinary & skeletal deformations (A) Phenytoin (B) Phenobarbital (C) Carbamazepine (D) Valproic Acid (E) Ethosuximide

C

______ therapy → increased risk of thromboembolic disorders & stroke; gallbladder disease; cardiovascular risk High _____ levels → blood clots, heart attacks & strokes; changes in vaginal bleeding, dry mouth, frequent urination, loss of appetite, unusual thirst (A) progestin, estrogen (B) androgens, estrogen (C) estrogen, progestin (D) estrogen, androgens

TSH

______ → uptake of iodide into thyroid follicular epithelium, Tg synthesis & secretion, iodination of Tg tyrosine residues, & T3 & T4 secretion into circulation

A

______ → ↑ in TSH → enlargement of thyroid (goitrous hypothyroidism) (A) iodine deficiency (B) primary hypothyroidism (C) secondary hypothyroidism (D) congenital hypothyroidism

D

_______ = cardiac output x aortic pressure (A) stroke volume (B) stroke work (C) cardiac output (D) cardiac minute work

C

_______ are any agents that can be bound by an antibody (A) immunoglobulins (B) immunogens (C) antigens (D) antibodies

B

_______ are cell adhesion molecules which allow cells to attach to ECM constituents → functionally & structurally link cytoskeleton with outside world (A) Cadherin (B) Integrins (C) Fibronectin (D) Laminin

B

_______ are foreign substances that elicit a response from B cells (A) immunoglobulins (B) immunogens (C) antigens (D) antibodies

D

_______ are glycosphingolipids containing sialic (N-acetylneuraminic) acid (A) sphingomyelin (B) neutral glycolipids (C) sulfatides (D) gangliosides

B

_______ binds to chromatin → restricts RNA polymerase access to coding genes → repression (A) RISC (B) lncRNA (C) XIST (D) CRISPR

B

_______ catalyzes conversion of angiotensin I to angiotensin II (vasoconstrictor) in the ______ (A) ACE, plasma (B) ACE, lungs (C) Renin, plasma (D) Renin, lungs

C

_______ catalyzes conversion of angiotensinogen to angiotensin I in the ________ (A) ACE, plasma (B) ACE, lungs (C) Renin, plasma (D) Renin, lungs

D

_______ causes both arteriolar dilation & venous constriction, which together produce a large increase in Pc & local edema (A) Antihistamine (B) ADH (C) Aldosterone (D) Histamine

B

_______ causes the Korotkoff sounds of BP (A) laminar flow (B) turbulent flow (C) thrombi (D) mitral valve closure

veins, arteries

_______ contain unstressed volume - large V under low P ______ contain stressed volume - small V under high P

B

_______ has a simple squamous parietal layer of glomerular (Bowman) capsule, continuous w/ the proximal tubule, & a specialized visceral layer of podocytes surrounding glomerular capillaries (A) Glomeruli (B) Renal corpuscle (C) Renal tubule (D) Collecting Duct

D

_______ invaginate cardiac muscles at Z lines, carrying action potentials to the cell interior (A) Actin filaments (B) Tropomyosin I (C) sarcomeres (D) transverse (T) tubules

E

_______ is a 44-AA peptide synthesized in the arcuate & ventromedial hypothalamic nuclei of the hypothalamus (A) GnRH (B) CRH (C) Ghrelin (D) Somatostatin (E) GHRH

TSH (thyroid stimulating hormone)

_______ is a glycoprotein heterodimer consisting of an alpha- and beta- subunit; ~ 15% carbohydrate by weight that is released by the anterior pituitary

D

_______ is a major component of the acute-phase response and is a marker of bacterial infection (A) Albumin (B) Transferrin (C) IgM (D) C-reactive protein (CRP)

C

_______ is a medical condition describing an excess of amniotic fluid in the amniotic sac (> 2,000 mL) (A) oligohydramnios (B) IUGR (C) polydramnios (D) preeclampsia

TRH (thyroid-releasing hormone)

_______ is a tripeptide released in peptidergic hypothalamic nuclei & transported to the anterior pituitary via portal circulation → stimulates TSH synthesis & secretion

F

_______ is an anabolic hormone that increases muscle mass by stimulating protein synthesis (A) FSH (B) LH (C) cortisol (D) GnRH (E) Estrogen (F) Testosterone

D

_______ is an autoimmune disorder that is the most common cause of hyperthyroidism. It involves the production of TSH-stimulation IgG antibodies and can present with an enlarged thyroid goiter (A) Cushing Syndrome (B) Acromegaly (C) Marfan Syndrome (D) Grave's Disease (E) Addison's Disease

A

_______ is an important "window" on metabolism, which is why it is used for many diagnostic tests (A) plasma (B) urine (C) platelets (D) erythrocytes

E

_______ is characterized by attacks of chest pain originating in the heart & felt beneath the sternum, often radiating to L shoulder & down the arm Caused by insufficient supply of O2 to the heart muscle b/c of coronary artery disease or exertion or emotion (A) MI (B) Cardiac tamponade (C) Pericarditis (D) Pericardial Effusion (E) Angina Pectoris

C

_______ is closed slightly before the tricuspid valve by ventricular systole → S1 (A) Tricuspid (Right AV) valve (B) Pulmonary valve (C) Mitral (Biscuspid/Left AV) valve (D) Aortic Valve

A

_______ is elevated in children with inborn errors of methionine metabolism or people with dietary deficiency of vitamins B6, B12, or folate (A) homocysteine (B) cysteine (C) phenylketones (D) albumin

B

_______ is formed after N-acetylation of sphingosine, and is the precursor & backbone of both sphinomyelin & glycosphingolipids (A) amide (B) ceramide (C) glycerol (D) sulfatide

B

_______ is opened by ventricular systole, shuts tightly after closure of aortic valve, & is most audible over left 2nd intercostal space (A) Tricuspid (Right AV) valve (B) Pulmonary valve (C) Mitral (Biscuspid/Left AV) valve (D) Aortic Valve

atresia, stenosis

_______ means that an opening is closed or absent. ______ describes a partial narrowing or an opening or passage

sympathetic, parasympathetic

_______ nervous system → increase in conduction velocity (positive dromotropic effect) _______ nervous system → decrease in conduction velocity (negative dromotropic effect)

A

_______ of the H chain is responsible for immunoglobulin functions OTHER than epitope recognition, such as complement activation (A) constant region (Fc) (B) antibody recognition region (Fab) (C) variable (V) region (D) J chain

C

_______ of the telencephalon develop as bilateral evaginations of the lateral walls of the prosencephalic vesicle (A) cerebellum (B) pons (C) cerebral hemispheres (D) cerebral cortex

C

_______ protect the body from infection (A) platelets (B) RBCs (C) WBCs (D) plasma

C

_______ regulates cholesterol efflux from cells & increases fatty acid synthesis (A) SCAP/SREBP complex binding to Insig-1 (B) HMG-CoA reductase (C) SREBP1c (D) liver X receptors (LXRs)

B

_______ which is most commonly caused by the acquisition of one extra copy of the X chromosome in each cell (47, XXY) has a prevalence of 1 in 500-1,000 of all phenotypic males (A) Cushing Syndrome (B) Klinefelter Syndrome (C) Marfan Syndrome (D) Down Syndrome

C

_______, such as Finasteride, block conversion of testosterone to dihydrotestosterone (A) inhibitors of testosterone synthesis & secretion (B) Androgen receptor agonists (C) 5α-reductase inhibitors (D) SERMs

kinases, phosphatases

________ add charged phosphate residues to target molecules and usually stimulate activity ________ usually play an inhibitory role in signal transduction by removing the charged phosphate residue

A

________ are fragments derived from megakaryocytes (A) platelets (B) RBCs (C) WBCs (D) plasma

megakaryocytes, proplatelets

________ are large polyploid cells of red bone marrow that produce platelets by releasing them from the ends of cytoplasmic processes called _______

`red, yellow

________ bone marrow is the site of hematopoiesis whereas _______ bone marrow contains adipocytes

A

________ can be used if seizure focus is well localized and presents in noncritical brain regions (A) Surgery (B) Responsive neurostimulation (RNS) (C) Dietary modifications (D) Deep Brain Stimulation (DBS)

B

________ cells are working cells while _______ cells rapidly spread action potentials over myocardium (A) myocardium, contractile (B) contractile, conducting (C) conducting, myocardium (D) contractile, pacemaker

E

________ enter circulation and lose all polyribosomes, at which point they become erythrocytes (A) Proerythroblasts (B) Basophilic erythroblasts (C) Polychromatophilic erythroblast (D) Orthochromatophilic erythroblasts (normoblasts) (E) Reticulocytes

D

________ give prokaryotes a form of acquired immunity to phages & plasmids (A) RISC (B) Dicer (C) XIST (D) CRISPR & Cas

C

________ has multiple actions in adipose tissue, acting to induce lipogenic genes & adipose endocrine function (A) ACTH (B) ADH (C) Cortisol (D) Gonadotropin (E) TSH

E

________ is a globulin that is the major transport protein for copper (A) TBG (thyroxine-binding globulin) (B) albumin (C) transferrin (D) Ferritin (E) ceruloplasmin

B

________ is a nonspecific response to tissue injury or infection (A) chronic phase response (B) acute phase response (C) initial response (D) adaptive response

A

________ is a partial agonist at estrogen receptors that stimulates estrogen receptors in bone & inhibits estrogen receptors in breast & other tissues (A) Raloxifene (B) Dihydrotestosterone (C) Clomiphene (D) Fulvestrant

C

________ is made up of 3 isoprene units and is the branching point for synthesis of dolichol (substrate for glycoprotein synthesis) & ubiquinone (A) mevalonic acid (B) lanosterol (C) Farnesyl pyrophosphate (D) Squalene

liver

________ is the largest organ in the body & has a substantial reserve metabolic capacity

cholesterol

________ is the precursor of ALL steroid hormones

C

________ of TFs directly or indirectly recruit histone-modifying enzymes, chromatin-remodeling complexes, & RNA polymerase (→ RNA synthesis) (A) DNA-binding domains (B) RNA-binding domains (C) protein-protein interaction domains (D) kinase domains

B

________ of hydrocortisone given daily for more than a week → glucocorticoid toxicity likely, meaning suppression of _______ from pituitary → suppression of adrenal corticosteroid production (A) > 10 to 20 mg, aldosterone (B) > 20 to 30 mg, ACTH (C) > 40 to 50 mg, ACTH (D) > 80 to 100 mg, aldosterone

B

________ transports fatty acids, bilirubin, & drugs (A) TBG (B) albumin (C) transferrin (D) Ferritin (E) ceruloplasmin

A

_________ are characterized by accumulation of undigested lipids in inclusion bodies in cells (A) Sphingolipidoses (B) Blood disorders (C) gangliosidoses (D) A & C

B

_________ are used for treatment of testosterone deficiency in adolescent boys & men (A) progesterone (B) androgens (C) estrogen (D) progestin

C

_________ can produce all cell types and are derived from the inner cell mass of the blastocyst when the developing embryo is implanted into uterus (A) totipotent stem cells (B) hemapoietic stem cells (C) embryonic stem cells (D) syncytiotrophoblasts

B

_________ is a prodrug, meaning it requires metabolic reduction to 10-hydroxy metabolite before becoming active (A) Valproic acid (B) Oxcarbazepine (C) Benzodiazepine (D) Ethosuximide

VP (ventriculoperitoneal) shunt

_________ is a surgical procedure that primarily treats hydrocephalus that shunts excess CSF from the cerebral ventricles into the peritoneal cavity where it is absorbed

B

_________ is characterized by presence of sclerotic plaques containing cholesterol & lipoid material → impairs myocardial blood flow → ischemia & MI (A) Coronary angioplasty (B) Coronary atherosclerosis (C) Pericarditis (D) Heart block (E) Coronary bypass

D

_________ is closed during ventricular diastole → S2 and is most audible over the right 2nd intercostal space (A) Tricuspid (Right AV) valve (B) Pulmonary valve (C) Mitral (Biscuspid/Left AV) valve (D) Aortic Valve

cholesterol

_________ is essential for cell structure & function. It is a precursor for steroid hormones, vitamin D, & bile acids

AV valves

_________ maintain one-way blood flow from the atrium to the ventricles

B

_________ of hormones is key to their role as controllers of homeostasis (A) stimulation (B) inactivation (C) structural modification (D) temperature dependence

D

_________ participate in bile synthesis & secretion (A) liver Y receptors (LYRs) (B) HMG-CoA reductase (C) SCAP (D) liver X receptors (LXRs)

anencephaly

_________ results from failure of anterior neuropore to close (rostral end of neural tube fails to develop)

C

_________ studies the effects of genetic heterogeneity on drug responsiveness (A) Pharmacokinetics (B) Pharmacodynamics (C) Pharmacogenomics (D) Pharmacometabolics

B

__________ extend large primary processes that curve around a capillary & exert short, interdigitating secondary processes (pedicels) b/w which are narrow spaces called slit pores (A) mesangial cells (B) podocytes (C) parietal epithelial cells of Bowman's capsule (D) juxtaglomerular cells

glycosphingolipids

__________ function as receptors for cell-cell recognition & interactions and as binding sites for symbiotic & pathogenic bacteria & for viruses

D

__________ is a linear molecule capable of a ring formation (A) mevalonic acid (B) lanosterol (C) Farnesyl pyrophosphate (D) Squalene

B

__________ is an adjunctive treatment, where leads are placed on foci → recording and stimulation of specific brain region (A) Surgery (B) Responsive neurostimulation (RNS) (C) Dietary modifications (D) Deep Brain Stimulation (DBS)

C

__________ is an enzyme found in osteoblasts that is used to diagnose bone disease or bone tumors. (A) ALT (Alanine Transaminase) (B) AST (Aspartate Transaminase) (C) ALP (Alkaline Phosphatase)

B

__________ is important for development, maintenance of steady-state tissue homeostasis, & replacement of dead or damaged cells (A) apoptosis (B) cell proliferation (C) cell-cell interaction (D) laminin

E

__________ is preferred for long-term use because they cause less disturbance of electrolyte & fluid balance due the greater ratio of glucocorticoid to mineralocorticoid (A) short-acting (B) intermediate-acting (C) long-acting (D) A & B (E) B & C

G

__________ is the longest phase of the cardiac cycle. Time required for this phase depends on HR (A) Atrial Systole (B) Isovolumetric Ventricular Contraction (C) Rapid Ventricular Ejection (D) Reduced Ventricular Ejection (E) Isovolumetric Ventricular Relaxation (F) Rapid Ventricular Filling (G) Reduced Ventricular Filling or Diastasis

gain-of-function

__________ mutation in proto-oncogenes → converted into oncogenes → uncontrolled cell proliferation & tumor formation

glucocorticoids

___________ are corticosteroids that have greater effects on carbohydrate metabolism

B

___________ esterifies cholesterol in CELLS → CE stored in lipid droplets in ER _________ esterifies cholesterol in PLASMA → CE in lipoproteins (A) Cholesterol-lecithin acyltransferase; Acyl-coA:cholesterol acyltransferase (ACAT) (B) Acyl-coA:cholesterol acyltransferase (ACAT); Cholesterol-lecithin acyltransferase

B

____________ by hormone-receptor complexes in the nucleus seems to be the major biological action of sex steroid hormones but they can also influence production of 2nd messenger systems (A) Regulation of gene translation (B) Regulation of gene transcription (C) Regulation of DNA replication (D) Regulation of neurotransmitter synthesis

C

_________immune tolerance occurs throughout the body when specific immune reactions are suppressed by regulatory T cells that also originate largely in the thymic medulla (A) T cell (B) central (C) peripheral (D) primary

sphingomyelin

_________is the only sphingolipid that contains phosphate and is the major phospholipid in the myelin sheath of nerves. It is important in the developing brain & present in breast milk

D

_______can occur by themselves, or may occur in association with genetic syndromes such as Goldenhar syndrome, hemifacial microsomia, and first and second branchial arch syndrome (A) lack of red reflex (B) anterior fontanelle (C) patent nares (D) preauricular tags or pits

B

amniotic fluid is generated from _________ (A) fetal plasma (B) maternal plasma (C) fetal urine (D) maternal urine

neutral, positive, negative

amount of N excreted = amount consumed in diet → (negative/neutral/positive) nitrogen balance amount of N excreted < amount consumed in diet → (negative/neutral/positive) nitrogen balance amount of N excreted > amount consumed in diet → (negative/neutral/positive) nitrogen balance

hCG

basis of pregnancy testing assayed in maternal blood at day 8 and in maternal urine at day 10

negative feedback

inhibition of hormone production from hormone itself or as a response to action of hormone is known as ______ _______

C

inhibitory neurotransmitter; precursor of heme (A) Arginine (B) Aspartate (C) Glycine (D) Glutamate

endosteum

inside medullary cavity

C

integrins - leukocyte attachment to other cells (A) pathogens & damage to neighboring cells (B) cell-cell contacts (C) cell-ECM contacts (D) secreted molecules (E) paracrine signaling (F) autocrine signaling (G) synaptic signaling (H) endocrine signaling

A

low of falling levels of hCG → (A) miscarriage or nonviable uterine gestation (B) ectopic (tubal) pregnancy (C) multiple gestations or molar pregnancy

F

molecules secreted by a cell affect the same cell → amplify positive or negative feedback response (A) pathogens & damage to neighboring cells (B) cell-cell contacts (C) cell-ECM contacts (D) secreted molecules (E) paracrine signaling (F) autocrine signaling (G) synaptic signaling (H) endocrine signaling

D

most common fetal infection transmitted to the fetus transplacentally and may cause perinatal infection (A) Toxoplasma gondii (TG) (B) Other (syphillis) (C) Rubella (D) Cytomegalovirus infection (CMV) (E) Herpes

D

precursor of catecholamines (dopamine, epinephrine, & norepinephrine) & thyroxine (A) Glutamate (B) Histidine (C) Tryptophan (D) Tyrosine

B

precursor of histamine (mediator of inflammation & NT) (A) Glutamate (B) Histidine (C) Tryptophan (D) Tyrosine

C

precursor of serotonin; precursor of melatonin (A) Glutamate (B) Histidine (C) Tryptophan (D) Tyrosine

A

precursor of urea & nitric oxide (A) Arginine (B) Aspartate (C) Glycine (D) Glutamate

motor, sensory

pyramidal cells are associated with the (sensory/motor) cortex Granular cells are associated with the (sensory/motor) cortex

C

reaction produces an unstable imine intermediate that hydrolyzes spontaneously to yield an α-keto acid & ammonia (A) Transamination (B) Oxidative deamination (C) Removal of a molecule of water by a dehydratase (D) Reductive deamination

C

reflects a generalized hepatocyte dysfunction (A) Prehepatic jaundice (B) Prehepatic hyperbilirubinemia (C) Intrahepatic jaundice (D) Posthepatic jaundice

A

regions of macromolecules recognized by lymphocytes to elicit specific immune response against them (A) antigens (B) antibodies (C) cytokines (D) interleukins

A

respond to damaged cells (danger signals) & foreign invaders (A) pathogens & damage to neighboring cells (B) cell-cell contacts (C) cell-ECM contacts (D) secreted molecules (E) paracrine signaling (F) autocrine signaling (G) synaptic signaling (H) endocrine signaling

B

results from excess production of bilirubin caused by hemolysis or a genetic abnormality in the hepatic uptake of unconjugated bilirubin (A) Cholestatic disease (B) Prehepatic hyperbilirubinemia (C) Intrahepatic jaundice (D) Posthepatic jaundice

cholesterol, phobic

steroids are derived from ________ and are hydro-(philic/phobic)

positive feedback

stimulation of hormone production from hormone itself or as a response to the action of the hormone is known as ______ ___________

vasoconstriction, vasodilation

sympathetic stimulation of alpha 1 receptors → (vasoconstriction/vasodilation) sympathetic stimulation of beta 2 receptors → (vasoconstriction/vasodilation)

A

the ____ is formed by the rhombic lips, which are thickened plates of the mantle layer. The rostral part of this structure is derived from caudal mesencephalon (A) cerebellum (B) pons (C) medulla oblongata (D) forebrain


Kaugnay na mga set ng pag-aaral

Chapter 27: Condominiums and Cooperatives

View Set

Top 200 Drug Interactions/Contraindications (1-200)

View Set

US History Unit 1 Indigenous Roots - Reconstruction

View Set

5. Life Insurance Policy Provisions, Options and Riders

View Set

Prep U for Brunner and Suddarth's Textbook of Medical Surgical Nursing, 13th Edition Chapter 37: Management of Patients With HIV Infection and AIDS

View Set

Nursing I: Fundamentals 2 Practice Test

View Set